Tải bản đầy đủ (.pdf) (88 trang)

Tuyển tập đề thi học sinh giỏi môn Toán lớp 9 tỉnh Phú Thọ từ năm 2007 đến 2019

Bạn đang xem bản rút gọn của tài liệu. Xem và tải ngay bản đầy đủ của tài liệu tại đây (3.74 MB, 88 trang )

<span class='text_page_counter'>(1)</span><div class='page_container' data-page=1>

<b>Sưu tầm và tổng hợp Trịnh Bình </b> TÀI LIỆU TOÁN HỌC


<b>TUYỂN TẬP ĐỀ THI </b>


<b>HSG TOÁN LỚP 9 </b>



</div>
<span class='text_page_counter'>(2)</span><div class='page_container' data-page=2>

<b>Sưu tầm và tổng hợp Trịnh Bình </b> TÀI LIỆU TỐN HỌC
<b>SỞ GIÁO DỤC VÀ ĐÀO TẠO </b>


<b>PHÚ THỌ </b>


<b>KỲ THI CHỌN HỌC SINH GIỎI CẤP TỈNH</b>


<b>Lớp 9 THCS năm học 2007-2008 </b>
<b>MơnTốn </b>


Thời gian làm bài: 150 phút, khơng kể thời gian giao đề
<i>Đề thi có 01 trang</i>


---


<b>Câu 1 (2 điểm) </b>


a) Chứng minh rằng phương trình 3 3


200720082009


<i>x</i> <i>y</i>  khơng có nghiệm ngun.
b) Cho a2008b2008c2008 1 (1) và a2009b2009c2009 1 (2),


Tính giá trị của tổng: a2007b2008c2009



<b>Câu 2 (2 điểm) </b>Giải hệ phương trình


  



  



  



xy 6 4x 5y 1
yz 6 3y 4z 2
zx 6 5z 3x 3


  




 




 <sub></sub> <sub></sub>



<b>Câu 3 (2 điểm) </b>


Cho a, b, c là các số dương thoả mãn điều kiện a2b2c2 3<b>. </b>
Tìm giá trị lớn nhất của tổng P a 3 b 3  c 3


<b>Câu 4 (2 điểm) </b>Cho hai đường tròn đồng tâm (O; R) và (O; r) (R > r). Gọi P là một điểm
cố định trên (O; r) và B là một điểm trên (O; R). Đường thẳng qua P và vng góc với PB


cắt (O; r) tại A, đường thẳng PB cắt (O; R) tại C và cắt (O; r) tại điểm thứ hai D. Chứng
minh rằng khi điểm B di chuyển trên đường trịn (O; R) thì:


a) Tổng AB2<sub> + BC</sub>2<sub> + CA</sub>2<sub> không đổi. </sub>


b) Trung điểm M của đoạn thẳng AB ln ln thuộc một đường trịn cố định.


<b>Câu 5 (2 điểm) </b>Xét các tam giác ABC có chung cạnh BC cố định v| có đỉnh A nằm trên
đường thẳng d cố định song song với BC. Gọi I là một điểm nằm trong tam giác và x, y, z
lần lượt là khoảng cách từ I đến các cạnh BC = a, CA = b, AB = c. Đặt


  a b c
P


x y z


a) Cố định đỉnh A của tam gi{c ABC, x{c định vị trí điểm I để P đạt giá trị nhỏ nhất.
b) X{c định tam gi{c ABC để P đạt giá trị nhỏ nhất, tìm giá trị nhỏ nhất đó.


<b>Hết </b>


Họ và tên thí sinh ... SBD ...
<b>Chú ý:</b> Cán bộ coi thi khơng giải thích gì thêm


</div>
<span class='text_page_counter'>(3)</span><div class='page_container' data-page=3>

<b>Sưu tầm và tổng hợp Trịnh Bình </b> TÀI LIỆU TOÁN HỌC
SỞ GIÁO DỤC VÀ ĐÀO TẠO PHÚ THỌ


<b>KỲ THI CHỌN HỌC SINH GIỎI CẤP TỈNH LỚP 9 THCS </b>


NĂM HỌC 2007-2008



<b>HƯỚNG DẪN CHẤM THI MƠN TỐN </b>


<b>(Đề chính thức, ngày thi: 06 tháng 3 năm 2008) </b>


<b> Một số chú ý khi chấm bài </b>


 Hướng dẫn chấm thi dưới đ}y dựa vào lời giải sơ lược của một cách, khi chấm thi giám
khảo cần bám sát yêu cầu trình bày lời giải đầy đủ, chi tiết và hợp logic.


 Thí sinh làm bài cách khác với Hướng dẫn chấm m| đúng thì tổ chấm cần thống nhất
cho điểm tương ứng với biểu điểm của Hướng dẫn chấm.


<b>Điểm bài thi</b> là tổng c{c điểm thành phần khơng làm trịn số.


<b>CÂU 1 (2 điểm)</b>.


a) Chứng minh rằng phương trình 3 3 


x y 200720082009 khơng có nghiệm
nguyên.


b) Cho 2008 2008 2008


a b c 1 (1) và 2009 2009 2009


a b c 1 (2), tính giá trị của tổng
2007 2008 2009


a b c



<b>ĐÁP ÁN </b> <b>BIỂU ĐIỂM </b>


<b>a)</b> Ta có


(x + y)3<sub> = x</sub>3<sub> + y</sub>3<sub> + 3xy(x + y) = 200720082009 + 3xy(x + y) </sub> 0,25 điểm
Suy ra (x + y)3<sub> chia hết cho 3 v| do đó x + y chia hết cho 3. </sub> <sub>0,25 điểm </sub>
Từ đó có x3<sub> + y</sub>3<sub> = (x + y)</sub>3<sub> - 3xy(x + y) 9. </sub> <sub>0,25 điểm </sub>
Vì x3<sub> + y</sub>3<sub> chia hết cho 9, cịn số 200720082009 khơng chia hết cho 9 nên </sub>


phương trình đã cho khơng có nghiệm ngun. 0,25 điểm


<b>b) </b>Từ (1) có 2009 2008 2009 2008 2009 2008


a 1, b 1, c 1, suy ra a a , b b ,c c . <sub>0,25 điểm </sub>
Do đó 2009 2009 2009 2008 2008 2008


a b c a b c (3) 0,25 điểm
Tõ (1), (2), (3) cã a2009 a2008, b2009 b2008,c2009 c2008 0,25 điểm
Từ 2009 2008



a a a a 1   0 a 1, a0 , do đó a2007<sub> = a</sub>2008<sub>. </sub>


</div>
<span class='text_page_counter'>(4)</span><div class='page_container' data-page=4>

<b>Sưu tầm và tổng hợp Trịnh Bình </b> TÀI LIỆU TỐN HỌC


2007 2008 2009 2008 2008 2008
a b c a b c 1


<b>BÀI 2 (2 điểm). </b>Giải hệ phương trình


  




  



  



xy 6 4x 5y 1
yz 6 3y 4z 2
zx 6 5z 3x 3


  




 




 <sub></sub> <sub></sub>




<b>ĐÁP ÁN </b> <b>BIỂU ĐIỂM </b>


Dễ thấy hệ phương trình nhận (0; 0; 0) làm nghiệm. 0,25 điểm
Ngoài ra, nếu một trong các ẩn nhận giá trị 0 thì các ẩn cịn lại cũng


nhận giá trị 0. Do đó ta chỉ cịn phải tìm các nghiệm (x; y; z) với x, y, z
khác 0.


0,25 điểm



Từ (1) và (2) có x 4x 5y 3xy 4zx 4zx 5yz 3x 5z
z 3y 4z




      


 (4) 0,50 điểm


Thay (4) v|o (3) được zx6.10z x 60 0,25 điểm
Thay x = 60 v|o (4) được 5z = 180, do đó z = 36 0,25 điểm
Thay x = 60 v|o (1) được 60y6 4.60 5y

5y240 y 48 <sub>0,25 điểm </sub>
Nghiệm (x; y; z) của hệ là: (0; 0; 0) và (60; 48; 36) 0,25 điểm


<b>BÀI 3 (2 điểm) </b>


Cho a, b, c là các số dương thoả mãn đ/kiện 2 2 2


a b c 3<b>. </b>Tìm giá trị lớn nhất
của tổng


a 3 b 3  c 3


<b>ĐÁP ÁN </b> <b>BIỂU ĐIỂM </b>


Ta có

a  3

4 2

a3 .4

4 a3 <sub>0,25 điểm </sub>
Suy ra a 3 a 7


4




  0,25 điểm


Tương tự b 3 b 7
4


  và c 3 c 7


4


  <sub>0,25 điểm </sub>


Do đó a 3 b 3 c 3 a b c 21
4
  


      (1) 0,25 điểm


L¹i cã


2 2 2


a 1 b 1 c 1


a , b , c


2 2 2



  


</div>
<span class='text_page_counter'>(5)</span><div class='page_container' data-page=5>

<b>Sưu tầm và tổng hợp Trịnh Bình </b> TÀI LIỆU TỐN HỌC
Do đó


2 2 2


a b c 3


a b c


2


  


   0,25 điểm


Suy ra


2 2 2


2 2 2


a 1 b 1 c 1


21


a b c 21 <sub>2</sub> <sub>2</sub> <sub>2</sub> a b c 45



6


4 4 8


 <sub></sub>  <sub></sub>  <sub></sub>


   <sub></sub> <sub></sub>    <sub></sub>


(2)


0,25 điểm


Từ (1) và (2) ta có: a 3 b 3  c 3 6
Khi a = b = c = 1 c{c đẳng thức xảy ra và 2 2 2


a b c 3
Vậy giá trị lớn nhất của P là 6


0,25 điểm


<b>Câu 4 (2 điểm) </b>Cho hai đường tròn đồng tâm (O; R) và (O; r) (R > r). Gọi P là một điểm
cố định trên (O; r) và B là một điểm trên (O; R). Đường thẳng qua P và vng góc với PB
cắt (O; r) tại A, đường thẳng PB cắt (O; R) tại C và cắt (O; r) tại điểm thứ hai D. Chứng
minh rằng khi điểm B di chuyển trên đường trịn (O; R) thì:


a) Tổng AB2<sub> + BC</sub>2<sub> + CA</sub>2<sub> không đổi. </sub>


b) Trung điểm M của đoạn thẳng AB luôn luôn thuộc một đường tròn cố định.


<b>ĐÁP ÁN </b> <b>BIỂU ĐIỂM </b>



<b>a) </b>Ta có


T = AB2<sub> + BC</sub>2<sub> + CA</sub>2


= (AP2<sub> + PB</sub>2<sub>)</sub><sub>+ (2CP + PD)</sub>2<sub> + ((PA</sub>2 <sub>+ (PD + CD)</sub>2<sub>) </sub>


= AP2<sub> + PB</sub>2 <sub>+ 4CP</sub>2<sub> + 4. CP. PD +PD</sub>2<sub> + PA</sub>2 <sub>+ PD</sub>2<sub> + 2PD.CD + </sub>
CD2


= 2AP2<sub> + 2PD</sub>2<sub> + 6CP</sub>2<sub> +6.CP. PD </sub>
= 8r2<sub> + 6. PC(PC + PD) </sub>


= 8r2<sub> + 6. PC. CD (1) </sub>


0,50 điểm


Gọi E, F là giao điểm của tia CO và (O; r) với E nằm giữa C và O. Khi đó có


</div>
<span class='text_page_counter'>(6)</span><div class='page_container' data-page=6>

<b>Sưu tầm và tổng hợp Trịnh Bình </b> TÀI LIỆU TỐN HỌC


A


E
D
P


O


B C



F




A


D
P


O


B C


M


I
K


<i>Chó ý: Trong hai ý a) và b) làm ý nào tr-ớc cũng đ-ợc</i>


<b>b)</b> Gọi I l| trung điểm của OC, K l| trung điểm của OP, ta có:


2.IK = CP và IK // CP; 2.OM = BD và OM // BD 0,50 điểm
Vì CP = BD nên IK = OM, IK = OM, do đó IKMO l| hình bình hành.


Suy ra


KM = IO = R
2



0,25 điểm


Vì P và O cố định nên K cố định. Do đó, khi điểm B di chuyển trên
đường trịn (O; R) thì M ln ln thuộc đường trịn (K; R


2 ) cố định.


0,25 điểm


<b>Câu 5 (2 điểm) </b>Xét các tam giác ABC có chung cạnh BC cố định v| có đỉnh A nằm trên
đường thẳng d cố định song song với BC. Gọi I là một điểm nằm trong tam giác và x, y, z
lần lượt là khoảng cách từ I đến các cạnh BC = a, CA = b, AB = c. Đặt


  a b c
P


x y z


a) Cố định đỉnh A của tam gi{c ABC, x{c định vị trí điểm I để P đạt giá trị nhỏ
nhất.


b) X{c định tam gi{c ABC để P đạt giá trị nhỏ nhất, tìm giá trị nhỏ nhất đó.


</div>
<span class='text_page_counter'>(7)</span><div class='page_container' data-page=7>

<b>Sưu tầm và tổng hợp Trịnh Bình </b> TÀI LIỆU TỐN HỌC


<b>a)</b> Ta có


  



  


 




S(ABC) S(IBC) S(ICA) S(IAB)


1 1 1


ax by cz


2 2 2


ax by cz
2


0,50 điểm


Suy ra ax + by + cz là hằng số không phụ thuộc vào vị trí điểm I. Ta có






       


  <sub></sub>   <sub></sub>    <sub></sub>  <sub></sub> <sub></sub>  <sub></sub> <sub></sub>  <sub></sub>


 



     


        


2 2 2


2
2 2 2


a b c x y y z z x


ax by cz a b c ab bc ca


x y z y x z y x z


a b c 2ab 2bc 2ca a b c


0,50 điểm


Dấu đẳng thức xảy ra khi và chỉ khi x = y = z, nghĩa l| I l| t}m đường
tròn nội tiếp tam gi{c ABC. Khi đó


 





2
min



a b c
P


2S(ABC)


0,50 điểm




d


z


x


y


A


B


A


C


I


D





<b>b)</b> Từ đề b|i suy ra c{c tam gi{c đang xét có diện tích khơng đổi như
nhau. Lấy D đối xứng với C qua d. Đường thẳng BD cắt d tại A/<sub> . Khi </sub>
đó mọi tam giác có cạnh BC v| đỉnh A thuộc d, ta có


b c CA AB DA AB BD


Dấu đẳng thức có được khi A trùng với A/<sub>, khi đó tam gi{c ABC c}n </sub>
tại A.


0,25 điểm


</div>
<span class='text_page_counter'>(8)</span><div class='page_container' data-page=8>

<b>Sưu tầm và tổng hợp Trịnh Bình </b> TÀI LIỆU TỐN HỌC


 


 


 


 


 




2
2
2
min



a
a 2 h


4
P


</div>
<span class='text_page_counter'>(9)</span><div class='page_container' data-page=9>

<b>Sưu tầm và tổng hợp Trịnh Bình </b> TÀI LIỆU TỐN HỌC
<b>SỞ GIÁO DỤC VÀ ĐÀO TẠO </b>


<b>PHÚ THỌ </b>


<b>KỲ THI CHỌN HỌC SINH GIỎI CẤP TỈNH</b>


<b>Lớp 9 THCS năm học 2008-2009 </b>
<b>MơnTốn </b>


Thời gian làm bài: 150 phút, khơng kể thời gian giao đề
<i>Đề thi có 01 trang</i>


---


<b>Câu 1 (2 điểm) </b>



Tìm nghiệm nguyên d-ơng của ph-ơng trình:

xyz x y z.

<b>Câu 2 (2 điểm) </b>



a)

Giải ph-ơng trình

x3 x2 x 1
3

.




b) Cho các số d-ơng x, y, z thỏa mÃn điều kiện xyz = 100. Tính giá trÞ cđa biĨu thøc


y


x 10 z


A


xy x 10 yz y 1 xz 10 z 10


  


 

.



<b>Câu 3 (2 điểm) </b>



a) Chøng minh r»ng nÕu c¸c sè x, y, z có tổng là một số không âm thì


3 3 3


x y z 3xyz.

b) Cho m, n là các số thỏa mÃn điều kiện

mn 1


2


. Tìm giá trị nhỏ nhất của biÓu thøc



2 2 2 2


2 2 2 2



m n m n


P .


m n m n








<b>Câu 4 (1.5 điểm) </b>



Trong mặt phẳng Oxy, cho đ-ờng thẳng (d) có ph-ơng trình

m 4 x

 

 m 3 y 1

(m là


tham số). Tìm m để khoảng cách từ gốc tọa độ đến -ng thng (d) l ln nht.



<b>Câu 5 (2,5 điểm) </b>



Cho đ-ờng tròn tâm O, đ-ờng kính BC = 2R. Từ điểm P trên tia tiếp tuyến Bt của đ-ờng tròn,


vẽ tiếp tuyến thứ hai PA (A là tiếp điểm) với đ-ờng tròn. Gọi H là hình chiếu của A lên BC, E


là giao điểm của PC và AH. Đ-ờng thẳng qua E và song song với BC cắt OA tại F.



a) Tính AH theo R và khoảng c¸ch d = PO.



b) Chứng minh rằng khi P di chuyển trên tia Bt thì F ln thuộc một cung trũn c


nh.



---H

t---




Họ và tên thí sinh ... SBD ...



</div>
<span class='text_page_counter'>(10)</span><div class='page_container' data-page=10>

<b>Sưu tầm và tổng hợp Trịnh Bình </b> TÀI LIỆU TOÁN HỌC


Sở giáo dục và đào tạo Phú Thọ



<b>H-íng dÉn chÊm thi chän häc sinh giái cấp tỉnh lớp 9 THcs </b>



năm học 2008-2009



<b>môn Toán</b>



<i>(H-ng dẫn chấm thi đề chính thức có 5 trang) </i>


<b>I. Một số chú ý khi chấm bài </b>



H-ớng dẫn chấm thi d-ới đây dựa vào lời giải sơ l-ợc của một cách, khi chấm thi


giám khảo cần bám sát yêu cầu trình bày lời giải đầy đủ, chi tiết và hợp logic.



Thí sinh làm bài cách khác với H-ớng dẫn chấm mà đúng thì tổ chấm cần thống nhất


cho điểm t-ơng ứng với biểu điểm của H-ớng dn chm.



<b>Điểm bài thi</b>

là tổng các điểm thành phần không làm tròn số.



<b>II. Đáp án và biểu điểm</b>


<b>câu 1 (2 điểm) </b>



Tìm nghiệm nguyên d-ơng của ph-ơng trình



xyz x y z

.




<b>Đáp án </b>

<b>biĨu </b>



<b>®iĨm </b>



Ph-ơng trình đã cho t-ơng đ-ơng với



1 1 1


1
xy yzzx

.



0,25 điểm



Không mất tính tổng quát, giả sử

x y z

(*)

0,25 ®iĨm


- NÕu

z3

th×

1 1 1 3<sub>2</sub> 1 1


xy yzzx  z  3

(loại).

0,25 điểm


- Nếu

z2

thì ph-ơng trình đã cho trở thành



2xy  x y 2

.


Hay

2x 1 2y 1



 

5

.



0,25 điểm



Do (*) nên chỉ có tr-ờng hợp 2x - 1 = 5 và 2y - 1 = 1, suy ra x = 3 vµ y =



1

0,25 ®iĨm



- Nếu

z1

thì ph-ơng trình đã cho trở thành




xy  x y 1


x 1 y 1



 

2

.



</div>
<span class='text_page_counter'>(11)</span><div class='page_container' data-page=11>

<b>Sưu tầm và tổng hợp Trịnh Bình </b> TÀI LIỆU TỐN HC


Do (*) nên chỉ có tr-ờng hợp x - 1 = 2 vµ y - 1 = 1, suy ra x = 3 vµ y = 2.

0,25 điểm


Nghiệm là: (3 ; 2 ; 1), (3 ; 1 ; 2), (2 ; 3 ; 1), (2 ; 1 ; 3), (1 ; 3 ; 2), (1 ; 2 ; 3). 0,25 điểm



<b>CÂU 2 (2 điểm) </b>



a) Giải ph-ơng trình

3 2 1


x x x


3

.



b) Cho các số d-ơng x, y, z tháa m·n ®iỊu kiƯn xyz = 100. Tính giá trị của biểu


thức



y


x 10 z


A


xy x 10 yz y 1 xz 10 z 10





.



<b>Đáp án </b>

<b>biĨu </b>



<b>®iĨm </b>



a) Ph-ơng trình đã cho t-ơng đ-ơng với ph-ơng trình


3 3 2


4x x 3x 3x 1


    

0,25 ®iĨm



3

3
4x x 1


  

0,25 ®iĨm



3


x 4 x 1


  


3 <sub>4 1 x</sub>

<sub>1</sub>

0,25 điểm


Nghiệm của ph-ơng trình:

x <sub>3</sub> 1


4 1





0,25 ®iĨm



b) Ta cã

xyz 10

0,25 ®iĨm



xy


x 10 z


A


xy x 10 xyz xy x xz 10 z xyz


  


     

0,25 ®iĨm







xy


x 10 z


A


xy x 10 10 xy x z x 10 xy



  


     

0,25 ®iĨm



A x xy 10


xy x 10 10 xy x x 10 xy


  


   

= 1

0,25 điểm



<b>CÂU 3 (2 ®iĨm) </b>



a) Chøng minh r»ng nÕu c¸c sè x, y, z có tổng là một số không âm thì


3 3 3


x y z 3xyz.

b) Cho m, n là các số thỏa mÃn điều kiện

mn 1


2


</div>
<span class='text_page_counter'>(12)</span><div class='page_container' data-page=12>

<b>Sưu tầm và tổng hợp Trịnh Bình </b> TÀI LIỆU TOÁN HỌC


2 2 2 2


m n m n


P .



m n m n








<b>Đáp án </b>

<b>biểu </b>



<b>®iĨm </b>



a) (1,25 ®iĨm). Ta có






P x y z xyz


x y xy x y z xyz


   


     


3 3 3


3 <sub>3</sub>


3



3 3


0,25 ®iÓm





 



x y z xy x y xyz


x y z x y z x y z xy x y z


 <sub> </sub> <sub></sub>


<sub></sub>   <sub></sub><sub></sub>   <sub></sub>


 


   <sub></sub>     <sub></sub>  


3 <sub>3</sub>


2 <sub>2</sub>


3 3


3

0,25 ®iĨm



 






x y z x y z x y z xy


x y z x y z xy yz zx


 


   <sub></sub>      <sub></sub>


       


2 <sub>2</sub>
2 2 2


3


0,25 ®iĨm


x y z

 

 x y

 

y z

 

z x



1   <sub></sub>  2  2  2<sub></sub>
0


2

(Do gi¶ thiÕt x + y + z

0



)



0,25 ®iĨm




Suy ra

Px3 y3  z3 xyz


3 0

và do đó

x3y3 z3  xyz


3

0,25 ®iĨm



b) Tõ

2 2

2


m n 2mn m n 0

vµ gi¶ thiÕt suy ra

2 2


m n 2mn1

.

0,25 ®iĨm



Do đó



2 2


2 2 2 2 2 2 2 2


2 2 2 2 2 2 2 2 2 2


15 m n


m n m n m n m n


P .


m n m n 16m n m n 16m n





 


 


  <sub></sub>  <sub></sub>


 <sub></sub>  <sub></sub>

0,25 ®iĨm



áp dụng BĐT

a b 2 ab

với a, b không âm, đấu đẳng thức có khi a = b,


ta có.



1 15 17
P


2 4 4


  


KÕt luËn:

min
17
P


4


, đạt đ-ợc khi

m n 1
2


 

.



0,25 điểm




<b>câu 4 (1.5 điểm) </b>



Trong mt phng Oxy, cho đ-ờng thẳng (d) có ph-ơng trình

m 4 x

 

 m 3 y 1

(m là


tham số). Tìm m để khoảng cách từ gc ta n -ng thng (d) l ln nht.



<b>Đáp ¸n </b>

<b>biÓu </b>



</div>
<span class='text_page_counter'>(13)</span><div class='page_container' data-page=13>

<b>Sưu tầm và tổng hợp Trịnh Bình </b> TÀI LIỆU TỐN HỌC


b) Với mọi m, đ-ờng thẳng (d) không đi qua gốc toạ độ O(0; 0).


m = 4, ta có đ-ờng thẳng y = 1, do đó khoảng cách từ O đến (d) là 1


(1).



m = 3, ta có đ-ờng thẳng x = -1, do đó khoảng cách từ O đến (d) là


1 (2).



0,50 ®iĨm



m

4, m

3 thì (d) cắt trục Oy, Ox lần l-ợt tại


A 0; 1


m 3


 


 <sub></sub> 


 




1


B ; 0


m 4


 


 <sub></sub> 


 

.



0,25 điểm



Hạ OH vuông góc với AB, trong tam giác vuông AOB, ta có



1 1


OA , OB


m 3 m 4


 


 


 

2

2 <sub>2</sub> 2


2 2 2



1 1 1 7 1 1


m 3 m 4 2m 14m 25 2 m


OH OA OB 2 2 2


 


          <sub></sub>  <sub></sub>  


 

.



0,50 ®iĨm



Suy ra

2


OH  2 OH 2

(3).



Từ (1), (2), (3) ta có GTLN của OH là

2

, đạt đ-ợc khi và chỉ khi m =

7
2

.


Kết luận: m =

7


2

.



</div>
<span class='text_page_counter'>(14)</span><div class='page_container' data-page=14>

<b>Sưu tầm và tổng hợp Trịnh Bình </b> TÀI LIỆU TON HC

<b>CÂU 5 (2,5 điểm) </b>



Cho đ-ờng tròn tâm O, đ-ờng kính BC = 2R. Từ điểm P trên tia tiếp tuyến Bt của đ-ờng


tròn, vẽ tiếp tuyến thứ hai PA (A là tiếp điểm) với đ-ờng tròn. Gọi H là hình chiếu của A


lên BC, E là giao điểm của PC và AH. Đ-ờng thẳng qua E và song song với BC cắt OA tại



F..



a) Tính AH theo R và khoảng cách d = PO.



b) Chứng minh rằng khi P di chuyển trên tia Bt thì F ln thuộc một cung trũn c


nh.



<b>Đáp án </b>

<b>biểu </b>



<b>điểm </b>




a) Ta cã

2 2


BP d R

<sub>0,25 ®iĨm </sub>



2 2 2 2


BP.BO R 2R


BI.PO BP.BO BI . d R BA . d R


PO d d


       

0,25 ®iĨm



2 4 2


2 2 2 2 2



2 2


4R 4R 2R


AC BC AB 4R (d R )


d d d


      

0,25 ®iĨm





2
2 2


2R 2R


AH.BC AB.AC AH.2R . d R .


d d


    2 2 2


2
2R


AH . d R


d



</div>
<span class='text_page_counter'>(15)</span><div class='page_container' data-page=15>

<b>Sưu tầm và tổng hợp Trịnh Bình </b> TÀI LIỆU TỐN HỌC


b) Ta có AH // PB (vì AH, PB cùng vuông góc víi BC)

EH CH
PB CB


 

(1)

0,25 điểm


Lại có AC // PO (vì AC, PO cùng vuông góc với AB) nên hai tam giác



vuụng AHC và PBO đồng dạng

AH CH
PB BO


 

(2)

0,25 điểm



Mà CB = 2.BO nên AH = 2. EH hay E là trung điểm của AH.

0,25 ®iĨm


EF // OH suy ra F là trung điểm của AO

OF R


2


.

0,25 ®iĨm



Nên F thuộc đ-ờng trịn cố định tâm O bán kính

R


2

.

0,25 ®iĨm



Khi P di chuyển trên tia Bt thì F thuộc cung MN cố định của đ-ờng trịn


tâm O bán kính

R


2

(phần nằm trên nửa mặt phẳng bờ BC chøa tia Bt).



</div>
<span class='text_page_counter'>(16)</span><div class='page_container' data-page=16>

<b>Sưu tầm và tổng hợp Trịnh Bình </b> TÀI LIỆU TỐN HỌC


<b>SỞ GIÁO DỤC VÀ ĐÀO TẠO </b>


<b>PHÚ THỌ </b>


<b>KỲ THI CHỌN HỌC SINH GIỎI CẤP TỈNH</b>


<b>Lớp 9 THCS năm học 2009-2010 </b>
<b>MơnTốn </b>


Thời gian làm bài: 150 phút, khơng kể thời gian giao đề
<i>Đề thi có 01 trang</i>


---


<b>Câu 1 </b>(4 điểm)


a) Chứng minh rằng A = (2n<sub> - 1)(2</sub>n<sub> + 1) chia hết cho 3 với mọi số tự nhiên n. </sub>
b) Tìm số các số nguyên n sao cho B = n2<sub> – n + 13 là số chính phương ? </sub>
<b>Câu 2 </b>(5 điểm)


a) Giải phương trình 2 2


2 3 2 2 4 3


<i>x</i>  <i>x</i>  <i>x</i>  <i>x</i>
b) Giải hệ phương trình


2 2
2 2



1
3 11


<i>x</i> <i>y</i> <i>xy</i>


<i>x</i> <i>y</i> <i>xy</i>


   




  




<b>Câu 3 </b>(3 điểm) Cho ba số x, y, z thoả mãn:


x y z 2010


1 1 1 1


x y z 2010
  





   


 .



Tính giá trị của biểu thức:

2007 2007



2009 2009



2011 2011



<i>P</i> <i>x</i> <i>y</i> <i>y</i> <i>z</i> <i>z</i> <i>x</i>


<b>Câu 4 </b>(6 điểm)


Cho đường tròn (O; R) và dây cung AB cố định, AB = <i>R</i> 2. Điểm P di động trên
dây AB (P khác A và B). Gọi (C; R1) l| đường tròn đi qua P v| tiếp xúc với đường
tròn (O; R) tại A, (D; R2) l| đường tròn đi qua P v| tiếp xúc với đường tròn (O; R)
tại B. Hai đường tròn (C; R1) và (D; R2) cắt nhau tại điểm thứ hai M.


a) Trong trường hợp P không trùng với trung điểm dây AB, chứng minh
OM//CD v| 4 điểm C, D, O, M cùng thuộc một đường tròn.


b) Chứng minh khi P di động trên d}y AB thì điểm M di động trên đường tròn
cố định v| đường thẳng MP ln đi qua một điểm cố định N.


c) Tìm vị trí của P để tích PM.PN lớn nhất ? diện tích tam giác AMB lớn nhất?


<b>Câu 5 </b>(2 điểm)


Cho các số dương x, y, z thoả mãn điều kiện: xy + yz + zx = 670.
Chứng minh rằng: 2 2 2


1


2010 2010 2010


<i>x</i> <i>y</i> <i>z</i>



<i>x</i> <i>yz</i>  <i>y</i> <i>zx</i>  <i>z</i> <i>xy</i>  <i>x</i> <i>y</i> <i>z</i>


</div>
<span class='text_page_counter'>(17)</span><div class='page_container' data-page=17>

<b>Sưu tầm và tổng hợp Trịnh Bình </b> TÀI LIỆU TỐN HỌC


S

GI

Á

O D

C V

À

ĐÀ

O T

O PH

Ú

TH



<b>H</b>

<b>ƯỚ</b>

<b>NG D</b>

<b>Ẫ</b>

<b>N CH</b>

<b>Ấ</b>

<b>M THI CH</b>

<b>Ọ</b>

<b>N H</b>

<b>Ọ</b>

<b>C SINH GI</b>

<b>Ỏ</b>

<b>I C</b>

<b>Ấ</b>

<b>P T</b>

<b>Ỉ</b>

<b>NH L</b>

<b>Ớ</b>

<b>P 9 THCS </b>



N

Ă

M H

C 2009-2010



<b>MÔN TO</b>

<b>Á</b>

<b>N</b>



<i>(Hướng dẫn chấm thi đề chính thức có 6 trang) </i>



<b>I. M</b>

<b>ộ</b>

<b>t s</b>

<b>ố</b>

<b> chú ý khi ch</b>

<b>ấ</b>

<b>m b</b>

<b>à</b>

<b>i </b>



Hướng dẫn chấm thi dưới đây dựa vào lời giải sơ lược của một cách, khi


chấm thi giám khảo cần bám sát yêu cầu trình bày lời giải đầy đủ, chi tiết và


hợp logic.



Thí sinh làm bài cách khác với Hướng dẫn chấm mà đúng thì tổ chấm cần


thống nhất cho điểm tương ứng với biểu điểm của Hướng dẫn chấm.



<b>Điểm bài thi</b>

là tổng các điểm thành phần khụng lm trũn s.


<b>II. Đáp án và biểu điểm</b>



<b>Câu 1 </b>

(4

đ

i

m)



a) Chøng minh r»ng A = (2

n

<sub> - 1)(2</sub>

n

<sub> + 1) chia hÕt cho 3 víi mọi số tự nhiên n. </sub>



b) Tìm số các số nguyªn n sao cho B = n

2

n + 13 là số chính ph-ơng ?



<b></b>

<b>P </b>

<b></b>

<b>N </b>

<b>BI</b>

<b></b>

<b>U </b>



<b>Đ</b>

<b>I</b>

<b>Ể</b>

<b>M </b>


a) Theo giả thiết n là số tự nhiên nên: 2

n

<sub> – 1, 2</sub>

n

<sub> , 2</sub>

n

<sub> + 1 là 3 số tự </sub>



nhiên liên tiếp.

0,5 điểm



Vì tích của 3 số tự nhiên liên tiếp ln chia hết cho 3 nên



(2

n

<sub> - 1).2</sub>

n

<sub>.(2</sub>

n

<sub> + 1) chia hết cho 3 </sub>

0,5 điểm



Mặt khác (2

n

<sub>, 3) = 1 nên </sub>

<sub>2</sub><i>n</i> <sub>1 2</sub>



<i>n</i> <sub>1</sub>

<sub> chia hết cho 3 </sub>



VËy A chia hÕt cho 3 víi mäi sè tù nhiªn n



0,5 điểm


b) Ta thấy B là số chính phương

4B là số chính phương



Đặt 4B = k

2

<sub> (k</sub>

<sub></sub>

<sub>N) thì 4B = 4n</sub>

2

<sub> – 4n + 52 = k</sub>

2 

<sub>(2n-1-k)(2n-1+k) </sub>



=-51



1,0 điểm



Vì 2n-1+k

2n-1-k nên ta có các hệ



</div>
<span class='text_page_counter'>(18)</span><div class='page_container' data-page=18>

<b>Sưu tầm và tổng hợp Trịnh Bình </b> TÀI LIỆU TOÁN HỌC
2 1 1



(1)
2 1 51


<i>n</i> <i>k</i>


<i>n</i> <i>k</i>


  


    


2 1 3
(2)
2 1 17


<i>n</i> <i>k</i>


<i>n</i> <i>k</i>


  


    


2 1 51
(3)



2 1 1


<i>n</i> <i>k</i>


<i>n</i> <i>k</i>


  


    


2 1 17
(4)
2 1 3


<i>n</i> <i>k</i>


<i>n</i> <i>k</i>


  


 


Giải hệ (1), (2), (3), (4) ta tìm ®-ỵc n = -12, n = -3, n = 13, n = 4



Vậy các số nguyên cần tìm là n

12; 3; 4;13

1,0

đ

i

m




<b>Câu 2 </b>

(5 điểm)



a) Giải phương trình:

2 2


2 3 2 2 4 3


<i>x</i>  <i>x</i>  <i>x</i>  <i>x</i>



b) Giải hệ phương trình



2 2
2 2


1
3 11


<i>x</i> <i>y</i> <i>xy</i>


<i>x</i> <i>y</i> <i>xy</i>


   




  




<b>ĐÁ</b>

<b>P </b>

<b>Á</b>

<b>N </b>

<b>BI</b>

<b>Ể</b>

<b>U </b>




<b>Đ</b>

<b>I</b>

<b>Ể</b>

<b>M </b>



a) Ta có:

2

2


2<i>x</i> 4<i>x</i> 3 2 <i>x</i>1  1 1

nên tập xác định của ph-ơng trình là



R

0,5 điểm



Phương trình đã cho tương đương với


2

2


2<i>x</i> 4<i>x</i> 3 4 2<i>x</i> 4<i>x</i>  3 3 0


Đặt

2


2 4 3 1


<i>y</i> <i>x</i>  <i>x</i> 

thì phương trình đã cho trở thành



2


4 3 0


<i>y</i>  <i>y</i> 


 1


3



<i>y</i>
<i>y</i>




 


(thoả mãn điều kiện)



1,0 điểm



Với y = 1 ta có

2 2


2<i>x</i> 4<i>x</i>  3 1 2<i>x</i> 4<i>x</i> 3 1


x = 1



Với y = 3 ta có

2 2


2<i>x</i> 4<i>x</i>  3 3 2<i>x</i> 4<i>x</i> 3 9


1


3


<i>x</i>
<i>x</i>








Vậy ph-ơng trình cã 3 nghiÖm x

1

= 1, x

2

= -1, x

3

=3.



1,0 điểm



b) Hệ đã cho tương đương với




2 2


2 2


11 11


3 11


<i>x</i> <i>xy</i> <i>y</i>


<i>x</i> <i>xy</i> <i>y</i>


 <sub></sub> <sub></sub> <sub></sub>





  


 




2 2


2 2 2 2


1


11 3


<i>x</i> <i>xy</i> <i>y</i>


<i>x</i> <i>xy</i> <i>y</i> <i>x</i> <i>xy</i> <i>y</i>


   





    





</div>
<span class='text_page_counter'>(19)</span><div class='page_container' data-page=19>

<b>Sưu tầm và tổng hợp Trịnh Bình </b> TÀI LIỆU TỐN HỌC

<sub></sub>

<sub></sub>

<sub></sub>



2 2


1
2 5 3 0


<i>x</i> <i>xy</i> <i>y</i>



<i>x</i> <i>y</i> <i>x</i> <i>y</i>


   




   




(*)



Từ hệ (*) ta suy ra




2 2


1
2 0


<i>x</i> <i>xy</i> <i>y</i>


<i>x</i> <i>y</i>


   




 


(I) hoặc




2 2


1
5 3 0


<i>x</i> <i>xy</i> <i>y</i>


<i>x</i> <i>y</i>


   




 


(II)



0,5 điểm



Giải hệ (I) ta tìm được (x; y) = ( 2; -1), (-2; 1)


Hệ (II) vơ nghiệm



Vặy hệ có nghiệm (x; y) = ( 2; -1), (-2; 1).



1,0 điểm



<b>Câu 3 </b>

(3 điểm)

<b> </b>



Cho ba số x, y, z thoả mãn:




x y z 2010


1 1 1 1


x y z 2010
  





   



Tính giá trị của biểu thức:

2007 2007



2009 2009



2011 2011



<i>P</i> <i>x</i> <i>y</i> <i>y</i> <i>z</i> <i>z</i> <i>x</i>



<b>Đáp án </b>

<b>biểu </b>



<b>điểm </b>


T gi thiết suy ra x, y, z khác 0 và



1 1 1 1


x   y z x y z


0,5 điểm



1 1 1 1 0


x y z x y z


   


<sub></sub>  <sub> </sub>  <sub></sub>


 


   


0,5


®iĨm


x y

<sub></sub>

x y

<sub></sub>

0


xy z x y z


 


  


 


x y

1 1 <sub>2</sub> 0


xy xz yz z


 


  <sub></sub>  <sub></sub>



 


 


0,5 điểm



2



x y xz yz z xy 0


     


2



x y  xz z yz xy  0


  <sub></sub>    <sub></sub>

0,5 điểm



xy

 

<sub></sub>z zx

 

y zx

<sub></sub>0


</div>
<span class='text_page_counter'>(20)</span><div class='page_container' data-page=20>

<b>Sưu tầm và tổng hợp Trịnh Bình </b> TÀI LIỆU TỐN HỌC




2007 2007 2007 2007
2009 2009 2009 2009
2011 2011 2011 2011


0 0



0 0


0 0


<i>x</i> <i>y</i> <i>x</i> <i>y</i> <i>x</i> <i>y</i> <i>x</i> <i>y</i>


<i>z</i> <i>y</i> <i>y</i> <i>z</i> <i>y</i> <i>z</i> <i>y</i> <i>z</i>


<i>x</i> <i>z</i> <i>z</i> <i>x</i> <i>z</i> <i>x</i> <i>z</i> <i>x</i>


 


       


 


 


 <sub>  </sub> <sub>  </sub> <sub> </sub> <sub></sub> <sub></sub> <sub></sub>


 


 


 


         


   



nên P = 0

<sub>0,5 điểm </sub>



<b>Câu 4 </b>(6 điểm)


Cho đường tròn (O; R) và dây cung AB cố định, AB = <i>R</i> 2. Điểm P di động
trên dây AB (P khác A và B). Gọi (C; R1) l| đường tròn đi qua P v| tiếp xúc với
đường tròn (O; R) tại A, (D; R2) l| đường tròn đi qua P v| tiếp xúc với đường tròn
(O; R) tại B. Hai đường tròn (C; R1) và (D; R2) cắt nhau tại điểm thứ hai M.


a) Trong trường hợp P không trùng với trung điểm dây AB, chứng minh
OM//CD v| 4 điểm C, D, O, M cùng thuộc một đường tròn.


b) Chứng minh khi P di động trên d}y AB thì điểm M di động trên đường trịn
cố định v| đường thẳng MP luôn đi qua một điểm cố định N.


c) Tìm vị trí của P để tích PM.PN lớn nhất ? diện tích tam giác AMB lớn nhất?


<b>ĐÁP ÁN </b>

<b>BIỂU </b>


<b>ĐIỂM </b>



N
K
H
M


D
C


O



A P B


a) Nối CP, PD ta có

ACP,

OAB lần lượt cân tại C, O nên


CPA =

CAP =

OBP do đó CP//OD (1)



Tương tự

DBP,

OAB lần lượt cân tại D, O nên

DPB =



</div>
<span class='text_page_counter'>(21)</span><div class='page_container' data-page=21>

<b>Sưu tầm và tổng hợp Trịnh Bình </b> TÀI LIỆU TOÁN HỌC

DBP =

OAB



nên OC//DP (2). Từ (1) và (2) suy ra tứ giác ODPC là hình bình


hành



Giả sử CD cắt MP tại H cắt OP tại K thỡ K là trung điểm của OP


Theo tớnh chất 2 đường trũn cắt nhau ta cú CD

MP và H là trung


điểm MP Vậy HK//OM, do đó CD//OM



0,5 điểm



Ta phải xét 2 trường hợp AP < BP và AP > BP, đáp án chỉ yêu cầu


xét 1 trường hợp giả sử AP < BP



Vì tứ giác OCPD là hình bình hành nên OD = CP = CM = R

2

nên



tứ giác CDOM là hình thang cân.



Vậy 4 điểm C, D, O, M cùng thuộc một đường tròn.



0,5 điểm




b) Xét tam giác AOB có

: 2 2 2 2


2


<i>OA</i> <i>OB</i>  <i>R</i>  <i>AB</i>

nên tam giác AOB


vuông cân tại O



Vì 4 điểm C, D, O, M cùng thuộc 1 đường tròn (kể cả M trùng


O) nên

COD =

CMD (1)



0,5 điểm



Xét

MAB và

MCD có



MAB =

MCD (cùng bằng

1


2

<i>MP</i>

của (C))



MBA =

MDC ( cùng bằng

1


2

<i>MP</i>

của (D))



nên

MAB đồng dạng với

MCD (g.g)



0,5 điểm



MAB đồng dạng với

MCD suy ra

AMB =

CMD (2)


Từ (1) và (2) suy ra

AMB =

AOB =

0



90



Do AB cố định nên điểm M thuộc đ-ờng tròn tâm I đ-ờng kính AB



0,5 điểm



Ta có

0


90


<i>ACP</i> <i>BDP</i> <i>AOB</i>


     

nên



AMP =

1


2 

ACP =


0


45

(góc nội tiếp và góc ở tâm của (C))



BMP =

1


2 

BDP =


0


45

(góc nội tiếp và góc ở tâm của (D))



Do đó MP là phân giác

<i>AMB</i>



0,5 điểm



</div>
<span class='text_page_counter'>(22)</span><div class='page_container' data-page=22>

<b>Sưu tầm và tổng hợp Trịnh Bình </b> TÀI LIỆU TỐN HỌC


AOB



Giả sử MP cắt đường tròn (I) tại N thì N là điểm chính giữa của


cung AB khơng chứa điểm O nên N cố định



Vậy MP luôn đi qua điểm N cố định.



0,5 điểm



c)

MAP và

BNP có

MPA =

BPN (đđ),

AMP =

PBN (góc


nội tiếp cùng chắn 1 cung) nên

MAP đồng dạng với

BNP


(g.g)



0,5 điểm



Do đó



2 <sub>2</sub> <sub>2</sub>


. .


2 4 2


<i>PA</i> <i>PM</i> <i>PA PB</i> <i>AB</i> <i>R</i>


<i>PM PN</i> <i>PA PB</i>



<i>PN</i> <i>PB</i>




 


   <sub></sub> <sub></sub>  


 

(không



đổi)



Vậy PM.PN lớn nhất bằng



2
2


<i>R</i>


khi PA = PB hay P là trung điểm


dây AB



0,5 điểm



Vì tam giác AMB vng tại M nên



2 2


2 2



1 1


.


2 4 4 2


<i>AMB</i>


<i>AB</i> <i>R</i>


<i>S</i>  <i>AM BM</i>  <i>AM</i> <i>BM</i>  


Di

n tích tam giác AMB l

n nh

t b

ng


2
2


<i>R</i>


khi PA = PB hay P là trung


điểm dây AB



0,5 điểm



<b>CÂU 5 </b>

(2 điểm)



Cho các số dương x, y, z thoả mãn điều kiện: xy + yz + zx = 670.



Chứng minh rằng:

2 2 2



1


2010 2010 2010


<i>x</i> <i>y</i> <i>z</i>


<i>x</i> <i>yz</i> <i>y</i> <i>zx</i> <i>z</i> <i>xy</i> <i>x</i> <i>y</i> <i>z</i>


<b>Đáp ¸n </b>

<b>biĨu </b>



<b>®iĨm </b>


Trước tiên ta chứng minh bất đẳng thức: Với

a, b, c

R và x, y,



z > 0 ta có





2


2 2 2


<i>a b c</i>


<i>a</i> <i>b</i> <i>c</i>


<i>x</i> <i>y</i> <i>z</i> <i>x</i> <i>y</i> <i>z</i>


 


  



 

(*)



Dấu “=” xảy ra

 <i>a</i> <i>b</i> <i>c</i>


<i>x</i>  <i>y</i> <i>z</i>


Thật vậy, với a, b

R và x, y > 0 ta có



</div>
<span class='text_page_counter'>(23)</span><div class='page_container' data-page=23>

<b>Sưu tầm và tổng hợp Trịnh Bình </b> TÀI LIỆU TỐN HỌC



2
2 2


<i>a b</i>


<i>a</i> <i>b</i>


<i>x</i> <i>y</i> <i>x</i> <i>y</i>




 


(**)



2 2

2
<i>a y b x</i> <i>x</i><i>y</i> <i>xy a b</i>



2
0


<i>bx</i><i>ay</i> 

(luôn đúng)



Dấu “=” xảy ra

 <i>a</i> <i>b</i>
<i>x</i>  <i>y</i>


Áp dụng bất đẳng thức (**) ta có





2 2


2 2 2 2


<i>a b</i> <i>a b c</i>


<i>a</i> <i>b</i> <i>c</i> <i>c</i>


<i>x</i> <i>y</i> <i>z</i> <i>x</i> <i>y</i> <i>z</i> <i>x</i> <i>y</i> <i>z</i>


  


    


  


Dấu “=” xảy ra

 <i>a</i> <i>b</i> <i>c</i>



<i>x</i>  <i>y</i> <i>z</i>


Áp dụng bất đẳng thức (*) ta có



<sub>2</sub> <sub>2</sub> <sub>2</sub>


2010 2010 2010


<i>x</i> <i>y</i> <i>z</i>


<i>VT</i>


<i>x</i> <i>yz</i> <i>y</i> <i>zx</i> <i>z</i> <i>xy</i>


  


     


<sub></sub>

<sub> </sub>

<sub> </sub>

<sub></sub>



2 2 2


2 2 2


2010 2010 2010


<i>x</i> <i>y</i> <i>z</i>


<i>x x</i> <i>yz</i> <i>y y</i> <i>zx</i> <i>z z</i> <i>xy</i>



  


     


<sub></sub>

<sub></sub>



2
3 3 3


3 2010


<i>x</i> <i>y</i> <i>z</i>


<i>x</i> <i>y</i> <i>z</i> <i>xyz</i> <i>x</i> <i>y</i> <i>z</i>


 


     

(1)



Chú ý:

2



2010


<i>x x</i> <i>yz</i>

=

<i>x x</i>

2<i>xy</i><i>zx</i>1340

0

,

<i>y y</i>

2<i>zx</i>2010

0




2




2010 0


<i>z z</i> <i>xy</i> 


0,5 điểm



Chøng minh:

3 3 3

2 2 2



3


<i>x</i> <i>y</i>  <i>z</i> <i>xyz</i> <i>x</i> <i>y</i> <i>z</i> <i>x</i> <i>y</i> <i>z</i> <i>xy</i><i>yz</i><i>zx</i>


<i>x</i> <i>y</i> <i>z</i>

 

<sub></sub> <i>x</i> <i>y</i> <i>z</i>

23

<i>xy</i><i>yz</i><i>zx</i>

<sub></sub>

(2)



Do đó:

3 3 3



3 2010


<i>x</i> <i>y</i>  <i>z</i> <i>xyz</i> <i>x</i>  <i>y</i> <i>z</i>


<i>x</i> <i>y</i> <i>z</i>

 

<sub></sub> <i>x</i> <i>y</i> <i>z</i>

23

<i>xy</i><i>yz</i><i>zx</i>

2010<sub></sub>

=

<i>x</i> <i>y</i> <i>z</i>

3

(3)



0,5 điểm



Từ (1) và (3) ta suy ra








2
3


1


<i>x</i> <i>y</i> <i>z</i>


<i>VT</i>


<i>x</i> <i>y</i> <i>z</i>


<i>x</i> <i>y</i> <i>z</i>


 


 


 
 


Dấu “=” xảy ra

x = y = z =

2010


3

.



</div>
<span class='text_page_counter'>(24)</span><div class='page_container' data-page=24>

<b>Sưu tầm và tổng hợp Trịnh Bình </b> TÀI LIỆU TỐN HỌC
<b>SỞ GIÁO DỤC VÀ ĐÀO TẠO </b>


<b>PHÚ THỌ </b>


<b>KỲ THI CHỌN HỌC SINH GIỎI CẤP TỈNH</b>



<b>Lớp 9 THCS năm học 2010-2011 </b>
<b>MơnTốn </b>


Thời gian làm bài: 150 phút, không kể thời gian giao đề
<i>Đề thi có 01 trang</i>


---


<b>Câu 1 </b>(4 điểm) a) Cho

2



2



x 2011 x y 2011 y 2011.
Tính giá trị của biểu thức : Tx2011y2011.
b) Tính tổng : S = 4 3 8 15 ... 240 14399


1 3 3 5 119 121


 <sub></sub>  <sub> </sub> 


   .


(mỗi số hạng trong tổng trên có dạng


2


4n 4n 1


2n 1 2n 1


 



   , với n N và 1 n 60).


<b>Câu 2 </b>(3 điểm) Giải hệ phương trình:


3 2


3 2


3 2


x 3x 2x 5 y


y 3y 2y 5 z


z 3z 2z 3 x .


    




   




    


<b>Câu 3 </b>(4 điểm)



a) Tìm số nguyên dương n để Bn4n3n2 n 1 là số chính phương.
b) So sánh M và N biết M

2010201020112010

2011, N

2010201120112011

2010.


<b>Câu 4 </b>(2 điểm)<b> </b>Cho a, b, c là các số dương. Tìm giá trị nhỏ nhất của biểu thức


4a b 3c 8c


A


a b 2c 2a b c a b 3c


  


      .


<b>Câu 5 </b>(7 điểm)<b> </b>Cho đường trịn (O; R) đường kính AB. Qua B kẻ tiếp tuyến d của
đường tròn (O). MN là một đường kính thay đổi của đường tròn (M không trùng
với A, B). C{c đường thẳng AM và AN cắt đường thẳng d lần lượt tại C và D.


a) Chứng minh AM.ACAN.AD.
b) Tìm giá trị nhỏ nhất của tích AC.AD.


c) Chứng minh t}m đường tròn ngoại tiếp tam giác MNC thuộc một đường
thẳng cố định.


d) Gọi I là giao điểm của CO v| BM. Đường thẳng AI cắt đường tròn (O) tại
điểm thứ hai là E, cắt đường thẳng d tại F. Chứng minh ba điểm C, E, N thẳng hàng.


</div>
<span class='text_page_counter'>(25)</span><div class='page_container' data-page=25>

<b>Sưu tầm và tổng hợp Trịnh Bình </b> TÀI LIỆU TỐN HỌC


SỞ GIÁO DỤC VÀ ĐÀO TẠO PHÚ THỌ


<b>HƯỚNG DẪN CHẤM THI CHỌN HỌC SINH GIỎI LỚP 9 THCS CẤP TỈNH </b>


NĂM HỌC 2010-2011


<b>MƠN TỐN</b>



<i>(Hướng dẫn chấm thi đề chính thức có 5 trang) </i>


<b>I. Một số chú ý khi chấm bài </b>


 Hướng dẫn chấm thi dưới đ}y dựa vào lời giải sơ lược của một cách, khi chấm thi giám khảo
cần bám sát yêu cầu trình bày lời giải đầy đủ, chi tiết và hợp logic.


 Thí sinh làm bài cách khác với Hướng dẫn chấm m| đúng thì tổ chấm cần thống nhất cho
điểm tương ứng với biểu điểm của Hướng dẫn chấm.


 Tổ chấm có thể chia nhỏ đến 0,25 điểm. <b>Điểm bài thi</b> là tổng c{c điểm thành phần không
làm trũn s.


<b>II. Đáp án và biểu điểm </b>


<b>ỏp ỏn </b> <b>Điểm </b>


<b>Câu 1 </b>(4 điểm)


a) Cho

2



2



x 2011 x y 2011 y 2011. Tính giá trị của biểu thức


2011 2011


Tx y .
b) Tính tổng


S = 4 3 8 15 ... 240 14399


1 3 3 5 119 121


  


  


   .


(mỗi số hạng trong tổng trên có dạng


2


4n 4n 1


2n 1 2n 1


 


   , với n N và 1 n 60).


a) (2 điểm). Từ giả thiết, suy ra


2



2



2

2




2011 2011 2011 2011 2011


        


<i>x</i> <i>x</i> <i>x</i> <i>x</i> <i>y</i> <i>y</i> <i>x</i> <i>x</i> 0,50


2 2


2011 2011


<i>y</i> <i>y</i> <i>x</i> <i>x</i>


      (1) 0,50


Tương tự ta có: 2 2


2011 2011


<i>x</i> <i>x</i>  <i>y</i> <i>y</i>(2) 0,50


</div>
<span class='text_page_counter'>(26)</span><div class='page_container' data-page=26>

<b>Sưu tầm và tổng hợp Trịnh Bình </b> TÀI LIỆU TOÁN HỌC
b) (2điểm). Với k là số tự nhiên khác 0 ta có:


2


4 4 1


2 1 2 1



<i>k</i> <i>k</i>


<i>k</i> <i>k</i>


 


   =








2 3 3


4 4 1 2 1 2 1 <sub>2</sub> <sub>1</sub> <sub>2</sub> <sub>1</sub>


2
2 1 2 1 2 1 2 1


<i>k</i> <i>k</i> <i>k</i> <i>k</i> <i><sub>k</sub></i> <i><sub>k</sub></i>


<i>k</i> <i>k</i> <i>k</i> <i>k</i>


     <sub></sub> <sub></sub> <sub></sub>




     


0,75



Cho k lần lượt nhận các giá trị 1, 2, <, 60. Ta được:


3 3



4 3 1


3 1
2


1 3




 




3 3



8 15 1


5 3
2


3 5




 





<


3 3



240 14399 1


121 119
2


119 121


 <sub></sub> <sub></sub>




0,75


Vậy S = 1

3


121 1 665


2   0,5


<b>Câu 2 </b>(3 điểm)


Giải hệ phương trình


3 2



3 2


3 2


x 3x 2x 5 y


y 3y 2y 5 z


z 3z 2z 3 x .


    




   




    


Viết lại hệ đã cho dưới dạng


3 2


3 2


3 2



x 3x 2x 5 y


y 3y 2y 5 z


z 3z 2z 5 x 2 .


    




   




     


0,25


Đặt t = x – 2 thì x = t + 2, thế v|o phương trình thứ nhất của hệ ta được




3

2



3 2 2


3 2


t 2 3 t 2 2 t 2 5 y



t 6t 12t 8 3t 12t 12 2t 4 5 y
t 3t 2t 5 y


      


          


    


0,50


Khi đó có hệ phương trình
3 2


3 2


3 2


t 3t 2t 5 y
y 3y 2y 5 z
z 3z 2z 5 t


    




   





    


(I) 0,25


</div>
<span class='text_page_counter'>(27)</span><div class='page_container' data-page=27>

<b>Sưu tầm và tổng hợp Trịnh Bình </b> TÀI LIỆU TỐN HỌC
sử t = max

<i>t y z</i>, ,

.


1) Trường hợp <i>t</i> <i>y</i> <i>z</i>. Từ hệ (I) ta có


3 2
3 2


t 3t 2t 5 t
z 3z 2z 5 z


    





   





 



 




2
2


t 1 t 2 1 0
z 1 z 2 1 0


 <sub></sub>  <sub></sub> <sub> </sub>


  


 


 


   


 <sub></sub> <sub></sub>




1
1


<i>t</i>
<i>z</i>





  <sub></sub>





Do đó t = y = z = 1.


0,75


2) Trường hợp t z y. Tương tự ta có: 1
1


<i>t</i>
<i>y</i>




 


Do đó t = y = z = 1.


0,75


Nghiệm của hệ phương trình đã cho l|: (x: y: z) = (3: 1: 1) 0,25


<b>Câu 3 </b>(4 điểm)


a) Tìm số nguyên dương n để B = n4<sub> + n</sub>3<sub> + n</sub>2<sub> + n + 1 là số chính phương. </sub>
b) So sánh M và N biết

2010 2010

2011

2011 2011

2010


M 2010 2011 , N 2010 2011 <sub>. </sub>



a) (2 điểm). Đặt n4<sub> + n</sub>3<sub> + n</sub>2<sub> + n + 1 = k</sub>2<sub> (1) (với k nguyên dương) </sub> <sub>0,25 </sub>
Ta có (1)  4n4<sub> + 4n</sub>3<sub> + 4n</sub>2<sub> + 4n + 4 = 4k</sub>2


 (2n2<sub> +n)</sub>2<sub> +2n</sub>2<sub> +(n+2)</sub>2<sub> = (2k)</sub>2 0,75
 (2k)2<sub> > (2n</sub>2<sub> +n)</sub>2


 (2k)2 <sub> (2n</sub>2<sub> +n+1)</sub>2<sub> (do k v| n nguyên dương) </sub>
 4n4<sub> + 4n</sub>3<sub> + 4n</sub>2<sub> + 4n + 4</sub><sub> (2n</sub>2<sub> +n+1)</sub>2


 (n+1)(n-3)  0
 n  3


 n 

1; 2; 3



0,75


Thay các giá trị của n vào (1), chỉ có n = 3 thoả mãn đề bài. 0,25
b) (2 điểm). Đặt a20102010, b 2011 2010<sub>. Ta có: </sub>


2010


2011 2010


M (a b) , N(2010a +2011b)  2010(a +b)+b 0,50


Xét:









2010 <sub>2010</sub>


2010
2010 a b b


N 1 b


2010


M a b a b a b a b


 


  <sub></sub> <sub></sub>


 


  <sub></sub>  <sub></sub>


   


</div>
<span class='text_page_counter'>(28)</span><div class='page_container' data-page=28>

<b>Sưu tầm và tổng hợp Trịnh Bình </b> TÀI LIỆU TỐN HỌC


2010


2010



b b


1 2010 2011 b


a b a b


 


 <sub></sub>  <sub></sub>  


    0,50


Nên N b 1 N M


M ab   0,50


<b>Câu 4 </b>(2 điểm)


Cho a, b, c là các số dương. Tìm gi{ trị nhỏ nhất của biểu thức


4a b 3c 8c


a b 2c 2a b c a b 3c


  


     


<b>A</b> <sub>. </sub>



Đặt


x a b 2c a y z 2x
y 2a b c b 5x y 3z
z a b 3c c z x


     


 


 <sub></sub> <sub>  </sub> <sub></sub> <sub> </sub>


 


 <sub>  </sub>  <sub> </sub>


 


(x,y,z > 0) 0,50


Khi đó:


4(y z 2x) 2x y 8(z x) 4y 2x 4z 8x


A 17


x y z x y x z


 



     


   <sub></sub>  <sub> </sub>  <sub></sub>


 


 


0,50


Do đó


A2 82 32 17 12 2 17 0,50


Dấu “=” xảy ra khi và chỉ khi: 2y 2x
2z 2 2x


 











4 3 2



a t


2
10 7 2


b t


2
c 2 1 t


  






 <sub></sub>


 



 





(với t  R, t > 0)


</div>
<span class='text_page_counter'>(29)</span><div class='page_container' data-page=29>

<b>Sưu tầm và tổng hợp Trịnh Bình </b> TÀI LIỆU TỐN HỌC



<b>Câu 5 </b>(7 điểm)


Cho đường trịn (O; R) đường kính AB. Qua B kẻ tiếp tuyến d của đường tròn (O).
MN là một đường kính thay đổi của đường tròn (M không trùng với A, B). Các
đường thẳng AM và AN cắt đường thẳng d lần lượt tại C và D.


a) Chứng minh AM.ACAN.AD.
b) Tìm giá trị nhỏ nhất của tích AC.AD.


c) Ch/minh t}m đường tròn ngoại tiếp tam giác MNC thuộc một đường thẳng
cố định.


d) Gọi I l| giao điểm của CO v| BM. Đường thẳng AI cắt đường tròn (O) tại
điểm thứ hai là E, cắt đường thẳng d tại F. Chứng minh ba điểm C, E, N thẳng hàng
Hình vẽ:


E


I


K


D
d


N <sub>O</sub>


P


A P



B


A


M


N


B
C


F


C
M


D


a) (1,5 điểm). Ta có ANMABM, ABMACB. Suy ra: ACBANM
Do đó AMN và ADCđồng dạng.


</div>
<span class='text_page_counter'>(30)</span><div class='page_container' data-page=30>

<b>Sưu tầm và tổng hợp Trịnh Bình </b> TÀI LIỆU TỐN HỌC
AM AN


AM.AC AN.AD


AD  AC   0,75


b) (2 điểm). Ta có: AC.ADCD.AB2R.CD(1) 0,50



Lại có 2


CDBD BC 2 BD.CD2 AB 4R(2) 0,50
Từ (1) và (2), suy ra 2


CD.AD8R <sub>0,50 </sub>


Dấu “=” xảy ra khi MN vng góc với AB 0,50


c) (2 điểm). Gọi P l| t}m đường tròn ngoại tiếp MNC, K l| trung điểm


của CD, S l| giao điểm của AK với MN.


Ta thấy tứ giác MNDC là tứ giác nội tiếp đường tròn tâm P nên
AMNADC, SAMKCAANM. Suy ra: MN vng góc với AK


0,75


Lại có: PO vng góc với MN nên AK song song với OP, mà PK song


song với AO. Suy ra: tứ giác AOPK là hình bình hành, hay KP = AO =R 0,75
Vì d l| đường thẳng cố đinh, PK = R không đổi nên P thuộc đường thẳng


song song với d, cách d một khoảng R cố định. 0,50
d) (1,5 điểm). Trước hết ta chứng minh bài tốn: Nếu tam giác ABC có các


điểm M, N, P thẳng hàng và lần lượt thuộc c{c đường thẳng AB, BC, CA
thì:



AP CN BM
. .


PC NB MA = 1.


Thật vậy: Qua C kẻ đường thẳng song song với AB cắt MN tại D, ta có:


AP AM


PC  CD và


CN CD


NB  BM.
Do đó ta có điều phải chứng minh.


0,50


Áp dụng bài toán trên vào tam giác ACO với ba điểm thẳng hàng là B, I,
M, ta có:


AB OI CM


. . 1


BO IC MA  


OI MA


IC  2CM(1)



Tương tự với tam gi{c BCO v| ba điểm thẳng hàng là A, I, F ta có:


OI FB


IC  2CF(2)


0,25


Từ (1) và (2) ta có MA=FB


</div>
<span class='text_page_counter'>(31)</span><div class='page_container' data-page=31>

<b>Sưu tầm và tổng hợp Trịnh Bình </b> TÀI LIỆU TOÁN HỌC
Mà AB BC  MF BC  0


90


<i>MFC</i>


Ta có EFBEBA(cùng phụ với góc EAB)
EBAEMC (tứ giác AMEB nội tiếp)
EFBEMC  Tứ giác MEDC nội tiếp


0,25


 0


MECMDC90 . Do đó: ME  EC (3)


Lại có 0



MEN90 (góc nội tiếp chắn nửa đường trịn)  ME  EN (4)
Từ (3) và (4) suy ra M, E, N thẳng hàng.


0,25


</div>
<span class='text_page_counter'>(32)</span><div class='page_container' data-page=32>

<b>Sưu tầm và tổng hợp Trịnh Bình </b> TÀI LIỆU TOÁN HỌC
<b>SỞ GIÁO DỤC VÀ ĐÀO TẠO </b>


<b>PHÚ THỌ </b>


<b>KỲ THI CHỌN HỌC SINH GIỎI CẤP TỈNH</b>


<b>Lớp 9 THCS năm học 2011-2012 </b>
<b>MơnTốn </b>


Thời gian làm bài: 150 phút, khơng kể thời gian giao đề
<i>Đề thi có 01 trang</i>


---


<b>Câu 1</b> (3,0 điểm) Tìm tất cả các số nguyên dương n để hai số n26 và n 11


đều là lập phương của hai số nguyên dương n|o đó.


<b>Câu 2 </b>(4,0 điểm) Giả sử a là một nghiệm của phương trình: 2x2  x 1 0.


Khơng giải phương trình, hãy tính giá trị của biểu thức


4

2



2a 3
A


2 2a 2a 3 2a




  


<b>Câu 3 </b>(4,0 điểm) a) Giải phương trình 2


8x 1 x 3x 1


b) Giải hệ phương trình


2 2
2


2x y 1
xy x 2


  




 


<b>Câu 4</b> (7,0 điểm) Cho đường tròn (O; R) v| điểm M nằm ngo|i đường tròn.



Qua điểm M vẽ hai tiếp tuyến MA, MB tới đường tròn (A và B là các tiếp điểm).
Gọi D l| điểm di động trên cung lớn AB (D không trùng với A, B v| điểm chính
giữa của cung) và C là giao điểm thứ hai của đường thẳng MD với đường tròn


(O; R).


a) Giả sử H l| giao điểm của c{c đường thẳng OM với AB. Chứng minh
rằng MH.MOMC.MD, từ đó suy ra đường trịn ngoại tiếp tam giác HCD luôn
đi qua một điểm cố định.


b) Chứng minh rằng nếu dây AD song song với đường thẳng MB thì


đường thẳng AC đi qua trọng tâm G của tam giác MAB.


c) Kẻ đường kính BK của đường tròn (O; R), gọi I l| giao điểm của các
đường thẳng MK v| AB. Tính b{n kính đường trịn ngoại tiếp tam giác MBI theo
R, khi biết OM = 2R.


<b>Câu 5</b> (2,0 điểm)


Cho các số thực dương a, b, c thỏa mãn: abc a  b 3ab. Chứng minh rằng


ab b a


3.
a b 1 bc c 1   ca c 1 


</div>
<span class='text_page_counter'>(33)</span><div class='page_container' data-page=33>

<b>Sưu tầm và tổng hợp Trịnh Bình </b> TÀI LIỆU TỐN HỌC
SỞ GIÁO DỤC VÀ ĐÀO TẠO PHÚ THỌ



<b> KỲ THI CHỌN HỌC SINH GIỎI CẤP TỈNH </b>
<b>LỚP 9 THCS, NĂM HỌC 2011-2012 </b>


<b>HƯỚNG DẪN CHẤM THI MƠN TỐN </b>
<b>(Hướng dẫn chấm thi gồm 5 trang)</b>
<b>I. Một số chú ý khi chấm bài </b>


 Hướng dẫn chấm thi dưới đ}y dựa vào lời giải sơ lược của một cách, khi chấm thi
giám khảo cần bám sát yêu cầu trình bày lời giải đầy đủ, chi tiết, hợp logic và có thể
chia nhỏ đến 0,25 điểm.


 Thí sinh làm bài cách khác với Hướng dẫn chấm m| đúng thì tổ chấm cần thống nhất
cho điểm tương ứng với biểu điểm của Hướng dẫn chấm.


<b>Điểm bài thi</b> là tổng c{c điểm thành phần khơng làm trịn số.


<b>II. Đáp án và biểu điểm </b>
<b>Câu 1</b> (3,0 điểm)


Tìm tất cả các số nguyên dương n để hai số n26 và n 11 đều là lập phương
của hai số nguyên dương n|o đó.


<b>ĐÁP ÁN </b> <b>ĐIỂM </b>


Giả sử có số nguyên dương n sao cho: 3 3


n26x ; n 11 y (với x, y là hai số
nguyên dương v| x > y)



Khi đó x3 y3 37

xy x

2 xyy2

37


1,5 đ


Lại có 2 2


0  x y x xyy và 37 là số nguyên tố nên


2 2


x y 1 (1)
x xy y 37 (2)


 


 <sub></sub> <sub></sub> <sub></sub>



Thay x = y + 1 v|o (2) ta được: 2


y  y 12 0 y = 3 là nghiệm duy nhất thoả
mãn. Vậy n = 38 là giá trị cần tìm.


1,5 đ


<b>Câu 2 </b>(4,0 điểm)


Giả sử a là một nghiệm của phương trình: 2



2x   x 1 0. Khơng giải phương
trình, hãy tính giá trị của biểu thức


</div>
<span class='text_page_counter'>(34)</span><div class='page_container' data-page=34>

<b>Sưu tầm và tổng hợp Trịnh Bình </b> TÀI LIỆU TỐN HỌC




4

2


2a 3
A


2 2a 2a 3 2a




  


<b>ĐÁP ÁN </b> <b>ĐIỂM </b>


a

là nghiệm của phương trình nên:

2a

2

  

a 1 0


2a

2

 

1 a *

 



2a4 a2 2a 1 .


1,0 đ


Thay vào biểu thức

A

ta được:



2

2

2 2


2a

3

2a

3



A



2 a

4a

4

2a

2 a

2

2a









1,0 đ


= 2a 3 <sub>2</sub>
2 a 2 2a




 

2


2a 3
2 2 a 2a






  ( vì theo

 

* thì

a 1

) 1,0 đ


2



2a 3 1


2


2 2a a 1 2 2a 3




  


    1,0 đ


<b>Câu 3 </b>(4,0 điểm)


a) Giải phương trình
2


8x 1 x 3x 1


b) Giải hệ phương trình


2 2
2



2x y 1


xy x 2


  





 





<b>ĐÁP ÁN </b> <b>ĐIỂM </b>


<b>a) (2,0 điểm) </b>Phương trình


2


4 3 2


x 3x 1 0 (1)
8x 1 x 6x 7x 6x 1 (2)


   



 


     






Ta có (2) 4 3 2


x 6x 7x 14x 0


     

3 2



x x 6x 7x 14 0


1,0 đ


 x x 1 x

2 7x 14

0
 x 0


x 1


 


</div>
<span class='text_page_counter'>(35)</span><div class='page_container' data-page=35>

<b>Sưu tầm và tổng hợp Trịnh Bình </b> TÀI LIỆU TỐN HỌC
Kết hợp (1) ta tìm được x =1 là nghiệm của phương trình.


<b>b) (2,0 điểm) </b>


Từ hệ đã cho ta suy ra: 2 2 2 2 2
xyx 4x -2y 3x xy2y 0



(xy)(3x2y)0 x y


3x 2y





  <sub> </sub>


 1,0 đ


Nếu x y thì: x2<sub> = 1 </sub> <sub>x</sub> <sub>1</sub><sub>. </sub>
Nếu 3x 2y thì:


2
y


1
9


  (khơng thỏa mãn).


Vậy tập nghiệm của hệ phương trình đã cho l|: S

  

1; 1 ,  1; 1 .



1,0 đ


<b>Câu 4</b> (7,0 điểm)


Cho đường tròn (O; R) v| điểm M nằm ngo|i đường tròn. Qua điểm M vẽ hai tiếp
tuyến MA, MB tới đường tròn (A và B là các tiếp điểm). Gọi D l| điểm di động trên


cung lớn AB (D khơng trùng với A, B v| điểm chính giữa của cung) và C là giao điểm
thứ hai của đường thẳng MD với đường tròn (O; R).


a) Giả sử H l| giao điểm của OM với AB. Chứng minh rằng MH.MO = MC.MD,
từ đó suy ra đường trịn ngoại tiếp tam gi{c HCD ln đi qua một điểm cố định.


b) Chứng minh rằng nếu dây AD song song với đường thẳng MB thì đường
thẳng AC đi qua trọng tâm G của tam giác MAB.


</div>
<span class='text_page_counter'>(36)</span><div class='page_container' data-page=36>

<b>Sưu tầm và tổng hợp Trịnh Bình </b> TÀI LIỆU TỐN HỌC


I


C


H O


M


A


B


D


E


K


<b>ĐÁP ÁN </b> <b>ĐIỂM </b>



<b>a) (2,5 điểm) </b>Vì tam giác AOM vng tại A có AHOM nên 2
MH.MOMA .
Mặt khác MACADC nên MAC đồng dạng MDA (g.g), do đó


MA MC 2


MC.MD MA


MD  MA  


Vậy MH.MOMC.MD


1,5 đ


Khi đó MH MC


MD MO. Do đó MHC đồng dạng MDO  MHCMDO.


Từ đó suy ra OHCD nội tiếp, vì vậy đường trịn ngoại tiếp HCD ln đi qua
điểm O cố định.


1,0 đ


<b>b) (2,5 điểm) </b>Giả sử AC cắt MB tại E, vì CBEEAB nên EBC đồng dạng 


EAB. Do đó EB EC 2


EA.EC EB .



EA  EB  


1,0 đ


Vì AD // MB nên EMCMDAMAC. Do đó EMC đồng dạng EAM
EM EC 2


EA.EC EM .


EA  EM  


Vậy EB = EM, tức l| E l| trung điểm của MB.


Tam gi{c MAB có MH v| AE l| c{c đường trung tuyến, nên AC luôn đi qua
trọng tâm G của MAB.


</div>
<span class='text_page_counter'>(37)</span><div class='page_container' data-page=37>

<b>Sưu tầm và tổng hợp Trịnh Bình </b> TÀI LIỆU TOÁN HỌC


I


M


B


N


<b>c) (2,0 điểm) </b>Vì OM = 2R nên MAB l| tam gi{c đều, do đó 0
MBA60 .



Kẻ đường kính MN của đường trịn ngoại tiếp BMI thì trong tam giác vng
IMN ta có sin INM IM MN IM <sub>0</sub> 2IM


MN sin 60 3


    (1)


Ta có AK // MO nên HIM đồng dạng AIK (g.g). Do đó IM MH.
IK  AK
Dễ thấy OH R


2


 nên AK = R và MH 3R
2


 , do đó IM 3 IM 3IK
IK  2   2 (2)


1,0 đ


Mặt khác IH 3 IH 3.
IA  2  AH 5
Vì AH R 3


2


 nên IH 3R 3, IA R 3.


10 5



 


Khi đó IK 2R 7
5


 , do đó IM 3R 7
5


 (3)


Vậy đường tròn ngoại tiếp BMI có bán kính / R 21


R .


5


1,0 đ


<b>Câu 5</b> (2,0 điểm) Cho các số dương a, b, c thỏa mãn: abc a  b 3ab. Chứng minh rằng


ab b a


3.
a b 1 bc c 1  ca c 1 


<b>ĐÁP ÁN </b> <b>ĐIỂM </b>


</div>
<span class='text_page_counter'>(38)</span><div class='page_container' data-page=38>

<b>Sưu tầm và tổng hợp Trịnh Bình </b> TÀI LIỆU TỐN HỌC



1 1 1


3


1 1 1 1 1 1 1 1


. .c c c. c


a b a b b b a a


  


      (1)


Đặt

x

1

, y

1

, z

c



a

b



thì x, y, z0 và x  y z 3 đồng thời bất đẳng thức phải
chứng minh trở thành


1

1

1



3


xy

 

x

y

yz

 

y

z

zx

 

z

x

(2)


Ta chứng minh

x y 1

23 xy

 x y

, với  x, y.
Thật vậy, bất đẳng thức trên tương đương với





 

 



2 2


2 2 2


2 x

y

1 2xy

2y

2x

6 xy

x

y



x

y

x 1

y 1

0



 

 





Dấu “=”xảy ra  x y 1.


Do đó

1

3



x

y 1



xy

 

x

y

 

, với x, y 0. Dấu “=” xảy ra  x y 1.


Tương tự ta suy ra


1

1

1

3

3

3



x

y 1

y

z 1

z

x 1




xy

 

x

y

yz

 

y

z

zx

 

z

x

 

 

 

(3)


Dấu “=” xảy ra  x  y z 1.


Ta chứng minh: 1 1 1 9 , m, n, p 0
m  n p m n p  
Thật vậy, bất đẳng thức trên tương đương với


n p m p m n n m p m p n


1 1 1 9 6


m m n n p p m n m p n p


   


 


          <sub></sub>  <sub> </sub>  <sub> </sub>  <sub></sub>


     


Theo bất đẳng thức Cô si ta thấy bất đẳng thức trên luôn đúng.
Dấu “=”xảy ra

m

 

n

p.



Do đó: 3 3 3


x y 1 y z 1 z x 1



9 3




3



2 x

y

z

3





  

(4)


Từ (3) v| (4) suy ra điều phải chứng minh.


Dấu “=”xảy ra  x  y z 1 hay

a

  

b

c 1.



</div>
<span class='text_page_counter'>(39)</span><div class='page_container' data-page=39>

<b>Sưu tầm và tổng hợp Trịnh Bình </b> TÀI LIỆU TOÁN HỌC


<b>Câu1( 3,0 </b><i><b>điểm</b></i><b>)</b> 1) Giải phương trình nghiệm nguyên 2


8<i>x</i> 3x<i>y</i>5<i>y</i>25
2)Tìm tất cả số nguyên dương n sao cho <i> A=</i> <i>n</i>.4<i>n</i>3 7<i>n</i>


<b>Câu 2( 4,0 </b><i><b>điểm</b></i><b>)</b> 1) Rút gọn biểu thức: A= 2 10 30 2 2 6 : 2
2 10 2 2 3 1


  


 


2) Cho các số thực dương a,b,c,x,y,z kh{c 0 thoả mãn<i> </i>.


2 2 2



x


<i>x</i> <i>yz</i> <i>y</i> <i>z</i> <i>z</i> <i>xy</i>


<i>a</i> <i>b</i> <i>c</i>


 <sub></sub>  <sub></sub> 


Chứng minh rằng


2 2 2


<i>a</i> <i>bc</i> <i>b</i> <i>ca</i> <i>c</i> <i>ab</i>


<i>x</i> <i>y</i> <i>z</i>


 <sub></sub>  <sub></sub> 


<b> Câu 3( 4,0 </b><i><b>điểm</b></i><b>) 1</b>) Cho phương trình: 2


6x 0


<i>x</i>   <i>m</i> (Với m là tham số). Tìm m để phương
trình đã cho có hai nghiệm <i>x</i>1 và <i>x</i>2 thoả mãn <i>x</i><sub>1</sub>2 <i>x</i><sub>2</sub>2 12


<b> 2</b>) Giải hệ phương trình:


3 3 3



2 2


8x 27 18
4x 6x


<i>y</i> <i>y</i>


<i>y</i> <i>y</i>


  





 





<b>Câu 4( 7,0 </b><i><b>điểm</b></i><b>) </b>1) Cho đường tròn (O) đường kính BD=2R, dây cung AC của đường trịn
(O) thay đổi nhưng ln vng góc v| cắt BD tại H. Gọi P,Q,R,S lần lượt l| ch}n c{c đường
vng góc hạ từ H xuống AB,AD,CD,CB.


a) CMR: 2 2 2 2


D


<i>HA</i> <i>HB</i> <i>HC</i> <i>H</i> không đổi.
b) CMR :<i>PQ</i>RS là tứ giác nội tiếp.


2) Cho hình vng ABCD và MNPQ có bốn đỉnh M,N,P,Q lần lượt thuộc các cạnh


AB,BC,CD,DA của hình vng. CMR:<i>SABC</i>D ≤


4


<i>MN</i> <i>NP</i> <i>PQ QM</i>


<i>AC</i>   


<b>Câu 5( 2,0 </b><i><b>điểm</b></i><b>) </b>


Cho a,b,c là các số thực dương. CMR:


3 2 3 2a 3 2 6


<i>ab</i> <i>bc</i> <i>ca</i> <i>a b c</i>


<i>a</i> <i>b</i> <i>c</i> <i>b</i> <i>c</i> <i>c</i> <i>a</i> <i>b</i>


 


  


     


SỞ GD&ĐT PHÚ THỌ


<b>ĐỀ CHÍNH THỨC </b>


<b>ĐỀ THI CHỌN HỌC SINH GIỎI CẤP TỈNH </b>



NĂM HỌC 2012 - 2013


<b>MƠN: TỐN - LỚP 9 </b>


</div>
<span class='text_page_counter'>(40)</span><div class='page_container' data-page=40>

<b>Sưu tầm và tổng hợp Trịnh Bình </b> TÀI LIỆU TOÁN HỌC


<b>Hướng dẫn </b>
<b>Câu1.1)</b> 2


8<i>x</i> 3x<i>y</i>5<i>y</i>25 <i>Z</i>


<i>x</i>
<i>x</i>
<i>y</i>
<i>x</i>
<i>x</i>
<i>y</i>
<i>x</i>
<i>x</i>
<i>y</i> 














5
3
25
40
24
9
5
3
25
8
25
8
)
5
3
(
2
2


Khi 3x+5 l| ước 25 từ đó tìm được (<i>x</i>;<i>y</i>)

(10;31);(2;7);(0;5)


( cách khac nhân 2 vế với 9 đưavề tích)


<b>1.2)</b> Với n chẵn n=2k thì


<i>m</i> <i>N</i>



<i>m</i>


<i>t</i>
<i>n</i>
<i>t</i>
<i>k</i>
<i>k</i>
<i>k</i>
<i>k</i>


<i>A</i> <i>k</i>  <i>k</i>   <i>k</i>  <i>k</i>  <i>k</i>       14 114 6 


2
1
7
7
1
2
7
)
9
16
(
4
).
1
2
(
3
4
.



2 2 2 2  


Với n lẻ n=2k+1


<i>m</i> <i>N</i>



<i>m</i>
<i>n</i>
<i>t</i>
<i>k</i>
<i>k</i>
<i>k</i>
<i>k</i>


<i>A</i>(2 1).42<i>k</i>132<i>k</i>12 .42<i>k</i>1(42<i>k</i>132<i>k</i>1)72 7 7  14 1 


Vậy <i>n</i>14<i>m</i>6 hoặc <i>n</i>14<i>m</i>1 ( với mọi n<i>N</i>) thì A chia hết cho 7


<b>Câu2.1</b>) 2 10 30 2 2 6 : 2


2 10 2 2 3 1


  
  =
2
1
2
1
3
.


2
1
3
2
1
3
.
4
3
2
4
2
1
3
.
2
3
2
2
1
3
.
)
1
5
(
2
2
)
1

5
(
6
)
1
5
(
2


2  <sub></sub>   <sub></sub>   <sub></sub>   <sub></sub>








<b>2.2)</b>


2 2 2


x


<i>x</i> <i>yz</i> <i>y</i> <i>z</i> <i>z</i> <i>xy</i>


<i>a</i> <i>b</i> <i>c</i>


 <sub></sub>  <sub></sub> 
)
3


(
)
3
(
2
:
)
2
(
)
3
(
2
:
)
1
(
)
3
(
2
3
3
3
2
2
3
3
2
2

2
2
2
4
2
3
3
3
2
2
3
3
2
2
2
2
2
4
2
3
3
3
2
2
3
3
2
2
2
2

2
4
2
2
2
2
<i>xyz</i>
<i>z</i>
<i>y</i>
<i>x</i>
<i>z</i>
<i>ab</i>
<i>c</i>
<i>xyz</i>
<i>z</i>
<i>y</i>
<i>z</i>
<i>x</i>
<i>y</i>
<i>x</i>
<i>ab</i>
<i>y</i>
<i>x</i>
<i>xyz</i>
<i>Z</i>
<i>c</i>
<i>Tuongtu</i>
<i>xyz</i>
<i>z</i>
<i>y</i>

<i>x</i>
<i>y</i>
<i>ac</i>
<i>b</i>
<i>z</i>
<i>xy</i>
<i>yz</i>
<i>y</i>
<i>x</i>
<i>z</i>
<i>x</i>
<i>ac</i>
<i>z</i>
<i>x</i>
<i>xz</i>
<i>y</i>
<i>y</i>
<i>b</i>
<i>Tuongtu</i>
<i>xyz</i>
<i>z</i>
<i>y</i>
<i>x</i>
<i>x</i>
<i>bc</i>
<i>a</i>
<i>yz</i>
<i>x</i>
<i>xz</i>
<i>xy</i>

<i>z</i>
<i>y</i>
<i>bc</i>
<i>z</i>
<i>y</i>
<i>yz</i>
<i>x</i>
<i>x</i>
<i>a</i>
<i>xy</i>
<i>z</i>
<i>c</i>
<i>xz</i>
<i>y</i>
<i>b</i>
<i>yz</i>
<i>x</i>
<i>a</i>










































Từ (1) (2) (3) ta co ĐPCM



<b>Câu 3.1) </b>Để phương trình có nghiệm / 0<i>m</i>9(*)


Mặt khác ta phải có 8


2
.
4
2
.
6
12
.
6
2
2
1
1
2
1
2
1
2
1
2
2
2
1
2
1


2
1





































<i>m</i>
<i>x</i>
<i>m</i>
<i>x</i>
<i>x</i>
<i>x</i>
<i>x</i>
<i>x</i>
<i>m</i>
<i>x</i>
<i>x</i>
<i>x</i>
<i>x</i>
<i>x</i>
<i>x</i>
<i>m</i>
<i>x</i>
<i>x</i>
<i>x</i>
<i>x</i>


TM ĐK (*)



</div>
<span class='text_page_counter'>(41)</span><div class='page_container' data-page=41>

<b>Sưu tầm và tổng hợp Trịnh Bình </b> TÀI LIỆU TỐN HỌC
HD y =0 khơng là nghiệm của hệ chia 2 vế PT(1) cho y3<sub> PT(2) cho y</sub>2 <sub> Ta có hệ</sub>












1
6
4
18
27
8
2
2
3
3
<i>y</i>
<i>x</i>
<i>y</i>
<i>x</i>
<i>y</i>
<i>x</i>


Đặt






<i>b</i>
<i>y</i>
<i>a</i>
<i>x</i>
3
2


ta có hệ


















1
3
3
18
2
2
3
3
<i>ab</i>
<i>b</i>
<i>a</i>
<i>ab</i>
<i>b</i>
<i>a</i>
<i>b</i>
<i>a</i>


Hệ có 2 nghiệm




























5
3
6
;
4
5
3
;
5
3
6
;
4
5
3

)
,
(<i>x</i> <i>y</i>


<b>Câu 4.1) </b>
<b>O</b>
<b>H</b>
<b>R</b>
<b>S</b>
<b>P</b>
<b>Q</b>
<b>D</b>
<b>C</b>
<b>B</b>
<b>A</b>


<b>a) </b>theo Pitago <i>HA</i>2<i>HB</i>2 <i>AB</i>2;<i>HC</i>2<i>HB</i>2 <i>BC</i>2;<i>HC</i>2<i>HD</i>2 <i>CD</i>2;<i>HA</i>2<i>HD</i>2 <i>AD</i>2;
suy ra đpcm


b)Tứ giác HPBS nội tiếp <i>HPS</i><i>HBS</i><i>DBC</i>


Tứ giác HPAQ là hình chữ nhật <i>HPQ</i><i>HAQ</i><i>CAD</i><i>CBD</i>


Do đó <i>SPQ</i><i>HPS</i><i>HPQ</i>2<i>CBC</i>


Tương tự <i>SQR</i>2<i>BDC</i>


Do đó 0 0


180



180   






<i>DBC</i> <i>BDC</i> <i>SPQ</i> <i>SRQ</i> nên tứ giác PQRS nội tiếp ( đ/lí đảo)


</div>
<span class='text_page_counter'>(42)</span><div class='page_container' data-page=42>

<b>Sưu tầm và tổng hợp Trịnh Bình </b> TÀI LIỆU TỐN HỌC
<b>L</b>
<b>K</b>
<b>P</b>
<b>Q</b>
<b>I</b>
<b>C</b>
<b>N</b>
<b>D</b>
<b>M</b>
<b>A</b> <b>B</b>


<b>Cách 1</b> Gọi T, K, L l| trung điểm MQ, MP, NP theo t/c đường trung bình và trung tuyến tam
giác vng ta có <i>MN</i><i>NP</i><i>PQ</i><i>QM</i> 2(<i>KL</i><i>CL</i><i>IK</i><i>AI</i>)2<i>AC</i> từ đó suy ra đpcm


<b>Cách 2</b> Ta có theo Pitago


2
2
)
( 2


2
2


2 <i>BM</i> <i>BN</i>


<i>MN</i>
<i>BN</i>


<i>BM</i>
<i>BM</i>


<i>BN</i>


<i>MN</i>        <b> ( áp dụng BĐT Bunhiacoopsky </b>


Tương Tự
2
;
2
;
2
<i>AM</i>
<i>AQ</i>
<i>MQ</i>
<i>DQ</i>
<i>DP</i>
<i>PQ</i>
<i>NP</i>
<i>CN</i>



<i>NP</i>     


<b>Nên </b>


<i>MN</i> <i>NP</i> <i>PQ</i> <i>QM</i>

<i>a</i> <i>dpcm</i>
<i>a</i>
<i>a</i>
<i>a</i>
<i>AM</i>
<i>QA</i>
<i>DQ</i>
<i>PD</i>
<i>CP</i>
<i>NC</i>
<i>NB</i>
<i>BM</i>
<i>QM</i>
<i>PQ</i>
<i>NP</i>
<i>MN</i>



















2
4
2
2
2
2
4
2


Dấu “=” xảy ra khi MNPQ là hình chữ nhật


<b>Câu 5 </b>


Cho a,b c > 0 .Chứng minh rằng:


6
2
3
3
2
2
3


<i>c</i>
<i>b</i>
<i>a</i>
<i>c</i>
<i>b</i>
<i>a</i>
<i>ca</i>
<i>c</i>
<i>b</i>
<i>a</i>
<i>bc</i>
<i>c</i>
<i>b</i>
<i>a</i>


<i>ab</i> <sub></sub>  











Dự đo{n a = b = c tách mẫu để a+c = b+c = 2b


Tacó áp dụng BĐT <sub></sub>

























<i>z</i>
<i>y</i>
<i>x</i>
<i>z</i>
<i>y</i>
<i>x</i>
<i>z</i>
<i>y</i>


<i>x</i>
<i>z</i>
<i>y</i>


<i>x</i> 1 1 1


9
1
1
9
1
1
1
)
(


1

1

1

1



(1)



3

2

(

) (

) 2

9

2

9

2



<i>ab</i>

<i>ab</i>

<i>ab</i>

<i>ab</i>

<i>ab</i>

<i>a</i>



<i>a</i>

<i>b</i>

<i>c</i>

<i>a c</i>

<i>b c</i>

<i>b</i>

<i>a c</i>

<i>b c</i>

<i>b</i>

<i>a c</i>

<i>b c</i>





<sub></sub>

<sub></sub>

<sub></sub>

<sub></sub>




   



</div>
<span class='text_page_counter'>(43)</span><div class='page_container' data-page=43>

<b>Sưu tầm và tổng hợp Trịnh Bình </b> TÀI LIỆU TOÁN HỌC


1 1 1 1


(2)


2 3 ( ) ( ) 2 9 2 9 2


1 1 1 1


(2)


3 2 ( ) ( ) 2 9 2 9 2


<i>bc</i> <i>bc</i> <i>bc</i> <i>bc</i> <i>bc</i> <i>b</i>


<i>a</i> <i>b</i> <i>c</i> <i>a</i> <i>b</i> <i>a</i> <i>c</i> <i>c</i> <i>a</i> <i>c</i> <i>b</i> <i>c</i> <i>b</i> <i>a</i> <i>b</i> <i>b</i> <i>c</i>


<i>ac</i> <i>ac</i> <i>ac</i> <i>ac</i> <i>ac</i> <i>c</i>


<i>a</i> <i>b</i> <i>c</i> <i>a</i> <i>b</i> <i>b</i> <i>c</i> <i>a</i> <i>a</i> <i>b</i> <i>b</i> <i>c</i> <i>a</i> <i>a</i> <i>b</i> <i>b</i> <i>c</i>


   


  <sub></sub>   <sub></sub> <sub></sub>   <sub></sub>


      <sub></sub>   <sub></sub> <sub></sub>   <sub></sub>



   


  <sub></sub>   <sub></sub> <sub></sub>   <sub></sub>


             


Từ (1) (2) (3)


6
2


9


1 <i>a</i> <i>b</i> <i>c</i> <i>a</i> <i>b</i> <i>c</i>


<i>c</i>
<i>a</i>


<i>ab</i>
<i>bc</i>
<i>c</i>


<i>b</i>
<i>ac</i>
<i>ab</i>
<i>b</i>


<i>a</i>
<i>bc</i>
<i>ac</i>



<i>P</i>   








 <sub></sub>  












</div>
<span class='text_page_counter'>(44)</span><div class='page_container' data-page=44>

<b>Sưu tầm và tổng hợp Trịnh Bình </b> TÀI LIỆU TOÁN HỌC


<b>SỞ GIÁO DỤC VÀ ĐÀO TẠO </b>
<b>PHÚ THỌ </b>


<b>ĐỀ THI HỌC SINH GIỎI </b>


<b>MƠN: TỐN 9 </b>
<b>NĂM HỌC 2013 – 2014</b>


<b>Câu 1 </b>(<b>3,0 </b><i>điểm</i>)


a)Giải phương trình trên tập số nguyên 2 2


5 4 4 8 12 0.


<i>x</i>  <i>y</i>  <i>xy</i> <i>x</i> <i>y</i> 
b) Cho

 

3 2


3 14 2


<i>P x</i> <i>x</i>  <i>x</i>  <i>x</i> . Tìm số các số tự nhiên <i>x</i> nhỏ hơn 100 m| <i>P x</i>

 

chia
hết cho 11.


<b>Câu 2 </b>(<b>4,0 </b><i>điểm</i>)<b> </b>


a)Tính giá trị biểu thức


3


3 2


3 2


4 5 2


<i>a</i> <i>a</i>


<i>P</i>



<i>a</i> <i>a</i> <i>a</i>


 




   , biết


3 3


55 3024 55 3024 .


<i>a</i>   


b) Cho các số thực <i>x y z</i>, , đôi một khác nhau thỏa mãn
<i>x</i>3 3<i>x</i>1, <i>y</i>3 3<i>y</i>1 và <i>z</i>3 3<i>z</i>1.
Chứng minh rằng 2 2 2


6
<i>x</i> <i>y</i> <i>z</i>  .


<b>Câu 3 </b>(<b>4,0 </b><i>điểm</i>)<b> </b>a) Giải phương trình 3 1 1 3 1.
4


<i>x</i>


<i>x</i> <i>x</i>


<i>x</i>



   


b) Giải hệ phương trình


2 2


2 2


3 2 4 8 4 0


2 3 0.


<i>x</i> <i>y</i> <i>xy</i> <i>x</i> <i>y</i>


<i>x</i> <i>y</i> <i>x</i> <i>y</i>


      


 <sub></sub> <sub></sub> <sub>  </sub>



<b>Câu 4 </b>(<b>7,0 </b><i>điểm</i>)<b> </b>


Cho đường tròn (<i>O;</i> <i>R</i>) v| d}y cung <i>BC</i> không đi qua t}m. Gọi <i>A</i> l| điểm chính
giữa của cung nhỏ <i>BC</i>. Góc nội tiếp <i>EAF</i> quay quanh điểm <i>A</i> v| có số đo bằng 
khơng đổi sao cho <i>E, F</i> khác phía với điểm <i>A</i> so với <i>BC</i>; <i>AF</i> v| <i>AE</i> cắt đường thẳng


<i>BC</i> lần lượt tại <i>M</i> v| <i>N</i>. Lấy điểm <i>D</i> sao cho tứ giác <i>MNED</i> l| hình bình h|nh.
a) Chứng minh <i>MNEF</i> l| tứ gi{c nội tiếp.



b) Gọi <i>I </i>l| t}m đường tròn ngoại tiếp tam gi{c <i>MDF</i>. Chứng minh rằng khi góc nội
tiếp <i>EAF</i> quay quanh điểm <i>A</i> thì <i>I</i> chuyển động trên một đường thẳng cố định.
c) Khi 0


60


  và <i>BC</i><i>R</i>, tính theo <i>R</i> độ dài nhỏ nhất của đoạn thẳng <i>OI.</i>


<b>Câu 5 </b>(<b>2,0 </b><i>điểm</i>)Cho c{c số thực dương <i>x y z</i>, , thỏa mãn <i>x</i>  <i>y</i> <i>z</i> 3.
Chứng minh rằng


2 2 2 2 2 2 2 2 2


2 2 2


4 .


4 4 4


<i>x</i> <i>y</i> <i>z</i> <i>y</i> <i>z</i> <i>x</i> <i>z</i> <i>x</i> <i>y</i>


<i>xyz</i>


<i>yz</i> <i>zx</i> <i>xy</i>


  <sub></sub>   <sub></sub>   <sub></sub>


  



</div>
<span class='text_page_counter'>(45)</span><div class='page_container' data-page=45>

<b>Sưu tầm và tổng hợp Trịnh Bình </b> TÀI LIỆU TOÁN HỌC


<b>SỞ GIÁO DỤC VÀ ĐÀO TẠO </b>
<b>PHÚ THỌ </b>


<b>HDC THI HỌC SINH GIỎI </b>


<b>MƠN: TỐN 9 </b>
<b>NĂM HỌC 2013 – 2014</b>


<b>I. Một số chú ý khi chấm bài </b>


 Đ{p {n chấm thi dưới đ}y dựa vào lời giải sơ lược của một cách, khi chấm thi, giám
khảo cần bám sát yêu cầu trình bày lời giải đầy đủ, chi tiết, hợp lơ-gic và có thể chia nhỏ
đến 0,25 điểm.


 Thí sinh làm bài cách khác với Đ{p {n m| đúng thì tổ chấm cần thống nhất cho điểm
tương ứng với thang điểm của Đ{p {n.


<b>Điểm bài thi</b> là tổng điểm các câu khơng làm trịn số.


<b>II. Đáp án-thang điểm </b>
<b>Câu 1 ( 3,0 </b><i><b>điểm</b></i><b>) </b>


a)Giải phương trình sau trên tập số nguyên 2 2


5 4 4 8 12 0.


<i>x</i>  <i>y</i>  <i>xy</i> <i>x</i> <i>y</i> 
b)Cho

 

3 2


3 14 2


<i>P x</i>  <i>x</i>  <i>x</i>  <i>x</i> . Tìm số các số tự nhiên <i>x</i> nhỏ hơn 100 m| <i>P x</i>

 

chia hết
cho 11.


<b>Nội dung </b> <b>Điểm </b>


a) Phương trình tương đương với


2 2

2


4 4 4 2 4 16


<i>x</i>  <i>y</i>  <i>xy</i>  <i>x</i> <i>y</i>    <i>y</i> 

<i>x</i>2<i>y</i>2

2 16 <i>y</i>2; <b>0,5 </b>
mà <i>x y</i>,  nên

<i>x</i>2<i>y</i>2

2 16, <i>y</i>0 (1) hoặc <i>x</i>2<i>y</i> 2 0,<i>y</i>2 16 (2). <b>0,5 </b>


Ta có (1) <i>x</i> 2,<i>y</i>0 hoặc <i>x</i> 6,<i>y</i>0.
(2) <i>y</i> 4,<i>x</i>6 hoặc <i>y</i> 4,<i>x</i> 10.


Vậy phương trình đã cho có nghiệm là

<i>x y</i>;

 

2; 0 ,

 

6; 0 , 6; 4 ,

 

 

10;4

.


<b>0,5 </b>


b) Bổ đề: Cho <i>x, y</i> là các số tự nhiên và số ngun tố <i>p</i> có dạng <i>p</i>3<i>k</i>2 thì




3 3



mod mod


<i>x</i>  <i>y</i> <i>p</i>  <i>x</i> <i>y</i> <i>p</i> .


Thật vậy,

3 3



mod mod


<i>x</i> <i>y</i> <i>p</i> <i>x</i>  <i>y</i> <i>p</i> , đúng.


Với 3 3

3 3



mod <i>k</i> <i>k</i> mod


<i>x</i>  <i>y</i> <i>p</i> <i>x</i>  <i>y</i> <i>p</i> .


</div>
<span class='text_page_counter'>(46)</span><div class='page_container' data-page=46>

<b>Sưu tầm và tổng hợp Trịnh Bình </b> TÀI LIỆU TỐN HỌC
Với <i>x y</i>, cùng chia hết cho <i>p</i> thì hiển nhiên đúng.


Với

<i>x p</i>,

1,

<i>y p</i>,

1 ta có 1 1

3 1 3 1



1 mod mod


<i>p</i> <i>p</i> <i>k</i> <i>k</i>


<i>x</i>   <i>y</i>   <i>p</i> <i>x</i>   <i>y</i>  <i>p</i>




3 3



. <i>k</i> . <i>k</i> mod mod


<i>x x</i> <i>y y</i> <i>p</i> <i>x</i> <i>y</i> <i>p</i>


    vì 3 3



mod


<i>k</i> <i>k</i>


<i>x</i>  <i>y</i> <i>p</i> .


Áp dụng Bổ đề, ta có


 

 

 



 

 



3 3


3 3


mod 11 1 11 1 10 1 11 1 10 mod 11


1 1 mod 11 1 1 mod 11 mod 11 .


<i>P x</i> <i>P y</i> <i>x</i> <i>x</i> <i>y</i> <i>y</i>


<i>x</i> <i>y</i> <i>x</i> <i>y</i> <i>x</i> <i>y</i>



          


         


Do đó, <i>P x</i>

 

<i>P y</i>

 

mod 11

 <i>x</i> <i>y</i>

mod 11 .



<b>0,5 </b>


Suy ra với mỗi <i>n</i> , trong 11 giá trị <i>P n</i>

  

,<i>P n</i>1 ,

,<i>P n</i>

10

, có duy nhất
một giá trị chia hết cho 11. Do đó, trong c{c số <i>P</i>

   

1 ,<i>P</i> 2 , ,<i>P</i>

 

99 có đúng 9
số chia hết cho 11, còn <i>P</i>

 

0  2 khơng chia hết cho 11.


Vậy có đúng 9 số thỏa mãn yêu cầu bài toán.


<b>0,5 </b>


<b>Câu 2 ( 4,0 </b><i><b>điểm</b></i><b>) </b>


a)Tính giá trị biểu thức


3


3 2


3 2


4 5 2


<i>a</i> <i>a</i>



<i>P</i>


<i>a</i> <i>a</i> <i>a</i>


 




   , biết


3 3


55 3024 55 3024 .


<i>a</i>   


b) Cho các số thực <i>x y z</i>, , thỏa mãn <i>x</i>3 3<i>x</i>1, <i>y</i>3 3<i>y</i>1,<i>z</i>33<i>z</i>1. Chứng minh rằng
2 2 2


6.
<i>x</i> <i>y</i> <i>z</i> 


<b>Nội dung </b> <b>Điểm </b>


a) Ta có

 



 



2


3


2


3 2


1 2


3 2 2


4 5 2 <sub>1</sub> <sub>2</sub> 2


<i>a</i> <i>a</i>


<i>a</i> <i>a</i> <i>a</i>


<i>P</i>


<i>a</i> <i>a</i> <i>a</i> <i><sub>a</sub></i> <i><sub>a</sub></i> <i>a</i>


 


  


  


      ; <b>0,5 </b>


mà 3 3 2

3 3




110 3 55 3024 55 3024 55 3024 .


<i>a</i>       <b><sub>0,5 </sub></b>


3 3


110 3 3 110 0


<i>a</i> <i>a</i> <i>a</i> <i>a</i>


       .


2



5 5 22 0 5


<i>a</i> <i>a</i> <i>a</i> <i>a</i>


       . Suy ra 7


3


<i>P</i> . <b>1,0 </b>


b) Ta có <i>x</i>3 3<i>x</i>1(1), <i>y</i>3 3<i>y</i>1 (2),<i>z</i>3 3<i>z</i>1 (3).


Từ (1), (2) và (3) suy ra









3 3 <sub>2</sub> <sub>2</sub>


3 3 2 2


2 2


3 3


3 <sub>3 (4)</sub>


3 3 (5)


3 (6).
3


<i>x</i> <i>y</i> <i>x</i> <i>y</i> <i><sub>x</sub></i> <i><sub>xy</sub></i> <i><sub>y</sub></i>


<i>y</i> <i>z</i> <i>y</i> <i>z</i> <i>y</i> <i>yz</i> <i>z</i>


<i>z</i> <i>zx</i> <i>x</i>


<i>z</i> <i>x</i> <i>z</i> <i>x</i>


    <sub> </sub> <sub></sub> <sub></sub>


 <sub></sub>





     


 


 <sub></sub> <sub></sub> <sub></sub> <sub>   </sub>





<b>1,0 </b>


</div>
<span class='text_page_counter'>(47)</span><div class='page_container' data-page=47>

<b>Sưu tầm và tổng hợp Trịnh Bình </b> TÀI LIỆU TOÁN HỌC
2 2





0 0 0


<i>x</i> <i>z</i> <i>xy</i><i>yz</i>  <i>x</i><i>y</i> <i>x</i> <i>y</i> <i>z</i>     <i>x</i> <i>y</i> <i>z</i> ,
(vì <i>x, y, z</i> đôi một phân biệt).


Cộng (4), (5) và (6) theo vế với vế ta có


<sub>2</sub> <sub>2</sub> <sub>2</sub>

2 <sub>2</sub> <sub>2</sub> <sub>2</sub>


3 1


9 6



2 <i>x</i> <i>y</i> <i>z</i> 2 <i>x</i> <i>y</i> <i>z</i>  <i>x</i> <i>y</i> <i>z</i>  .
<b>Câu 3 ( 4,0 </b><i><b>điểm</b></i><b>) </b>


a) Giải phương trình 3 1 1 3 1.
4


<i>x</i>


<i>x</i> <i>x</i>


<i>x</i>


   


b) Giải hệ phương trình


2 2


2 2


3 2 4 8 4 0


2 3 0


<i>x</i> <i>y</i> <i>xy</i> <i>x</i> <i>y</i>


<i>x</i> <i>y</i> <i>x</i> <i>y</i>


      





    




<b>Nội dung </b> <b>Điểm </b>


a) Điều kiện x{c định: 1, 0
3


<i>x</i>  <i>x</i> .


Phương trình tương đương với 2



12<i>x</i>  3<i>x</i> 1 4<i>x</i> 3<i>x</i>1. Đặt <i>a</i>2 ,<i>x b</i> 3<i>x</i>1
ta có phương trình 2 2





3<i>a</i> <i>b</i> 2<i>ab</i> <i>b</i><i>a b</i>3<i>a</i>   0 <i>b</i> <i>a</i> hoặc <i>b</i> 3<i>a</i>. Khi
đó 3<i>x</i> 1 2<i>x</i> hoặc 3<i>x</i>  1 6<i>x</i>.


<b>1,0 </b>


+) Với 3<i>x</i> 1 2<i>x</i>, điều kiện <i>x</i>0, ta có


2 2


3<i>x</i> 1 2<i>x</i>3<i>x</i> 1 4<i>x</i> 4<i>x</i> 3<i>x</i>   1 0 <i>x</i> 1 hoặc 1
4



<i>x</i>  (loại). <b>0,5 </b>


+) Với 3<i>x</i>  1 6<i>x</i>, điều kiện 1 0
3 <i>x</i>


   , ta có


2 3 153


3 1 6 36 3 1


72


<i>x</i>   <i>x</i> <i>x</i>  <i>x</i>  <i>x</i>  hoặc 3 153
72


<i>x</i>  (loại).


Vậy phương trình có hai nghiệm 1, 3 153.
72


<i>x</i> <i>x</i> 


<b>0,5 </b>


b) Nhân cả hai vế của (2) với 2 ta có hệ phương trình


2 2



2 2


3 2 4 8 4 0 (1)


2 2 4 2 6 0 (2)


<i>x</i> <i>y</i> <i>xy</i> <i>x</i> <i>y</i>


<i>x</i> <i>y</i> <i>x</i> <i>y</i>


      




    




Lấy (1) trừ (2) theo vế với vế ta có


2 2

2



4 4 3 2 2 0 2 3 2 2 0


<i>x</i>  <i>xy</i> <i>y</i>  <i>x</i> <i>y</i>    <i>x</i> <i>y</i>  <i>x</i> <i>y</i>  

<i>x</i>2<i>y</i>1



<i>x</i>2<i>y</i>2

  0 <i>x</i> 2<i>y</i>1 hoặc <i>x</i>2<i>y</i>2.


</div>
<span class='text_page_counter'>(48)</span><div class='page_container' data-page=48>

<b>Sưu tầm và tổng hợp Trịnh Bình </b> TÀI LIỆU TỐN HỌC


<b>T</b>



<b>S</b>


<b>D</b>
<b>I</b>


<b>J</b>
<b>K</b>


<b>N</b>
<b>H</b>
<b>M</b>


<b>G</b>


<b>P</b>


<b>E</b>
<b>A</b>


<b>Q</b>


<b>O</b>


<b>B</b> <b><sub>C</sub></b>


<b>F</b>


+) Với <i>x</i>2<i>y</i>1, thế vào (2) và rút gọn ta có <i>y y</i>

   3

0 <i>y</i> 0 hoặc <i>y</i> 3.
Suy ra <i>x</i>1, <i>y</i>0 hoặc <i>x</i> 5, <i>y</i> 3.


+) Với <i>x</i>2<i>y</i>2, thế vào (2) và rút gọn ta có 3 2 13 5 0 13 109
6
<i>y</i>  <i>y</i>   <i>y</i>  
hoặc 13 109.


6
<i>y</i>  


Suy ra 7 109, 13 109


3 6


<i>x</i>   <i>y</i>   hoặc 7 109, 13 109.


3 6


<i>x</i>   <i>y</i>  


Vậy hệ có 4 nghiệm <i>x</i>1, <i>y</i>0; <i>x</i> 5, <i>y</i> 3;


7 109, 13 109


3 6


<i>x</i>   <i>y</i>   ; 7 109, 13 109.


3 6



<i>x</i>   <i>y</i>  


<b>1,0 </b>


<b>Câu 4 ( 7,0 </b><i><b>điểm</b></i><b>) </b>


Cho đường tròn (<i>O;R</i>) v| d}y cung <i>BC</i> không đi qua t}m. Gọi <i>A</i> l| điểm chính giữa của
cung nhỏ <i>BC</i>. Góc nội tiếp <i>EAF</i> quay quanh điểm <i>A</i> v| có số đo bằng  không đổi sao
cho <i>E, F</i> khác phía với điểm <i>A</i> so với <i>BC</i>; <i>AF</i> v| <i>AE</i> cắt đường thẳng <i>BC</i> lần lượt tại <i>M</i> v|


<i>N</i>. Lấy điểm <i>D</i> sao cho tứ giác <i>MNED</i> l| hình bình h|nh.
a) Chứng minh <i>MNEF</i> l| tứ gi{c nội tiếp.


b) Gọi <i>I </i>l| t}m đường tròn ngoại tiếp tam gi{c <i>MDF</i>. Chứng minh rằng khi góc nội tiếp
<i>EAF</i> quay quanh điểm <i>A</i> thì <i>I</i> chuyển động trên một đường thẳng cố định.


c) Tìm độ d|i nhỏ nhất của đoạn thẳng <i>OI</i> khi 0
60


  và <i>BC</i> <i>R</i> .


a) Ta có
<i>MNE</i> =1


2 (sđ<i>AC</i> sđ<i>BFE</i>) =
= 1


2 (sđ<i>AB</i>sđ<i>BFE</i>)
<i>AFE</i>  sđ<i>AC</i> sđ<i>CE</i>
Suy ra: <i>MNE</i><i>MFE</i>180<i>o</i>


Vậy tứ gi{c <i>MNEF</i> nội tiếp.


<b>2,5 </b>


</div>
<span class='text_page_counter'>(49)</span><div class='page_container' data-page=49>

<b>Sưu tầm và tổng hợp Trịnh Bình </b> TÀI LIỆU TỐN HỌC


<b>U</b>


<b>N</b>


<b>M</b> <b>Q</b>


<b>A</b>


<b>B</b> <b>M'</b> <b>C</b>


Lấy <i>G</i> đối xứng với <i>E</i> qua <i>AP</i>  <i>D</i> <i>EG G</i>, 

 

<i>O</i>


Ta có <i>MDG</i><i>NEG</i>, <i>AEG</i> <i>AFG</i>180<i>o</i> <i>MDG</i><i>MFG</i>180<i>o</i>
Suy ra tứ gi{c <i>MDGF</i> nội tiếp (1)
Gọi giao điểm của <i>AG</i> v| <i>BC</i> l| <i>H</i>


Chứng minh tương tự a) có tứ gi{c <i>MHGF</i> nội tiếp (2)
Từ (1) v| (2) suy ra c{c điểm <i>M, H, D, G, </i>


<i>F</i> nằm trên một đường tròn.


Trung trực của đoạn thẳng <i>FG</i> đi qua <i>O</i>


v| cắt đường tròn <i>(O)</i> tại <i>J; I</i><i>OJ, </i>sđ<i>JF</i>



<i>=</i>sđ<i>JG</i> và sđ<i>PG=</i>sđ<i>PE</i>nên <i>JOP</i>
hay <i>I</i> nằm trên đường thẳng cố định. Đó
l| đường thẳng đi qua <i>O</i> và tạo với <i>AO</i>


một góc  khơng đổi.


<b>1,5 </b>


c) Hạ <i>IT</i> <i>BC T</i>

<i>BC</i>

<i>TH</i> <i>TM</i>. Do <i>QH</i> <i>QN</i>, suy ra 1
2


<i>IS</i>  <i>MN</i>


Tam gi{c vng <i>OSI</i> có <i>IOS</i>  không đổi nên <i>OI</i> nhỏ nhất khi v| chỉ khi <i>IS</i>


nhỏ nhất  <i>MN</i> nhỏ nhất. Ta chứng minh <i>MN</i> nhỏ nhất khi v| chỉ khi tam
gi{c <i>AMN</i> c}n tại <i>A</i>.


<b>1,0 </b>


Thật vậy, trên <i>BC</i> lấy <i>M’ N’</i> sao cho <i>M AN</i>' '. Khơng mất tính tổng qu{t giả
sử <i>QM</i>'<i>QN</i>' suy ra <i>AM</i>'<i>AN</i>'. Trên đoạn <i>AM</i>' lấy điểm <i>U</i> sao cho


'


<i>AU</i>  <i>AN</i>


'



<i>AUM</i> <i>ANN</i>


    (c.g.c)<i>SAM M</i>' <i>SANN</i>'<i>MM</i>'<i>NN</i>'<i>M N</i>' '<i>MN</i>


Với

60 ;<i>o</i> <i>BC</i><i>R</i> suy ra 3

2 3



2 2


<i>R</i>
<i>R</i>


<i>AQ</i> <i>R</i>




   ,


2 3

<sub>2</sub>

2 3 3


.


2 3 3


<i>R</i> <i>R</i>


<i>MN</i>    

2 3 3

.


6
<i>R</i>
<i>OI</i>





 


<b>1,0 </b>


<b>Câu 5 ( 2,0 </b><i><b>điểm</b></i><b>) </b>Cho các số thực dương <i>x y z</i>, , thỏa mãn <i>x</i>  <i>y</i> <i>z</i> 3. Chứng minh rằng:


2 2 2 2 2 2 2 2 2


2 2 2


4 .


4 4 4


<i>x</i> <i>y</i> <i>z</i> <i>y</i> <i>z</i> <i>x</i> <i>z</i> <i>x</i> <i>y</i>


<i>xyz</i>


<i>yz</i> <i>zx</i> <i>xy</i>


  <sub></sub>   <sub></sub>   <sub></sub>


</div>
<span class='text_page_counter'>(50)</span><div class='page_container' data-page=50>

<b>Sưu tầm và tổng hợp Trịnh Bình </b> TÀI LIỆU TỐN HỌC


<b>Nội dung </b> <b><sub> Điểm </sub></b>


Chứng minh được: 2 2 2



2<i>x</i> <i>y</i> <i>z</i> 2<i>x y</i><i>z</i> .


Tương tự ta có 2 2 2

2 2 2



2<i>y</i> <i>z</i> <i>x</i> 2<i>y z</i><i>x</i> , 2<i>z</i> <i>x</i>  <i>y</i> 2<i>z x</i><i>y</i> .


Do đó ta sẽ chứng minh

2


4 4 4


<i>x y</i> <i>z</i> <i>y z</i> <i>x</i> <i>z x</i> <i>y</i>


<i>xyz</i>


<i>yz</i> <i>zx</i> <i>xy</i>


  


  


   .


Bất đẳng thức n|y tương đương với


4

2

4

2

4

1.


<i>y</i> <i>z</i> <i>z</i> <i>x</i> <i>x</i> <i>y</i>


<i>yz</i> <i>yz</i> <i>zx</i> <i>zx</i> <i>xy xy</i>



 <sub></sub>  <sub></sub>  <sub></sub>


  


<b>0,5 </b>


Ta có

<sub></sub>

<sub></sub>





2

 

1



4 2 <sub>2</sub> <sub>2</sub> <sub>2</sub> <sub>2</sub> <sub>2</sub>


<i>yz</i>
<i>y</i> <i>z</i>


<i>yz</i> <i>yz</i> <i><sub>yz</sub></i> <i><sub>yz</sub></i> <i><sub>yz</sub></i> <i><sub>yz</sub></i> <i><sub>yz</sub></i> <i><sub>yz</sub></i>


 <sub></sub> <sub></sub>


 <sub></sub> <sub></sub> <sub></sub> <sub></sub> , dễ có


2


0 2 <i>yz</i> <i>yz</i>   <i>xy</i> 1  1 1 nên


 

1

1 .


2


2 <i>yz</i> <i>yz</i> 2 <i>yz</i> <i>yz</i>






 


Vậy nên


4

2 2 1


<i>y</i> <i>z</i>


<i>yz</i> <i>yz</i> <i>yz</i>


 <sub></sub>


  , tương tự có



1


4 2 2


<i>z</i> <i>x</i>


<i>zx</i> <i>zx</i> <i>zx</i>


 <sub></sub>


  và



4

2 2 1 .


<i>x</i> <i>y</i>


<i>xy</i> <i>xy</i> <i>xy</i>


 <sub></sub>


 


Do đó

<sub></sub>

<sub></sub>

<sub></sub>

<sub></sub>

<sub></sub>

<sub></sub>

1 1 1


4 2 4 2 4 2 2 2


<i>y</i> <i>z</i> <i>z</i> <i>x</i> <i>x</i> <i>y</i>


<i>yz</i> <i>yz</i> <i>zx</i> <i>zx</i> <i>xy xy</i> <i>xy</i> <i>yz</i> <i>zx</i>


  


    


      .


<b>0,5 </b>


Với <i>a, b, c></i>0 có


1 1 1



3 <i>a</i> <i>b</i> <i>b</i> <i>c</i> <i>c</i> <i>a</i> 3 2 2 2 9


<i>a</i> <i>b</i> <i>c</i>


<i>a</i> <i>b</i> <i>c</i> <i>b</i> <i>a</i> <i>c</i> <i>b</i> <i>a</i> <i>c</i>


       


  <sub></sub>   <sub></sub> <sub></sub>  <sub> </sub>  <sub> </sub>  <sub></sub>    


        nên


1 1 1 9


<i>a</i>   <i>b</i> <i>c</i> <i>a</i> <i>b</i> <i>c</i>(*).


Áp dụng (*) ta có 1 1 1 9 1


2 <i>xy</i> 2 <i>yz</i> 2 <i>zx</i> 6 <i>xy</i>  <i>yz</i> <i>zx</i>  ;


(Vì 3


2 2 2


<i>x</i> <i>y</i> <i>y</i> <i>z</i> <i>z</i> <i>x</i>


<i>xy</i>  <i>yz</i>  <i>zx</i>          <i>x</i> <i>y</i> <i>z</i> ).


Vậy



4

2

4

2

4

1


<i>y</i> <i>z</i> <i>z</i> <i>x</i> <i>x</i> <i>y</i>


<i>yz</i> <i>yz</i> <i>zx</i> <i>zx</i> <i>xy xy</i>


 <sub></sub>  <sub></sub>  <sub></sub>


   .


</div>
<span class='text_page_counter'>(51)</span><div class='page_container' data-page=51>

<b>Sưu tầm và tổng hợp Trịnh Bình </b> TÀI LIỆU TỐN HỌC
Do vậy ta có


2 2 2 2 2 2 2 2 2


2 2 2


4


4 4 4


<i>x</i> <i>y</i> <i>z</i> <i>y</i> <i>z</i> <i>x</i> <i>z</i> <i>x</i> <i>y</i>


<i>xyz</i>


<i>yz</i> <i>zx</i> <i>xy</i>


  <sub></sub>   <sub></sub>   <sub></sub>


   .



Đẳng thức xảy ra khi và chỉ khi <i>x</i>  <i>y</i> <i>z</i> 1.


</div>
<span class='text_page_counter'>(52)</span><div class='page_container' data-page=52>

<b>Sưu tầm và tổng hợp Trịnh Bình </b> TÀI LIỆU TỐN HỌC


<b>SỞ GIÁO DỤC VÀ ĐÀO TẠO </b>


<b>PHÚ THỌ </b>



<b>ĐỀ THI HỌC SINH GIỎI </b>


<b>Môn thi: Toán lớp 9 </b>


<b>NĂM HỌC 2014-2015 </b>



<b>Câu 1</b> (3,0 điểm)


a)Tìm các số nguyên x, y thỏa mãn phương trình: <i>x</i>2 <i>y</i>2 <i>xy</i><i>x</i> <i>y</i>2.


b) Chứng minh rằng với ba số tự nhiên a,b,c trong đó có đúng một số lẻ và
hai số chẵn ta ln có

 

3

 

3

 

3

3


<i>c</i>
<i>b</i>
<i>a</i>
<i>a</i>
<i>c</i>
<i>b</i>
<i>c</i>
<i>b</i>
<i>a</i>
<i>c</i>
<i>b</i>



<i>a</i>           Chia hết cho 96


<b>Câu 2 </b>(4,0 điểm)


a) Chứng minh rằng với mọi số nguyên dương n ta có


2
1
1
1
2
1
1
1
2













<i>n</i>


<i>n</i>
<i>n</i>
<i>n</i>


b) Tính tổng


2
2
2
2
2016
1
2014
1
1
...
5
1
3
1
1
4
1
2
1
1
3
1
1
1 






 <sub></sub>








 







 







 



<i>S</i>


<b>Câu 3 </b>(4,0 điểm)


a) Giải phương trình: 2 2
2
2<i>x</i> <i>x</i>  <i>x</i><i>x</i>


b) Giải hệ phương trình

 

















0
6
2
4


1
2
1
1
2
2
2
2
<i>y</i>
<i>x</i>
<i>y</i>
<i>x</i>
<i>xy</i>
<i>x</i>
<i>y</i>
<i>y</i>
<i>x</i>


<b>Câu 4</b> (7,0 điểm) Cho BC là dây cung cố định của đường tròn (O; R) ,( BC<2R),A
l| điểm di động trên cung lớn BC,( A không trùng B,C). Gọi AD, BE, CF là các
đường cao của tam giác ABC;EF cắt BC tại P ,qua D kẻ đường thẳng song song với
EF cắt AC tại Q và cắt AB tại R .


a) Chứng minh tứ giác BQCR là tứ giác nội tiếp


b) Gọi M l| trung điểm cạnh BC .Chứng minh hai tam giác EPM,và DEM là
hai tam gi{c đồng dạng.


c) Chứng minh rằng đường tròn ngoại tiếp tam gi{c PQR luôn đi qua một
điểm cố định



<b>Câu 5</b> (2,0 điểm) Cho các số thực dương x,y,z thỏa mãn <i>x</i>2  <i>y</i>2 <i>z</i>2 3


Chứng minh rằng <i>xy</i> <i>yz</i> <i>xz</i>


<i>xy</i>
<i>z</i>
<i>xz</i>
<i>y</i>
<i>yz</i>
<i>x</i>





3
3
3


</div>
<span class='text_page_counter'>(53)</span><div class='page_container' data-page=53>

<b>Sưu tầm và tổng hợp Trịnh Bình </b> TÀI LIỆU TOÁN HỌC


<b>HƯỚNG DẪN </b>
<b>Câu 1</b> (3,0 điểm)


a)Tìm các số nguyên x, y thỏa mãn phương trình: <i>x</i>2  <i>y</i>2 <i>xy</i> <i>x</i><i>y</i>2.


b) Chứng minh rằng với ba số tự nhiên a,b,c trong đó có đúng một số lẻ và hai số chẵn ta
ln có

<i>a</i><i>b</i><i>c</i>

 

3  <i>a</i><i>b</i><i>c</i>

 

3  <i>b</i><i>c</i><i>a</i>

 

3  <i>a</i><i>b</i><i>c</i>

3 Chia hết cho 96



<b>Hướng dẫn </b>
a)
6
)
1
(
)
1
(
)
(
6
2
2
2
2
2
2
2
2
2
2
2
2
2
2




















<i>y</i>
<i>x</i>
<i>y</i>
<i>x</i>
<i>y</i>
<i>x</i>
<i>xy</i>
<i>y</i>
<i>x</i>
<i>y</i>
<i>x</i>
<i>xy</i>
<i>y</i>
<i>x</i>



PT có 6 nghiệm (<i>x</i>;<i>y</i>)

    

2;0; 3;2; 1;0

và 3 hoán vị
Đặt a+b-c =z; b+c-a=x; a+c-b=y thì x+y+z=a+b+c


Ta có

<i>x</i><i>y</i><i>z</i>

3<i>x</i>3 <i>y</i>3<i>z</i>3 3(<i>x</i><i>y</i>)(<i>y</i><i>z</i>)(<i>x</i><i>z</i>)


<b>Câu 2 </b>(4,0 điểm)


c) Chứng minh rằng với mọi số nguyên dương n ta có


2
1
1
1
2
1
1
1
2














<i>n</i>
<i>n</i>
<i>n</i>
<i>n</i>


d) Tính tổng


2
2
2
2
2016
1
2014
1
1
...
5
1
3
1
1
4
1
2
1
1
3
1


1
1 




 <sub></sub>








 







 








 


<i>S</i>


<b>Hướng dẫn a) </b>


2
2
2
2
2
2
2
2
2
2
1
1
1
)
2
(
2
2
2
2
)
2


(
1
1
1
2
1
1
1
)
2
(
2
)
2
(
4
)
2
(
1
1
1
)
2
(
2
)
2
(
1

1
1
2
1
1
1




















































<i>n</i>
<i>n</i>
<i>n</i>

<i>n</i>
<i>n</i>
<i>n</i>
<i>n</i>
<i>n</i>
<i>n</i>
<i>n</i>
<i>n</i>
<i>n</i>
<i>n</i>
<i>n</i>
<i>n</i>
<i>n</i>
<i>n</i>
<i>n</i>
<i>n</i>
<i>n</i>
<i>n</i>
<i>n</i>
Nên
2
1
1
1
2
1
1
1
2














<i>n</i>
<i>n</i>
<i>n</i>
<i>n</i>
b)
2016
1
2015
1
2
1
2015
2016
1
2004
1
1
...

5
1
3
1
1
4
1
2
1
1
3
1
1


1               




<i>S</i>


<b>Câu 3 </b>(4,0 điểm)


a) Giải phương trình


2<i>x</i>2 <i>x</i> 2<i>x</i><i>x</i>2


b) Giải hệ phương trình


</div>
<span class='text_page_counter'>(54)</span><div class='page_container' data-page=54>

<b>Sưu tầm và tổng hợp Trịnh Bình </b> TÀI LIỆU TỐN HỌC
<b>Hướng dẫn </b>


a) ĐKXĐ:















2
2
1
0
0
)
2
(
0
)
1
2
(
<i>x</i>

<i>x</i>
<i>x</i>
<i>x</i>
<i>x</i>
<i>x</i>






































1
2
2
0
1
2
2
0
)
1
(
2
1
2
2
2
2
2
2
2
2

2
2
2
2
2
<i>x</i>
<i>x</i>
<i>x</i>
<i>x</i>
<i>x</i>
<i>x</i>
<i>x</i>
<i>x</i>
<i>x</i>
<i>x</i>
<i>x</i>
<i>x</i>
<i>x</i>
<i>x</i>
<i>x</i>
<i>x</i>
<i>x</i>
<i>x</i>
<i>x</i>
<i>x</i>
<i>x</i>
<i>x</i>
<i>x</i>
<i>x</i>
<i>x</i>

<i>x</i>


Giải ra x = 1 hoặc x = 1


b)

 

















)
2
(
;
0
6
2
4
)
1


(
;
1
2
1
1
2
2
2
2
<i>y</i>
<i>x</i>
<i>y</i>
<i>x</i>
<i>xy</i>
<i>x</i>
<i>y</i>
<i>y</i>
<i>x</i>


từ PT (1) ta có :



























1
2
0
)
1
)(
2
(
0
)
1
(
2
)

1
)(
(
0
)
1
(
2
)
(
2
2
<i>xy</i>
<i>x</i>
<i>y</i>
<i>xy</i>
<i>y</i>
<i>x</i>
<i>xy</i>
<i>xy</i>
<i>y</i>
<i>x</i>
<i>xy</i>
<i>y</i>
<i>x</i>
<i>xy</i>
<i>y</i>
<i>x</i>


thay vào PT (2) giải ra có 5 nghiệm


  















 




















5
14
;
5
4
;
3
1
;
2
1
3
;
1
3
;
2
1
3
;
2
;
5
,
0
;

1
;
1
)
(<i>xy</i>
<b>M</b>
<b>R</b>
<b>Q</b>
<b>D</b>
<b>P</b>
<b>E</b>
<b>F</b>
<b>O</b>
<b>B</b>
<b>C</b>
<b>A</b>


a) Do tứ giác BCEF nội tiếp suy ra <i>AFE</i> <i>BCQ</i>mà <i>AFE</i> <i>BRQ</i> ( so le )
Suy ra <i>BCQ</i><i>BRQ</i> nên tứ giác BQCR nội tiếp


b) EM là trung tuyến tam giác vuông BEC nên tam giác ECM cân tại M suy ra


<i>ACB</i>


<i>EMD</i> 


</div>
<span class='text_page_counter'>(55)</span><div class='page_container' data-page=55>

<b>Sưu tầm và tổng hợp Trịnh Bình </b> TÀI LIỆU TOÁN HỌC


0



180
2     




<i>EMD</i> <i>ACB</i> <i>AFE</i> <i>BFD</i> <i>EMD</i> <i>DFE</i> suy ra tứ giác DMEF nội tiếp
suy ra <i>BDF</i> <i>PEM</i> mà <i>BDF</i><i>BAC</i> <i>MDE</i> nên tam gi{c EPM,v| DEM đồng dạng
(g.g)


c)do DMEF nội tiếp suy ra <i>PFD</i><i>EMD</i> mà <i>PDF</i><i>EDM</i> nên tam gi{c PFD đồng
dạng tam giác EMD (g.g) suy ra ;


<i>MD</i>
<i>ED</i>
<i>DF</i>


<i>PD</i> <sub></sub>


do <i>RED</i><i>AEF</i> <i>FRD</i> nên tam giác FDR
cân tại D suy ra FD=DR;tương tự tam giác DEQ cân tại D nên DE=DQ


mà FD=DR; DE=DQ suy ra ;


<i>MD</i>
<i>DQ</i>
<i>DR</i>


<i>PD</i> <sub></sub>


suy ra tam gi{c PDR đồng dang tam giác QDM ( c.g.c) suy ra <i>PRQ</i><i>PMQ</i> suy ra tứ giác


PRMQ nội tiếp nên đường tròn ngoại tiếp tam gi{c PQR đi qua điểm M cố định


<b>Câu 5 </b><i><b>( 2.0 điểm)</b></i>


Cho các số thực dương x,y,z thảo mãn <i>x</i>2  <i>y</i>2 <i>z</i>2 3
Chứng minh rằng <i>xy</i> <i>yz</i> <i>xz</i>


<i>xy</i>
<i>z</i>
<i>xz</i>
<i>y</i>
<i>yz</i>
<i>x</i>





3
3
3
<b>Hướng dẫn </b>
3
3
3
3
3
3
3
3


3 <i><sub>xyz</sub></i>
<i>z</i>
<i>z</i>
<i>xyz</i>
<i>y</i>
<i>y</i>
<i>xyz</i>
<i>x</i>
<i>x</i>
<i>xy</i>
<i>z</i>
<i>xz</i>
<i>y</i>
<i>yz</i>
<i>x</i>


<i>A</i>     


Ta có 3 2  2  2 33 2 2 2  1


<i>xyz</i>
<i>z</i>
<i>y</i>
<i>x</i>
<i>z</i>
<i>y</i>
<i>x</i>


Nên <i><sub>A</sub></i> <i><sub>x</sub></i>3 <i><sub>x</sub></i> <i><sub>y</sub></i>3 <i><sub>y</sub></i> <i><sub>z</sub></i>3 <i><sub>z</sub></i>



Áp dụng BĐT Bunhia cho 2 dãy dãy 1 : 3 2 3 2 3 2
;
; <i>y</i> <i>z</i>
<i>x</i>


Dãy 2 : 3 <i><sub>x</sub></i><sub>;</sub>3 <i><sub>y</sub></i><sub>;</sub>3 <i><sub>z</sub></i>


<i>x</i>3 <i>x</i><i>y</i><sub>3</sub> <i>y</i> <i>z</i>3 <i>z</i>

3 <i>x</i>2 3 <i>y</i>2 3 <i>z</i>2

<i>x</i><i>y</i><i>z</i>

2 3

<i>xy</i><i>yz</i><i>xz</i>

<sub>(*) </sub>
Ấp dụng Côsi


3
1
1
1
.
1
.
2
3 2  <i>x</i>  


<i>x</i> ;
3
1
1
1
.
1
.
2
3 2  <i>y</i>  



<i>y</i> ;
3
1
1
1
.
1
.
2
3 2  <i>z</i>  


<i>z</i>


Nên 3


3
6
.
2
2
2
3 2
3 2


3 2    <i>x</i> <i>y</i> <i>z</i>  


<i>z</i>
<i>y</i>
<i>x</i>



Thay Vào (*) Ta có


<i>xz</i>
<i>yz</i>
<i>xy</i>
<i>z</i>
<i>z</i>
<i>y</i>
<i>y</i>
<i>x</i>
<i>x</i>


<i>A</i> 3  3  3   


Hay <i>xy</i> <i>yz</i> <i>xz</i>


<i>xy</i>
<i>z</i>
<i>xz</i>
<i>y</i>
<i>yz</i>
<i>x</i>





3
3


3


</div>
<span class='text_page_counter'>(56)</span><div class='page_container' data-page=56>

<b>Sưu tầm và tổng hợp Trịnh Bình </b> TÀI LIỆU TOÁN HỌC
<b>Cách khác </b>
3
1
1
.


3 <i><sub>yz</sub></i>  <i>y</i><i>z</i> <sub>;</sub>


3
1
1


.


3 <i><sub>xz</sub></i>  <i>x</i><i>z</i>
;


3
1
1


.


3 <i><sub>yx</sub></i>  <i>y</i><i>x</i>
Nên
3
;


3
9
)
(
3
)
(
:
3
)
(
)
(
3
)
(
2
)
(
3
)
(
2
)
(
3
3
1
1
1

3
2
2
2
2
2
2
2
2
2
2
2
2
2
2
2
2
2
2
2
2
3
3
3

















































































<i>z</i>
<i>y</i>
<i>x</i>
<i>xz</i>
<i>yz</i>
<i>xy</i>
<i>z</i>
<i>y</i>
<i>x</i>
<i>z</i>
<i>y</i>
<i>x</i>
<i>z</i>
<i>y</i>
<i>x</i>
<i>z</i>
<i>y</i>
<i>x</i>

<i>do</i>
<i>xz</i>
<i>yz</i>
<i>xy</i>
<i>z</i>
<i>y</i>
<i>x</i>
<i>z</i>
<i>y</i>
<i>x</i>
<i>z</i>
<i>y</i>
<i>x</i>
<i>xz</i>
<i>yz</i>
<i>xy</i>
<i>z</i>
<i>y</i>
<i>x</i>
<i>z</i>
<i>y</i>
<i>x</i>
<i>xz</i>
<i>yz</i>
<i>xy</i>
<i>z</i>
<i>y</i>
<i>x</i>
<i>B</i>
<i>B</i>

<i>z</i>
<i>xz</i>
<i>yz</i>
<i>z</i>
<i>y</i>
<i>yz</i>
<i>xy</i>
<i>y</i>
<i>x</i>
<i>xz</i>
<i>xy</i>
<i>x</i>
<i>x</i>
<i>y</i>
<i>z</i>
<i>z</i>
<i>x</i>
<i>y</i>
<i>z</i>
<i>y</i>
<i>x</i>
<i>xy</i>
<i>z</i>
<i>xz</i>
<i>y</i>
<i>yz</i>
<i>x</i>
<i>A</i>


<i>Có thể cịn cách khác hoặc cách giải chưa chính xác mong các bạn bổ sung nhé </i>



</div>
<span class='text_page_counter'>(57)</span><div class='page_container' data-page=57>

<b>Sưu tầm và tổng hợp Trịnh Bình </b> TÀI LIỆU TỐN HỌC


<b>SỞ GIÁO DỤC VÀ ĐÀO TẠO </b>



<b>PHÚ THỌ </b>


<b>ĐỀ THI HỌC SINH GIỎI </b>


<b>Mơn: TỐN </b>



<b>NĂM HỌC 2015 – 2016 </b>


<b>Câu 1 (3,0 </b>

<i><b>điểm</b></i>

<b>). </b>



a)

Cho S =

1.2.32.3.4...<i>n</i>(<i>n</i>1)(<i>n</i>2)

với

<i>n</i>

là số tự nhiên khác

0

.



Chứng minh rằng

4

S

1

là số chính phương.



b)

Tìm các số ngun

<i>x</i>

<i>y</i>

thỏa mãn

<i>x</i>2 2<i>y</i>2 2<i>xy</i> <i>y</i>2

.



<b>Câu 2 (4,0 </b>

<i><b>điểm</b></i>

<b>). </b>



a)

Tính giá trị biểu thức P =



11
2
3
9
17
4
2


4
3
5






<i>x</i>
<i>x</i>
<i>x</i>
<i>x</i>
<i>x</i>
<i>x</i>

với


4
1
1
2   


<i>x</i>
<i>x</i>


<i>x</i>


.



b)

Cho

<i>a</i>,<i>b</i>,<i>c</i>

là các số thực dương thỏa mãn

<i>a</i><i>b</i><i>c</i>5

<i>a</i> <i>b</i> <i>c</i> 3

.




Chứng minh rằng



)
2
)(
2
)(
2
(
4
2
2


2       


 <i>c</i> <i>a</i> <i>b</i> <i>c</i>


<i>c</i>
<i>b</i>
<i>b</i>
<i>a</i>
<i>a</i>

.



<b>Câu 3 (4,0 </b>

<i><b>điểm</b></i>

<b>). </b>



a)

Giải phương trình

3


2
3


5
1
2
1


3<i>x</i> <i>x</i>2   <i>x</i>2  <i>x</i>

.



b)

Giải hệ phương trình


















0
4
0
2
5
2


2
2
2
2
<i>y</i>
<i>x</i>
<i>y</i>
<i>x</i>
<i>x</i>
<i>y</i>
<i>xy</i>
<i>y</i>
<i>x</i>

.



<b>Câu 4 (7,0 </b>

<i><b>điểm</b></i>

<b>). </b>



Cho đường tròn t}m O đường kính AB =

2

R. Gọi M là một điểm bất kỳ



thuộc đường tròn (O) (M khác A, khác B). Các tiếp tuyến với đường tròn


(O) tại A và M cắt nhau ở E. Vẽ MP vng góc với AB (P thuộc AB). Vẽ


MQ vng góc với AE (Q thuộc AE).



a)

Chứng minh rằng AEMO là tứ giác nội tiếp và APMQ là hình chữ nhật.


b)

Chứng minh rằng PQ, OE, MA đồng qui.



c)

Gọi K l| giao điểm của EB và MP. Chứng minh rằng K l| trung điểm MP.


d)

Đặt AP =

<i>x</i>

, tính MP theo R và

<i>x</i>

. Tìm vị trí của M trên đường trịn (O)



để hình chữ nhật APMQ có diện tích lớn nhất.




<b>Câu 5 (2,0 </b>

<i><b>điểm</b></i>

<b>). </b>

Cho các số thực phân biệt

<i>a</i>,<i>b</i>,<i>c</i>

. Chứng minh rằng



2
9
)
(
1
)
(
1
)
(
1
2
2
2
2
2
2 














<i>a</i>
<i>c</i>
<i>c</i>
<i>b</i>
<i>b</i>
<i>a</i>
<i>c</i>
<i>b</i>
<i>a</i>

.



</div>
<span class='text_page_counter'>(58)</span><div class='page_container' data-page=58>

<b>---Sưu tầm và tổng hợp Trịnh Bình </b> TÀI LIỆU TỐN HỌC


<b>SỞ GIÁO DỤC VÀ ĐÀO TẠO </b>



<b>PHÚ THỌ </b>


<b>HƯỚNG DẪN CHẤM </b>


<b>ĐỀTHI HỌC SINH GIỎI </b>



<b>Mơn: TỐN </b>


<b>NĂM HỌC 2015 – 2016 </b>



<b>I. Một số chú ý khi chấm bài </b>


 Hướng dẫn chấm thi dưới đ}y dựa v|o lời giải sơ lược của một c{ch, khi chấm thi
gi{m khảo cần b{m s{t yêu cầu trình b|y lời giải đầy đủ, chi tiết, hợp logic v| có thể
chia nhỏ đến 0,25 điểm.



 Thí sinh l|m b|i theo c{ch kh{c với Hướng dẫn chấm m| đúng thì tổ chấm cần thống
nhất cho điểm tương ứng với thang điểm của Hướng dẫn chấm.


<b>Điểm bài thi</b> l| tổng c{c điểm th|nh phần khơng l|m trịn số.


<b>II. Đáp án và biểu điểm </b>


<b>Câu 1 (3,0 </b>

<i><b>điểm</b></i>

<b>). </b>



a)

Cho S =

1.2.32.3.4...<i>n</i>(<i>n</i>1)(<i>n</i>2)

với

<i>n</i>

là số tự nhiên khác

0

.



Chứng minh rằng

4

S

1

là số chính phương.



b)

Tìm các số nguyên

<i>x</i>

<i>y</i>

thỏa mãn

<i>x</i>2 2<i>y</i>2 2<i>xy</i> <i>y</i>2

.



<b>ĐÁP ÁN </b> <b>ĐIỂM </b>


<b>a) (1,5 điểm).</b> Ta có


4

S =

1.2.3.42.3.4.(51)3.4.5.(62)...<i>n</i>(<i>n</i>1)(<i>n</i>2)

(<i>n</i>3)(<i>n</i>1)

0,25


=

1.2.3.42.3.4.51.2.3.43.4.5.62.3.4.5...<i>n</i>(<i>n</i>1)(<i>n</i>2)(<i>n</i>3)(<i>n</i>1)<i>n</i>(<i>n</i>1)(<i>n</i>2) 0,25


=

<i>n</i>(<i>n</i>1)(<i>n</i>2)(<i>n</i>3) 0,25


Do đó

4

S

1

=

<i>n</i>(<i>n</i>1)(<i>n</i>2)(<i>n</i>3)1

=

23



23 2

1


<i>n</i>
<i>n</i>


<i>n</i>


<i>n</i> 0,25


=

<i>n</i>23<i>n</i>

 

22<i>n</i>23<i>n</i>

1 0,25


=

2

2
1
3 


 <i>n</i>


<i>n</i>

. Vậy

4

S

1

là số chính phương.

0,25


<b>b) (1,5 điểm). </b>Ta có


<i>x</i> <i>y</i>

<i>y</i> <i>y</i>

<i>x</i> <i>y</i>

 

<i>y</i>



<i>y</i>



<i>y</i>
<i>xy</i>
<i>y</i>


</div>
<span class='text_page_counter'>(59)</span><div class='page_container' data-page=59>

<b>Sưu tầm và tổng hợp Trịnh Bình </b> TÀI LIỆU TỐN HỌC


Với <i>y</i>1, PT trở th|nh <i>x</i>22<i>x</i>10<i>x</i>1<i>Z</i> 0,25


Với <i>y</i>0, PT trở th|nh <i>x</i>220<i>x</i><i>Z</i> 0,25


Với <i>y</i>1, PT trở th|nh <i>x</i>22<i>x</i>10<i>x</i><i>Z</i> 0,25
Với <i>y</i>2, PT trở th|nh <i>x</i>24<i>x</i>40<i>x</i>2<i>Z</i>.



Vậy có 2 cặp

 

<i>x</i>;<i>y</i> thỏa mãn đề b|i

  

1;1; 2;2

. 0,25


<b>Câu 2 (4,0 </b>

<i><b>điểm</b></i>

<b>). </b>



a)

Tính giá trị biểu thức P =



11
2
3
9
17
4
2
4
3
5






<i>x</i>
<i>x</i>
<i>x</i>
<i>x</i>
<i>x</i>
<i>x</i>

với



4
1
1
2   


<i>x</i>
<i>x</i>


<i>x</i>


.



b)

Cho

<i>a</i>,<i>b</i>,<i>c</i>

là các số thực dương thỏa mãn

<i>a</i><i>b</i><i>c</i>5

<i>a</i> <i>b</i> <i>c</i> 3

.



Chứng minh rằng



)
2
)(
2
)(
2
(
4
2
2


2       


 <i>c</i> <i>a</i> <i>b</i> <i>c</i>



<i>c</i>
<i>b</i>
<i>b</i>
<i>a</i>
<i>a</i>

.



<b>ĐÁP ÁN </b> <b>ĐIỂM </b>


<b>a) (2,0 điểm). </b>


Ta có 4 1 3 1


4
1
1


2
2


2    <i>x</i><i>x</i> <i>x</i>  <i>x</i>  <i>x</i>
<i>x</i>


<i>x</i>


<i>x</i> 0,5


Khi đó <i>x</i>3<i>x</i>2.<i>x</i>

3<i>x</i>1

<i>x</i>3<i>x</i>2<i>x</i>3

3<i>x</i>1

<i>x</i>8<i>x</i>3 0,25
<i>x</i>4<i>x</i>3.<i>x</i>

8<i>x</i>3

<i>x</i>8<i>x</i>23<i>x</i>8

3<i>x</i>1

3<i>x</i>21<i>x</i>8 0,25

<i>x</i>5<i>x</i>4.<i>x</i>

21<i>x</i>8

<i>x</i>21<i>x</i>28<i>x</i>21

3<i>x</i>1

8<i>x</i>55<i>x</i>21 0,25
Suy ra

P =



11
2
3
9
17
4
2
4
3
5






<i>x</i>
<i>x</i>
<i>x</i>
<i>x</i>
<i>x</i>


<i>x</i>

 



21 8

 

33 1

2 11
9
17

3
8
4
21
55











<i>x</i>
<i>x</i>
<i>x</i>
<i>x</i>
<i>x</i>
<i>x</i>
0,25

16
3
32
6 <sub></sub>

<i>x</i>

<i>x</i>


( do <i>x</i>0 ). Vậy P =
16


3


. 0,5


<b>b) (2,0 điểm). </b>Ta có


9 2


2


3          





 <i>b</i> <i>c</i> <i>a</i> <i>b</i> <i>c</i> <i>ab</i> <i>bc</i> <i>ca</i> <i>ab</i> <i>bc</i> <i>ca</i>


<i>a</i> 0,25


Do đó <i>a</i>2<i>a</i> <i>ab</i> <i>bc</i> <i>ca</i> 

<i>a</i> <i>b</i>



<i>a</i> <i>c</i>

0,25


<i>b</i>2<i>b</i> <i>ab</i> <i>bc</i> <i>ca</i> 

<i>b</i> <i>c</i>



<i>b</i> <i>a</i>

0,25


<i>c</i>2<i>c</i> <i>ab</i> <i>bc</i>  <i>ca</i> 

<i>c</i>  <i>a</i>



<i>c</i>  <i>b</i>

0,25



Suy ra




 



 

<i>c</i> <i>a</i>



<i>c</i> <i>b</i>



<i>c</i>
<i>a</i>
<i>b</i>
<i>c</i>
<i>b</i>
<i>b</i>
<i>c</i>
<i>a</i>
<i>b</i>
<i>a</i>
<i>a</i>
<i>c</i>
<i>c</i>
<i>b</i>
<i>b</i>
<i>a</i>
<i>a</i>















2 2 2


</div>
<span class='text_page_counter'>(60)</span><div class='page_container' data-page=60>

<b>Sưu tầm và tổng hợp Trịnh Bình </b> TÀI LIỆU TOÁN HỌC

<sub></sub>

 

<sub></sub>

<sub></sub>

 

<sub></sub>


<i>a</i>
<i>c</i>
<i>c</i>
<i>b</i>
<i>b</i>
<i>a</i>
<i>b</i>
<i>a</i>
<i>c</i>
<i>a</i>
<i>c</i>
<i>b</i>
<i>c</i>
<i>b</i>
<i>a</i>









 0,25


)
2
)(
2
)(
2
(
2






<i>c</i>
<i>b</i>
<i>a</i>
<i>ca</i>
<i>bc</i>
<i>ab</i>
<sub>0,25 </sub>


)
2
)(
2

)(
2
(
4




<i>c</i>
<i>b</i>
<i>a</i>

Vậy


)
2
)(
2
)(
2
(
4
2
2


2       


 <i>c</i> <i>a</i> <i>b</i> <i>c</i>


<i>c</i>
<i>b</i>
<i>b</i>


<i>a</i>
<i>a</i>

.


0,25


<b>Câu 3 (4,0 </b>

<i><b>điểm</b></i>

<b>). </b>



a)

Giải phương trình

3


2
3
5
1
2
1


3<i>x</i> <i>x</i>2   <i>x</i>2  <i>x</i>

.



b)

Giải hệ phương trình



















0
4
0
2
5
2
2
2
2
2
<i>y</i>
<i>x</i>
<i>y</i>
<i>x</i>
<i>x</i>
<i>y</i>
<i>xy</i>
<i>y</i>
<i>x</i>

.



<b>ĐÁP ÁN </b> <b>ĐIỂM </b>


<b>a) (2,0 điểm). </b>
Điều kiện



2
1




<i>x</i> 0,25


PT 2

3<i>x</i>1

2<i>x</i>2110<i>x</i>23<i>x</i>6


4

2<i>x</i>21

2

3<i>x</i>1

2<i>x</i>2 12<i>x</i>23<i>x</i>20


0,25


Đặt 2<i>x</i>2 1<i>t</i> (<i>t</i>0), ta được 4<i>t</i>22

3<i>x</i>1

<i>t</i>2<i>x</i>23<i>x</i>20 0,25


Ta có

2

2

2

2


3
9
6
2
3
2
4
1
3
'         


 <i>x</i> <i>x</i> <i>x</i> <i>x</i> <i>x</i> <i>x</i> 0,25



nên PT















4
3
1
3
4
3
1
3
<i>x</i>
<i>x</i>
<i>t</i>
<i>x</i>
<i>x</i>


<i>t</i>











2
1
2
2
2
<i>x</i>
<i>t</i>
<i>x</i>
<i>t</i>
0,25
Với
2
2

 <i>x</i>


<i>t</i> thì

<sub></sub>

<sub></sub>
























0
8
4
7
2
2
1
2
4
2

2
2
1


2 2 <sub>2</sub> <sub>2</sub> <sub>2</sub>


</div>
<span class='text_page_counter'>(61)</span><div class='page_container' data-page=61>

<b>Sưu tầm và tổng hợp Trịnh Bình </b> TÀI LIỆU TỐN HỌC
Với
2
1
2 
 <i>x</i>


<i>t</i> thì
























0
5
4
4
2
1
1
2
1
2
4
2
1
2
1
2
1
2
2
2
2
2
<i>x</i>
<i>x</i>

<i>x</i>
<i>x</i>
<i>x</i>
<i>x</i>
<i>x</i>
<i>x</i>

2
6
1
2
6
1
2
1














 <i>x</i>

<i>x</i>
<i>x</i>
.
0,25


Kết hợp điều kiện


2
1




<i>x</i> ta được nghiệm của PT l|







   

2
6
1
;
7
60
2


<i>x</i> . 0,25



<b>b) (2,0 điểm). </b>


Xét hệ phương trình


















0
4
0
2
5
2
2
2
2
2


<i>y</i>
<i>x</i>
<i>y</i>
<i>x</i>
<i>x</i>
<i>y</i>
<i>xy</i>
<i>y</i>
<i>x</i>


)
2
(
)
1
(


PT

(1) 2<i>x</i>2<i>y</i>2<i>xy</i><i>y</i>5<i>x</i>20<i>y</i>2

<i>x</i>1

<i>y</i>2<i>x</i>25<i>x</i>20


0,25


Ta có

2

2

2

2


1
9
9
18
9
2
5


2
4
1
'          


 <i>x</i> <i>x</i> <i>x</i> <i>x</i> <i>x</i> <i>x</i> 0,25


Khi đó PT



















2
1
3
1

2
1
3
1
)
1
(
<i>x</i>
<i>x</i>
<i>y</i>
<i>x</i>
<i>x</i>
<i>y</i>
0,25
<sub></sub>








1
2
2
<i>x</i>
<i>y</i>
<i>x</i>
<i>y</i>

0,25
Với <i>y</i><i>x</i>2, thay vào PT (2) ta được 2<i>x</i>24<i>x</i>20<i>x</i>1<i>y</i>1 0,25
Với <i>y</i>2<i>x</i>1, thay vào PT (2) ta được <sub></sub>











5
4
1
0
4
5 2
<i>x</i>
<i>x</i>
<i>x</i>
<i>x</i> <sub>0,25 </sub>


*) <i>x</i>1<i>y</i>1 *)


5
13
5



4<sub></sub> <sub></sub><sub></sub>




 <i>y</i>


<i>x</i> 0,25


Vậy nghiệm của hệ phương trình l|

 

1;1 và 




<sub></sub> <sub></sub>
5
13
;
5
4
0,25


<b>Câu 4 (7,0 </b>

<i><b>điểm</b></i>

<b>). </b>



Cho đường trịn t}m O đường kính AB =

2

R. Gọi M là một điểm bất kỳ



</div>
<span class='text_page_counter'>(62)</span><div class='page_container' data-page=62>

<b>Sưu tầm và tổng hợp Trịnh Bình </b> TÀI LIỆU TOÁN HỌC

a)

Chứng minh rằng AEMO là tứ giác nội tiếp và APMQ là hình chữ nhật.


b)

Chứng minh rằng PQ, OE, MA đồng qui.



c)

Gọi K l| giao điểm của EB và MP. Chứng minh rằng K l| trung điểm MP.



d)

Đặt AP =

<i>x</i>

, tính MP theo R và

<i>x</i>

. Tìm vị trí của M trên đường trịn (O) để



hình chữ nhật APMQ có diện tích lớn nhất.



<b>ĐÁP ÁN </b> <b>ĐIỂM </b>


<b> </b>


P


Q


E
I


A
B


O


M


K


<b>a) (2,0 điểm). </b>


Xét tứ gi{c AEMO có góc OAE = OME = 900 nên tứ gi{c AEMO nội tiếp. 1,0
Xét tứ gi{c APMQ có góc MPA = PAQ = AQM = 900nên tứ gi{c APMQ l| hình


chữ nhật. 1,0



<b>b) (2,0 điểm). </b>


Do APMQ l| hình chữ nhật nên hai đường chéo PQ v| MA cắt nhau tại trung điểm I
của mỗi đường.


1,0


Do tiếp tuyến tại A v| M cắt nhau tại E, I l| trung điểm MA nên O, I, E thẳng h|ng.


Vậy PQ, OE, MA đồng qui tại I 1,0


<b>c) (2,0 điểm). </b>


O l| trung điểm AB, I l| trung điểm MA nên OI song song với MB MBP = EOA
Mà MPB = EAO = 900 nên MPB đồng dạng với EAO (g.g).


0,5


Suy ra PB : AO = PM : AE  PB. AE = PM. AO (1) 0,5


Do PK song song với AE nên PB : AB = PK : AE  PB. AE = PK. AB (2)


</div>
<span class='text_page_counter'>(63)</span><div class='page_container' data-page=63>

<b>Sưu tầm và tổng hợp Trịnh Bình </b> TÀI LIỆU TOÁN HỌC


Từ (1) v| (2) suy ra PM. AO = PK. AB  PM. 2AO = 2PK. AB  PM = 2PK
(do 2AO = AB)
Vậy K l| trung điểm MP.


0,5



<b>d) (1,0 điểm). </b>


Trong tam giác vng MPO, ta có MP2<sub> = OM</sub>2<sub> – OP</sub>2<sub> = R</sub>2<sub> – (R - </sub><i><sub>x</sub></i><sub>)</sub>2<sub> khi P thuộc đoạn </sub>
OA


MP2<sub> = OM</sub>2<sub> – OP</sub>2<sub> = R</sub>2<sub> – (</sub><i><sub>x</sub></i><sub> - R)</sub>2<sub> khi P thuộc đoạn OB </sub>
Khi đó MP2<sub> = (2R - </sub><i><sub>x</sub></i><sub>)</sub><i><sub>x</sub></i><sub>. Suy ra MP = </sub> <sub>(</sub><sub>2</sub><i><sub>R</sub></i><i><sub>x</sub></i><sub>)</sub><i><sub>x</sub></i><sub> </sub>


0,25


Diện tích hình chữ nhật APMQ l| S = MP. AP = 3
)
2


( <i>R</i><i>x</i> <i>x</i> 0,25


Áp dụng BĐT 4


4
2


2 <i>ab</i> <i>cd</i> <i>abcd</i>


<i>d</i>
<i>c</i>
<i>b</i>


<i>a</i>      với mọi <i>a</i>,<i>b</i>,<i>c</i>,<i>d</i> 0
hay



4
4 







   
 <i>a</i> <i>b</i> <i>c</i> <i>d</i>


<i>abcd</i> . Dấu “=” xảy ra <i>a</i><i>b</i><i>c</i><i>d</i>


0,25


S = 2


4
3
4
3
3
4
3
3
3
2
.
27


3
.
3
.
3
).
2
(
27
)
2
( <i>R</i>
<i>x</i>
<i>x</i>
<i>x</i>
<i>x</i>
<i>R</i>
<i>x</i>
<i>x</i>
<i>x</i>
<i>x</i>
<i>R</i>
<i>x</i>
<i>x</i>
<i>R</i> 










 <sub></sub> <sub></sub> <sub></sub> <sub></sub>





Dấu “=” xảy ra <i>R</i> <i>x</i> <i>x</i> <i>x</i> <i>R</i>


2
3
3


2    


 . Suy ra P l| trung điểm OB. Do đó ta x{c định
được M để diện tích hình chữ nhật APMQ lớn nhất.


0,25


<b>Câu 5 (2,0 </b>

<i><b>điểm</b></i>

<b>). </b>

Cho các số thực phân biệt

<i>a</i>,<i>b</i>,<i>c</i>

. Chứng minh rằng




2
9
)
(

1
)
(
1
)
(
1
2
2
2
2
2
2 













<i>a</i>
<i>c</i>
<i>c</i>
<i>b</i>

<i>b</i>
<i>a</i>
<i>c</i>
<i>b</i>
<i>a</i>

.



<b>ĐÁP ÁN </b> <b>ĐIỂM </b>


Ta có 






 <sub></sub>







 <sub></sub>








 <sub></sub>








 <sub></sub>







 <sub></sub>







 <sub></sub>


1
1

1
1
1
1
<i>a</i>
<i>c</i>
<i>a</i>
<i>c</i>
<i>c</i>
<i>b</i>
<i>c</i>
<i>b</i>
<i>b</i>
<i>a</i>
<i>b</i>
<i>a</i>
<i>a</i>
<i>c</i>
<i>a</i>
<i>c</i>
<i>c</i>
<i>b</i>
<i>c</i>
<i>b</i>
<i>b</i>
<i>a</i>
<i>b</i>
<i>a</i>


. . . 1


















<i>b</i>
<i>a</i>
<i>b</i>
<i>a</i>
<i>a</i>
<i>c</i>
<i>a</i>
<i>c</i>
<i>a</i>
<i>c</i>
<i>a</i>
<i>c</i>
<i>c</i>
<i>b</i>


<i>c</i>
<i>b</i>
<i>c</i>
<i>b</i>
<i>c</i>
<i>b</i>
<i>b</i>
<i>a</i>
<i>b</i>
<i>a</i> 0,25


Khi đó 0


2















<i>a</i>


<i>c</i>
<i>a</i>
<i>c</i>
<i>c</i>
<i>b</i>
<i>c</i>
<i>b</i>
<i>b</i>
<i>a</i>
<i>b</i>
<i>a</i>
2.
2
2
2






























<i>a</i>
<i>c</i>
<i>a</i>
<i>c</i>
<i>c</i>
<i>b</i>
<i>c</i>
<i>b</i>
<i>b</i>
<i>a</i>
<i>b</i>
<i>a</i>
2
.
.
. 















<i>b</i>
<i>a</i>
<i>b</i>
<i>a</i>
<i>a</i>
<i>c</i>
<i>a</i>
<i>c</i>
<i>a</i>
<i>c</i>
<i>a</i>
<i>c</i>
<i>c</i>
<i>b</i>
<i>c</i>
<i>b</i>

<i>c</i>
<i>b</i>
<i>c</i>
<i>b</i>
<i>b</i>
<i>a</i>
<i>b</i>
<i>a</i>
0,25


</div>
<span class='text_page_counter'>(64)</span><div class='page_container' data-page=64>

<b>Sưu tầm và tổng hợp Trịnh Bình </b> TÀI LIỆU TỐN HỌC
2
2
2
2
2
2
2
2
2
2
2
2
2
2
2
2
2
2
)


(
)
(
)
(
)
(
)
(
)
(
)
(
)
(
)
(
)
(
)
(
)
(
.
2
<i>a</i>
<i>c</i>
<i>a</i>
<i>c</i>
<i>a</i>

<i>c</i>
<i>c</i>
<i>b</i>
<i>c</i>
<i>b</i>
<i>c</i>
<i>b</i>
<i>b</i>
<i>a</i>
<i>b</i>
<i>a</i>
<i>b</i>
<i>a</i>
<i>a</i>
<i>c</i>
<i>a</i>
<i>c</i>
<i>c</i>
<i>b</i>
<i>c</i>
<i>b</i>
<i>b</i>
<i>a</i>
<i>b</i>
<i>a</i>































































 3 2 3 5


2
2


2
<i>a</i>
<i>c</i>
<i>a</i>
<i>c</i>
<i>c</i>
<i>b</i>
<i>c</i>
<i>b</i>
<i>b</i>
<i>a</i>
<i>b</i>
<i>a</i>
2
5
)
(
)
(
)
( 2
2
2
2
2
2
2
2
2
















<i>a</i>
<i>c</i>
<i>a</i>
<i>c</i>
<i>c</i>
<i>b</i>
<i>c</i>
<i>b</i>
<i>b</i>
<i>a</i>
<i>b</i>
<i>a</i>
(1)


Mặt kh{c 







 <sub></sub>






 <sub></sub>






 <sub></sub>







 <sub></sub>







 <sub></sub>






 <sub></sub>


 1 1 1 1 1 <i>a</i> <i>b</i> 1


<i>c</i>
<i>a</i>
<i>c</i>
<i>b</i>
<i>c</i>
<i>b</i>
<i>a</i>
<i>b</i>
<i>a</i>
<i>c</i>
<i>a</i>
<i>c</i>
<i>b</i>
<i>c</i>
<i>b</i>
<i>a</i>



. . . 1











<i>c</i>
<i>b</i>
<i>a</i>
<i>b</i>
<i>a</i>
<i>c</i>
<i>b</i>
<i>a</i>
<i>c</i>
<i>a</i>
<i>c</i>
<i>b</i>
<i>a</i>
<i>c</i>
<i>b</i>
<i>c</i>
<i>b</i>
<i>a</i> 0,25



Khi đó 0


2












 <i>a</i> <i>b</i>


<i>c</i>
<i>a</i>
<i>c</i>
<i>b</i>
<i>c</i>
<i>b</i>
<i>a</i>
2.
2
2
2



























<i>b</i>
<i>a</i>
<i>c</i>
<i>a</i>
<i>c</i>

<i>b</i>
<i>c</i>
<i>b</i>
<i>a</i>
2
.
.
. 








 <i>b</i> <i>c</i>


<i>a</i>
<i>b</i>
<i>a</i>
<i>c</i>
<i>b</i>
<i>a</i>
<i>c</i>
<i>a</i>
<i>c</i>
<i>b</i>
<i>a</i>
<i>c</i>


<i>b</i>
<i>c</i>
<i>b</i>
<i>a</i>
(2)
0,25


Từ (1) v| (2) suy ra


<sub></sub>












 2 2 <sub>2</sub> <sub>2</sub> <sub>2</sub>


2
)
(
1
)
(


1
)
(
1
<i>a</i>
<i>c</i>
<i>c</i>
<i>b</i>
<i>b</i>
<i>a</i>
<i>c</i>
<i>b</i>
<i>a</i>















 2 2<sub>2</sub> 2 2<sub>2</sub> 2 2<sub>2</sub>



)
(
)
(
)


( <i>c</i> <i>a</i>


<i>a</i>
<i>c</i>
<i>c</i>
<i>b</i>
<i>c</i>
<i>b</i>
<i>b</i>
<i>a</i>
<i>b</i>
<i>a</i>
2
9
2
2
5
2
2
2




























<i>b</i>
<i>a</i>
<i>c</i>
<i>a</i>
<i>c</i>
<i>b</i>
<i>c</i>

<i>b</i>
<i>a</i>
.


Dấu “=” xảy ra

1 1 1 0


0
0






































<i>a</i>
<i>c</i>
<i>c</i>
<i>b</i>
<i>b</i>
<i>a</i>
<i>c</i>
<i>b</i>
<i>a</i>
<i>b</i>
<i>a</i>
<i>c</i>
<i>a</i>
<i>c</i>
<i>b</i>

<i>c</i>
<i>b</i>
<i>a</i>
<i>a</i>
<i>c</i>
<i>a</i>
<i>c</i>
<i>c</i>
<i>b</i>
<i>c</i>
<i>b</i>
<i>b</i>
<i>a</i>
<i>b</i>
<i>a</i>


Chẳng hạn <i>a</i>0,<i>b</i>1,<i>c</i>1.


0,5


</div>
<span class='text_page_counter'>(65)</span><div class='page_container' data-page=65>

<b>Sưu tầm và tổng hợp Trịnh Bình </b> TÀI LIỆU TOÁN HỌC


<b>SỞ GIÁO DỤC VÀ ĐÀO TẠO </b>


<b>PHÚ THỌ </b>


<b>KỲ THI CHỌN HỌC SINH GIỎI LỚP 9 THCS CẤP TỈNH </b>
<b>NĂM HỌC 2016-2017 </b>


<b>Mơn: TỐN </b>



<i>Thời gian làm bài:<b> 150 </b>phút, khơng kể thời gian giao đề </i>
<i>(Đề thi có 03 trang) </i>


<b>Thí sinh làm bài (cả phần trắc nghiệm khách quan và phần tự luận) ra tờ giấy thi. </b>
<b>A. PHẦN TRẮC NGHIỆM KHÁCH QUAN (8 điểm) </b>


<b>Câu 1. </b>Biểu thức


2
5 3
6


<i>x</i>


<i>x</i> <i>x</i>




  có nghĩa khi n|o?


<b>A</b>.   3 <i>x</i> 2. <b>B.</b> 5 2.


3 <i>x</i> <b>C.</b> <i>x</i> 3 hoặc <i>x</i>2. <b> D</b>.


5
3


3
<i>x</i>


   .


<b>Câu 2. </b>Cho biểu thức 4 4 45 2


2 15 3


<i>x x</i> <i>x</i> <i>x</i> <i>x</i>


<i>Q</i>


<i>x</i> <i>x</i> <i>x</i>


   


 


   (<i>x</i>0;<i>x</i>25).
Tìm gi{ trị nhỏ nhất của <i>Q</i>.


<b>A. </b> 2


3. <b>B. </b>


7


3. <b>C. </b>2. <b>D. </b>3.


<b>Câu 3. </b>Trên mặt phẳng tọa độ <i>Oxy</i> cho đường thẳng (<i>d</i>):<i>y</i> 

2<i>m</i>3

<i>x</i>4<i>m</i>3. Gọi <i>h</i>


l| khoảng cách từ điểm <i>O</i> đến đường thẳng (<i>d</i>) . Tìm gi{ trị lớn nhất của <i>h</i>.



<b>A</b>.2 3. <b>B</b>. 13. <b>C</b>. 15. <b>D.</b> 5.


<b>Câu 4. </b>Trên mặt phẳng tọa độ <i>Oxy</i> cho ba điểm <i>A</i>

2;3 ;

 

<i>B</i>  4; 4 ;

 

<i>C</i> 5; 1

. Tính
diện tích tam gi{c <i>ABC</i> .


<b>A</b>. 30,5. <b>B</b>. 28,5. <b>C</b>. 42. <b>D.</b> 38.


<b>Câu 5. </b>Trên mặt phẳng tọa độ <i>Oxy</i> cho ba đường thẳng

 

1


2 1 2


:


3 2 3


<i>d</i> <i>x</i> <i>y</i> ;


 

2


1 1


:


3 2


<i>d</i> <i>y</i>  <i>x</i> ;

  

<i>d</i>3 : 2<i>m</i>3

<i>x</i>3<i>my</i>0<b>. </b>Tìm <i>m</i> để ba đường thẳng đã cho đồng
quy.


<b>A. </b> 1.


2


<b>B. </b>1.


2 <b>C. </b>


3
.


2 <b>D. </b>


2
.
3




<b>Câu 6. </b> Cho Parabol (<i>P</i>): 2


<i>y</i><i>x</i> v| đường thẳng (<i>d</i>) có phương trình




2 2 5 16


<i>y</i> <i>m</i> <i>x</i> <i>m</i> . Tìm gi{ trị của <i>m</i> để (<i>d</i>) luôn cắt (<i>P)</i> tại 2 điểm phân biệt
nằm về hai phía của trục tung.



</div>
<span class='text_page_counter'>(66)</span><div class='page_container' data-page=66>

<b>Sưu tầm và tổng hợp Trịnh Bình </b> TÀI LIỆU TOÁN HỌC


<b>A.</b> 16.


5


<i>m</i> <b>B. </b>3 16.


5
<i>m</i>
 
<b>C.</b><i>m</i> 4 hoặc 16.


5


<i>m</i> <b>D. </b> 16.


5
<i>m</i>


<b>Câu 7.</b> Gọi

<i>x y</i>0; 0

l| nghiệm của phương trình



2 2


9 4 7 2 3 7


<i>x</i>  <i>y</i>  <i>x</i>  <i>y</i> <i>x</i> sao cho
0


<i>y</i> đạt gi{ trị lớn nhất . Tính tổng <i>x</i>0 <i>y</i>0.



<b>A.</b> 4. <b>B. </b> 5.


2


<b>C.</b> 3.


2


<b>D. </b>5.


<b>Câu 8. </b>Tìm <i>m</i> để phương trình 2


( 4) 3 0


<i>x</i>  <i>m</i> <i>x</i>  <i>m</i> có hai nghiệm <i>x x</i>1; 2l| độ dài
hai cạnh góc vng của một tam gi{c vng có độ dài cạnh huyền bằng 26.


<b>A.</b> <i>m</i> 8 hoặc <i>m</i>2 . <b>B. </b><i>m</i>2<b> . </b>


<b>C.</b> <i>m</i> 8 . <b>D. </b><i>m</i>8 hoặc <i>m</i> 2 .


<b>Câu 9. </b>Cho hình thang <i>ABCD </i>(<i>AB//CD</i>) có <i>AB</i>2,5<i>cm AD</i>; 3,5<i>cm BD</i>; 5<i>cm</i> và
<i>DBC</i><i>DAB</i>. Tính tổng <i>BC+DC</i>.


<b>A</b>. 17 (cm) . <b>B</b>. 19 (cm). <b>C</b>. 20 (cm). <b>D. </b>22 (cm)<i>.</i>


<b>Câu 10. </b>Cho tam giác <i>ABC</i> vuông tại <i>A</i> đường phân giác <i>AD</i>,

<i>D</i><i>BC</i>

. Đẳng

thức n|o sau đ}y đúng ?


<b>A</b>. 1 1 3


<i>AB</i> <i>AC</i>  <i>AD</i>. <b>B</b>.


1 1 2


<i>AB</i> <i>AC</i>  <i>AD</i>. <b>C</b>.


1 1 1


<i>AB</i> <i>AC</i>  <i>AD</i>. <b>D.</b>


1 1 2


<i>AB</i>  <i>AC</i>  <i>AD</i>.


<b>Câu 11. </b> Cho tam giác nhọn<i> ABC</i> có 0
30


<i>BAC</i>  , kẻ hai đường cao <i>BD</i>,<i> CE </i>


<i>D</i><i>AC E</i>; <i>AB</i>

. Gọi <i>S S</i>; ' lần lượt l| diện tích <i>ABC</i>,<i>ADE</i>. Tính tỉ số <i>S</i>'


<i>S</i> .


<b>A.</b> 3.


4 <b>B.</b>


1


.


4 <b>C.</b>


1
.


2 <b>D. </b>


3
.
2


<b>Câu 12.</b> Cho tam giác <i>ABC</i> vuông tại <i>A.</i> Kẻ <i>AH </i> <i> BC , HD</i><i>AB, HE</i><i>AC</i>


<i>H</i><i>BC D</i>, <i>AB E</i>, <i>AC</i>

. Đẳng thức n|o sau đ}y đúng ?


<b>A</b>.<i>AD AB</i>.  <i>AE AC</i>. . <b>B</b>.<i>BD BA</i>. <i>CE CA</i>. . <b>C</b>.


2


. . .


<i>AD DB</i><i>AE EC</i><i>AH</i> <b>D.</b> 2


. .


<i>BD BA</i> <i>AH</i>



<b>Câu 13.</b> Cho tam gi{c nhọn <i>ABC</i> có <i>ABC</i> <i>ACB</i>, kẻ đường cao <i>AH</i>, trung tuyến <i>AM</i>


</div>
<span class='text_page_counter'>(67)</span><div class='page_container' data-page=67>

<b>Sưu tầm và tổng hợp Trịnh Bình </b> TÀI LIỆU TOÁN HỌC


<b>A</b>.tan cot - cot .
2


<i>C</i> <i>B</i>


<i>HAM</i>  <b>B</b>.tan cot - cot .
2


<i>B</i> <i>C</i>


<i>HAM</i> 


<b>C</b>.tan tan - tan .
2


<i>C</i> <i>B</i>


<i>HAM</i>  <b>D.</b>tan cos - cos .


2


<i>C</i> <i>B</i>


<i>HAM</i> 



<b>Câu 14.</b> Cho đường trịn tâm <i>O</i>, đường kính <i>AB =2R</i> . Gọi <i>M, N</i> lần lượt là trung
điểm của <i>OA</i>, <i>OB. </i> Qua <i>M </i>kẻ dây cung <i>CD</i>, qua <i>N</i> kẻ dây cung<i> EF</i> sao cho <i>CD//EF </i>


(<i>C, F</i> cùng thuộc nửa đường tròn đường kính <i>AB</i>) và 0
30


<i>CMO</i> . Tính diện tích tứ
giác <i>CDEF</i> theo <i>R</i>.


<b>A</b>.
2


15
8
<i>R</i>


. <b>B</b>.
2


13
4
<i>R</i>


. <b>C</b>.
2


15
4
<i>R</i>



. <b>D.</b>
2


3 15


8
<i>R</i>


.


<b>Câu 15. </b> Cho đường tròn tâm <i>O</i>, đường kính <i>AB=2R</i>. Điểm <i>M</i> thuộc tia đối của tia


<i>AB</i>, qua <i>M</i> kẻ tiếp tuyến <i>MC</i> với đường tròn (<i>O</i>) ( <i>C</i> là tiếp điểm), kẻ <i>CH</i> vng góc
với <i>AB</i>

<i>H</i><i>AB</i>

biết <i>MA</i><i>a MC</i>; 3<i>a</i>(<i>a</i>0).Tính <i>CH</i> theo <i>a. </i>


<b>A. </b>12
5
<i>a</i>


. <b>B</b>. 9
5


<i>a</i>


. <b>C</b>. 8
5


<i>a</i>


. <b>D.</b>14


5
<i>a</i>


.


<b>Câu 16. </b>Một ngọn hải đ ng ở vị trí <i>A</i> c{ch bờ biển (l| đường thẳng) một khoảng
3 ( )


<i>AH</i>  <i>km</i> . Một người g{c hải đ ng muốn từ vị trí <i>A</i> trở về vị trí <i>B</i> trên bờ biển
(<i>HB</i> = 24 (<i>km)</i>)<i>,</i> bằng c{ch ch o thuyền với vận tốc 3<i> (km/h)</i> tới vị trí <i>M</i> trên bờ (<i>M</i>


nằm giữa <i>H</i> v| <i>B</i>) sau đó từ <i>M</i> chạy bộ dọc theo bờ biển đến <i>B</i> với vận tốc gấp bốn
lần vận tốc ch o thuyền. Biết tổng thời gian di chuyển từ <i>A</i> về đến <i>B</i> hết 3 giờ 20
phút. Tính khoảng cách <i>MB</i> ?


HB=24km


3km


<b>M</b> <b><sub>B</sub></b>


<b>H</b>
<b>A</b>


<b>A</b>. 12 (km). <b>B</b>. 16 (km). <b>C</b>. 18 (km) . <b>D.</b> 20 (km).


<b>B. PHẦN TỰ LUẬN (12 điểm) </b>
<b>Câu 1 (3,0 </b><i><b>điểm</b></i><b>) </b>


a) Cho các số dương <i>a b c</i>, , thỏa mãn <i>ab bc ca</i>  1. Chứng minh rằng



2 2 2 0


1 1 1


<i>a b</i> <i>b c</i> <i>c</i> <i>a</i>


<i>c</i> <i>a</i> <i>b</i>


 <sub></sub>  <sub></sub>  <sub></sub>


</div>
<span class='text_page_counter'>(68)</span><div class='page_container' data-page=68>

<b>Sưu tầm và tổng hợp Trịnh Bình </b> TÀI LIỆU TỐN HỌC
b) Chứng minh rằng nếu <i>a b</i>. 3thì hai phương trình sau:


3 2 4


(<i>a</i> <i>a x</i>) <i>a y</i><i>a</i>  1 0; (<i>b</i>3<i>b x b y</i>)  2 <i>b</i>4 1 0 (<i>a,b </i>là các tham số) khơng có
nghiệm ngun chung.


<b>Câu 2(3,5 </b><i><b>điểm</b></i><b>) </b>


a)Giải phương trình 2<i>x</i> 3 <i>x</i> 1 1.
b) Giải hệ phương trình


3 2 2 2


3 5 4


3 1 1.



<i>x</i> <i>x y</i> <i>x</i> <i>xy</i> <i>y</i> <i>x</i> <i>y</i>


<i>x</i> <i>y</i> <i>x</i>


      





   


 .


<b>Câu 3(4,0 </b><i><b>điểm</b></i><b>). </b>Cho đường tròn ( ; )<i>O R</i> v| điểm <i>A</i> cố định trên ( ; )<i>O R</i> . Gọi <i>M, N </i> là
c{c giao điểm của hai đường tròn ( ; )<i>O R</i> và ( ; )<i>A R</i> ; <i>H</i> l| điểm thay đổi trên cung nhỏ
<i>MN</i> của đường tròn ( ; )<i>A R</i> . Đường thẳng qua <i>H</i> và vng góc với <i>AH</i> cắt ( ; )<i>O R</i> tại


<i>B, C</i>. Kẻ <i>HI</i> <i>AB I</i>( <i>AB HK</i>), <i>AC K</i>( <i>AC</i>).


a) Chứng minh rằng <i>IK</i> luôn vng góc với một đường thẳng cố định và


2


. 2


<i>AB AC</i> <i>R</i> .


b) Tìm giá trị lớn nhất của diện tích <i>AIK</i> khi <i>H</i> thay đổi.


<b>Câu 4(1,5 </b><i><b>điểm</b></i><b>). </b>Cho các số dương <i>a b c</i>, , thỏa mãn <i>a b c</i>  1. Tìm giá trị nhỏ


nhất của biểu thức 2 2 2 2 2 2


2( ) ( ) 4 .


<i>P</i> <i>a b b c</i> <i>c a</i>  <i>a</i> <i>b</i> <i>c</i>  <i>abc</i>
...HẾT...


<i>Họ và tên thí sinh: ... SBD: ... </i>


</div>
<span class='text_page_counter'>(69)</span><div class='page_container' data-page=69>

<b>Sưu tầm và tổng hợp Trịnh Bình </b> TÀI LIỆU TỐN HỌC


<b>SỞ GIÁO DỤC VÀ ĐÀO TẠO </b>
<b>PHÚ THỌ </b>


<b>KỲ THI CHỌN HỌC SINH GIỎI LỚP 9 THCS CẤP TỈNH </b>
<b>NĂM HỌC 2016-2017 </b>


<b>HƯỚNG DẪN CHẤM MƠN TỐN </b>


Hướng dẫn chấm có 05 trang



<b>I.</b> <b>Một số chú ý khi chấm bài </b>


- Đ{p {n chấm thi dưới đ}y dựa vào lời giải sơ lược của một cách. Khi chấm thi giám
khảo cần bám sát yêu cầu trình bày lời giải đầy đủ, chi tiết, hợp logic và có thể chia
nhỏ đến 0,25 điểm.


- Thí sinh làm bài theo cách khác với đ{p m| đúng thì tổ chấm cần thống nhất cho
điểm tương ứng với thang điểm của đ{p {n.



- Điểm bài thi là tổng điểm các câu khơng làm trịn số.


<b>II.</b> <b>Đáp án – thang điểm </b>


<b>1.</b> <b>Phần trắc nghiệm khách quan </b>


<b>Câu </b> <b>1 </b> <b>2 </b> <b>3 </b> <b>4 </b> <b>5 </b> <b>6 </b> <b>7 </b> <b>8 </b> <b>9 </b> <b>10 11 12 13 14 15 16 </b>


<b>Đáp </b>
<b>án </b>
<b>đúng </b>


<b>D </b> <b>C </b> <b>B </b> <b>B </b> <b>A </b> <b>D </b> <b>A </b> <b>B </b> <b>A </b> <b>B </b> <b>A </b> <b>A, </b>


<b>C </b> <b>A </b> <b>C </b> <b>A </b> <b>D </b>


<b>Điểm </b> 0,5 0,5 0,5 0,5 0,5 0,5 0,5 0,5 0,5 0,5 0,5 0,5 0,5 0,5 0,5 0,5


<b>2.</b> <b>Phần tự luận </b>


<b>Nội dung </b> <b>Điểm </b>


<b>Câu 1. (3,0 điểm) </b>


a) Cho các số dương <i>a b c</i>, , thỏa mãn <i>ab bc ca</i>  1. Chứng minh rằng


2 2 2 0


1 1 1



<i>a b</i> <i>b c</i> <i>c</i> <i>a</i>


<i>c</i> <i>a</i> <i>b</i>


 <sub></sub>  <sub></sub>  <sub></sub>


  


<b>1,5 </b>


Ta có 2 2


1<i>a</i> <i>ab bc</i> <i>ca</i><i>a</i> (<i>a</i><i>b a</i>)( <i>c</i>). <sub>0,25 </sub>


Tương tự


2 2


2 2


1 ( )( );


1 ( )( ).


<i>b</i> <i>ab</i> <i>bc</i> <i>ca</i> <i>b</i> <i>b</i> <i>a b</i> <i>c</i>


<i>c</i> <i>ab</i> <i>bc</i> <i>ca</i> <i>c</i> <i>c</i> <i>a c</i> <i>b</i>


       



       


0,25


</div>
<span class='text_page_counter'>(70)</span><div class='page_container' data-page=70>

<b>Sưu tầm và tổng hợp Trịnh Bình </b> TÀI LIỆU TỐN HỌC


<b>Nội dung </b> <b>Điểm </b>


Suy ra <sub>2</sub> 1 1


1 ( )( )


<i>a b</i> <i>a b</i>


<i>c</i> <i>c</i> <i>a c b</i> <i>c b</i> <i>c</i> <i>a</i>


 <sub></sub>  <sub></sub> <sub></sub>


     . 0,25



2


2


1 1


;


1 ( )( )



1 1


.


1 ( )( )


<i>b c</i> <i>b c</i>


<i>a</i> <i>a</i> <i>b a</i> <i>c</i> <i>a</i> <i>c</i> <i>a</i> <i>b</i>


<i>c</i> <i>a</i> <i>c</i> <i>a</i>


<i>b</i> <i>b</i> <i>a b</i> <i>c</i> <i>b</i> <i>a</i> <i>b</i> <i>c</i>


 <sub></sub>  <sub></sub> <sub></sub>


    


 <sub></sub>  <sub></sub> <sub></sub>


    


0,25


Vậy <sub>2</sub> <sub>2</sub> <sub>2</sub> 1 1 1 1 1 1 0


1 1 1


<i>a b</i> <i>b c</i> <i>c</i> <i>a</i>



<i>c</i> <i>a</i> <i>b</i> <i>c</i> <i>b</i> <i>c</i> <i>a</i> <i>a</i> <i>c</i> <i>a</i> <i>b</i> <i>b</i> <i>a</i> <i>b</i> <i>c</i>


 <sub></sub>  <sub></sub>  <sub></sub> <sub></sub> <sub></sub> <sub></sub> <sub></sub> <sub></sub> <sub></sub>


         . 0,5


b) Chứng minh rằng nếu <i>a b</i>. 3 thì hai phương trình:


3 2 4


(<i>a</i> <i>a x</i>) <i>a y</i><i>a</i>  1 0 (1); (<i>b</i>3<i>b x b y b</i>)  2   4 1 0 (2) (<i>a,b </i>là các tham số)
khơng có nghiệm nguyên chung.


<b>1,5 </b>


Giả sử (1) và (2) có nghiệm nguyên chung ( ;<i>x y</i>0 0), ta có


(<i>a</i>3<i>a x</i>) 0<i>a y</i>2 0<i>a</i>4 1 0 (3) ; (<i>b</i>3<i>b x</i>) 0<i>b y</i>2 0  <i>b</i>4 1 0 (4)


0,25


Vì <i>a b</i>, 0 ta có<b> </b>


4 3 2


0 0 0


2
2



0 0 0 0


2


4 3 2


0 0 0


2
2


0 0 0 0


2


(3) 1 0


1 1 1 1


0 2 0;


(4) 1 0


1 1 1 1


0 2 0.


<i>a</i> <i>x a</i> <i>y a</i> <i>x a</i>



<i>a</i> <i>a</i> <i>x</i> <i>y</i> <i>a</i> <i>x</i> <i>a</i> <i>y</i>


<i>a</i> <i>a</i> <i>a</i> <i>a</i>


<i>b</i> <i>x b</i> <i>y b</i> <i>x b</i>


<i>b</i> <i>b</i> <i>x</i> <i>y</i> <i>b</i> <i>x</i> <i>b</i> <i>y</i>


<i>b</i> <i>b</i> <i>b</i> <i>b</i>


     


       


<sub></sub>  <sub> </sub>  <sub></sub>   <sub></sub>  <sub></sub>  <sub></sub>  <sub></sub>  


       


     


       


<sub></sub>  <sub> </sub>  <sub></sub>   <sub></sub>  <sub></sub>  <sub></sub>  <sub></sub>  


       


Suy ra<b> </b> 1 2


1 1



;


<i>t</i> <i>a</i> <i>t</i> <i>b</i>


<i>a</i> <i>b</i>


    là hai nghiệm của phương trình bậc hai (ẩn <i>t</i>)
2


0 0 2 0
<i>t</i> <i>x t</i> <i>y</i>   <b>. </b>


0,25


Theo định lí Viet:


0 <sub>0</sub>


0 0


1 1


1 1 1


2 2.


<i>a</i> <i>b</i>


<i>a</i> <i>b</i> <i>x</i> <i>a</i> <i>b</i> <i>x</i>



<i>a</i> <i>b</i> <i>ab</i>


<i>a</i> <i>b</i>


<i>a</i> <i>b</i> <i>y</i> <i>ab</i> <i>y</i>


<i>a</i> <i>b</i> <i>b</i> <i>a</i> <i>ab</i>


 <sub>    </sub>  <sub> </sub>  <sub> </sub>


 <sub></sub>


 <sub></sub> 


 


  


<sub></sub> <sub></sub> <sub></sub> <sub></sub> <sub></sub><sub></sub> <sub></sub>  <sub> </sub> <sub></sub> <sub></sub> <sub></sub>




   




0,25


</div>
<span class='text_page_counter'>(71)</span><div class='page_container' data-page=71>

<b>Sưu tầm và tổng hợp Trịnh Bình </b> TÀI LIỆU TỐN HỌC



<b>Nội dung </b> <b>Điểm </b>


2 2 2


2 2 2


0
0


0


2 2 2 2


0 0 0


9


3 <sub>9</sub>


2


6
16


4


16


16 16



( ) 3 16.


3 3


<i>a</i> <i>b</i> <i>ab</i> <i>x</i>


<i>a</i> <i>b</i> <i>x</i>


<i>a</i> <i>b</i> <i>x</i>


<i>a</i> <i>b</i>


<i>y</i> <i>a</i> <i>b</i> <i>ab y</i> <i>a</i> <i>b</i> <i>y</i>


<i>b</i> <i>a</i>




 <sub>  </sub> <sub></sub> <sub></sub> <sub></sub> <sub></sub>




   


 <sub></sub> <sub></sub>


  


 <sub> </sub> <sub></sub>  <sub></sub> <sub></sub> <sub></sub> <sub></sub> <sub></sub> <sub></sub> <sub></sub>



 


 


Suy ra 2 2


0 0 0 0


9


6 3 16 9 48 160 (4)


16<i>x</i>   <i>y</i>   <i>x</i>  <i>y</i>  . Điều này vơ lí vì VT(4) chia
hết cho 3 nhưng VT(4) không chia hết cho 3.


Vậy nếu <i>a b</i>. 3thì hai phương trình (1), (2) khơng có nghiệm ngun chung.
0,5


<b>Câu 2(3,5 </b><i><b>điểm</b></i><b>) </b>


a) Giải phương trình 2<i>x</i> 3 <i>x</i> 1 1 (1) <b><sub>2,0 </sub></b>


Điều kiện: <i>x</i> 1. 0,5


Ta có:




(1) 2 3 1 1



2 3 2 2 1


2 1 1


<i>x</i> <i>x</i>


<i>x</i> <i>x</i> <i>x</i>


<i>x</i> <i>x</i>


    


     


   


0,5




2


2
1


4( 1) 2 1


1


2 3 0



<i>x</i>


<i>x</i> <i>x</i> <i>x</i>


<i>x</i>


<i>x</i> <i>x</i>


 

 


   



 

 


  




0,5


3
1
<i>x</i>
<i>x</i>





  <sub> </sub>


 .


Vậy phương trình đã cho có hai nghiệm: <i>x</i> 1;<i>x</i>3.


0,5


b) Giải hệ phương trình


3 2 2 2


3 5 4 (1)


3 1 1 (2)


<i>x</i> <i>x y</i> <i>x</i> <i>xy</i> <i>y</i> <i>x</i> <i>y</i>


<i>x</i> <i>y</i> <i>x</i>


      





   



 . <b>1,5 </b>


Điều kiện <i>x</i>0;<i>y</i> 1. Ta có:


2 2 3 2


2
2


(1) ( 5 1) ( 3 4 ) 0


( 1)( 4 ) 0


4
1 0


<i>y</i> <i>x</i> <i>x</i> <i>y</i> <i>x</i> <i>x</i> <i>x</i>


<i>y</i> <i>x</i> <i>y</i> <i>x</i> <i>x</i>


<i>y</i> <i>x</i> <i>x</i>


<i>y</i> <i>x</i>


       


     


  
  <sub>  </sub>





</div>
<span class='text_page_counter'>(72)</span><div class='page_container' data-page=72>

<b>Sưu tầm và tổng hợp Trịnh Bình </b> TÀI LIỆU TỐN HỌC


<b>Nội dung </b> <b>Điểm </b>


Từ (2)3 <i>x</i>  <i>y</i>         1 <i>x</i> 1 1 <i>x</i> 0 <i>y</i> <i>x</i> 1 0 <i>y</i>  <i>x</i> 1 0.
Vậy ta có 2


(1) <i>y</i> <i>x</i> 4<i>x</i>. 0,25


Thay 2
4


<i>y</i> <i>x</i>  <i>x</i> vào (1) ta có 3 <i>x</i> <i>x</i>24<i>x</i>  1 <i>x</i> 1 (3) .
Vì <i>x</i>0 khơng là nghiệm của (3) nên


1 1


(3) <i>x</i> <i>x</i> 4 3


<i>x</i>
<i>x</i>


     


0,25


Đặt <i>t</i> <i>x</i> 1 (<i>t</i> 2) <i>x</i> 1 <i>t</i>2 2


<i>x</i>


<i>x</i>


       . Phương trình trên trở thành:


2 2


2 2


3 <sub>5</sub>


6 3 6 3


2
6 (3 )


<i>t</i>


<i>t</i> <i>t</i> <i>t</i> <i>t</i> <i>t</i>


<i>t</i> <i>t</i>





       <sub></sub>  


  



0,25


Suy ra 2


4


1 25 17


2 1 0 <sub>1</sub>


4 4


4
<i>x</i>


<i>x</i> <i>x</i> <i>x</i>


<i>x</i> <i>x</i>






       


 


. 0,25



Từ đó suy ra hệ phương trình đã cho có hai nghiệm: (4;0);( ;1 15)
4 16




. 0,25


<b>Câu 3</b>.Cho đường tròn ( ; )<i>O R</i> v| điểm <i>A</i> cố định trên ( ; )<i>O R</i> . Gọi <i>M, N </i> là các
giao điểm của hai đường tròn ( ; )<i>O R</i> và ( ; )<i>A R</i> ; <i>H</i> l| điểm thay đổi trên cung
nhỏ <i>MN</i> của đường tròn ( ; )<i>A R</i> . Đường thẳng qua <i>H</i> và vng góc với <i>AH</i>


cắt ( ; )<i>O R</i> tại <i>B, C</i>. Kẻ <i>HI</i>  <i>AB I</i>( <i>AB HK</i>),  <i>AC K</i>( <i>AC</i>).


<b>4,0 </b>


<b>t</b>


<b>N</b>


<b>M</b>


A'
J


K


<b>I</b> O


H



C
B


A


</div>
<span class='text_page_counter'>(73)</span><div class='page_container' data-page=73>

<b>Sưu tầm và tổng hợp Trịnh Bình </b> TÀI LIỆU TOÁN HỌC


<b>Nội dung </b> <b>Điểm </b>


2


. 2


<i>AB AC</i> <i>R</i> .


Ta có <i><sub>AIH</sub></i> <sub>90 ;</sub>0 <i><sub>AKH</sub></i> <sub>90</sub>0


. Vì <i><sub>AIH</sub></i> <i><sub>AKH</sub></i> <sub>180</sub>0


nên tứ giác <i>AIHK</i> nội tiếp. 0,5
Kẻ tiếp tuyến <i>At </i>của đường tròn ( ; )<i>O R</i> tại <i>A</i>.


Ta có:


0
0
90


(1)


90


<i>ACB</i> <i>HAC</i>


<i>ACB</i> <i>AHK</i>


<i>AHK</i> <i>HAC</i>


 <sub></sub> <sub></sub>


 <sub></sub> <sub></sub>




 





0,5


Ta lại có: <i>AHK</i> <i>AIK</i> (do tứ giác <i>AIHK</i> nội tiếp) (2)


<i>BAt</i> <i>ACB</i>(cùng bằng 1


2sđ<i>AB</i>) (3).


Từ (1), (2), (3) suy ra: <i>BAt</i> <i>AIK</i><i>At IK</i>.


0,5



Mặt khác <i>OA</i><i>At</i><i>IK</i> <i>OA</i>. Vậy <i>IK</i> luôn vuông góc với đường thẳng cố


định <i>OA.</i> 0,5


Gọi <i>J </i> l| giao điểm của <i>AO</i> và <i>IK</i>; <i> A’</i> l| điểm đối xứng với <i>A</i> qua <i>O. </i>


Ta có:

0



' ' 90 ; '


<i>ACH</i> <i>AA B AHC</i> <i>ABA</i> <i>ACH</i> <i>AA B</i>


      . 0,25


2


. . ' 2 . 2


'


<i>AC</i> <i>AH</i>


<i>AB AC</i> <i>AH AA</i> <i>R AH</i> <i>R</i>


<i>AA</i> <i>AB</i>


      . 0,25


b) Tìm giá trị lớn nhất của diện tích <i>AIK</i> khi <i>H</i> thay đổi. <b>1,5 </b>



Ta có 2


. .


<i>AK</i> <i>AH</i>


<i>AKH</i> <i>AHC</i> <i>AK AC</i> <i>AH</i>


<i>AH</i> <i>AC</i>


       <sub>0,25 </sub>


Gọi <i>S S</i>, 'lần lượt là diện tích các tam giác <i>ABC</i> và <i>AIK.</i>


Ta có <i>AIK</i> <i>ACB</i> <i>AI</i> <i>AK</i> <i>IK</i> <i>AJ</i>


<i>AC</i> <i>AB</i> <i>BC</i> <i>AH</i>


      , suy ra: 0,25




2 2 <sub>4</sub> <sub>2</sub>


2 2


1
.


' <sub>2</sub> . 1



.


1 <sub>.</sub> <sub>.2</sub> <sub>4</sub> <sub>4</sub>


.
2


<i>AJ IK</i>


<i>S</i> <i>AJ</i> <i>IK</i> <i>AK</i> <i>AK AC</i> <i>AH</i> <i>AH</i>


<i>S</i> <i><sub>AH BC</sub></i> <i>AH BC</i> <i>AB</i> <i>AB AC</i> <i><sub>AH R</sub></i> <i>R</i>


   


  <sub></sub> <sub></sub> <sub></sub> <sub></sub>   


    . 0,5


Suy ra


2


1 1


' . . . .2


4 8 8 8 4



<i>R</i> <i>R</i> <i>R</i>


<i>S</i>  <i>S</i>  <i>AH BC</i>  <i>BC</i>  <i>R</i> . 0,25


Vậy giá trị lớn nhất của tam giác <i>AIK</i> bằng
2
4
<i>R</i>


, đạt khi <i>H</i> <i>O</i>. 0,25


</div>
<span class='text_page_counter'>(74)</span><div class='page_container' data-page=74>

<b>Sưu tầm và tổng hợp Trịnh Bình </b> TÀI LIỆU TỐN HỌC


<b>Nội dung </b> <b>Điểm </b>


của biểu thức 2 2 2 2 2 2


2( ) ( ) 4


<i>P</i> <i>a b b c</i> <i>c a</i>  <i>a</i> <i>b</i> <i>c</i>  <i>abc</i>.
Ta có:


2 2 2 2 2 2


( )( ) ( ) ( ) 3


<i>ab bc</i> <i>ca</i> <i>a</i> <i>b</i> <i>c ab bc</i> <i>ca</i>  <i>a b b c</i> <i>c a</i>  <i>ab</i> <i>bc</i> <i>ca</i>  <i>abc</i>
Suy ra





2 2 2 2


2 2 2 2 2 2


( ) 2( ) 1 2( )


1 2 ( ) ( ) 3


<i>a</i> <i>b</i> <i>c</i> <i>a</i> <i>b</i> <i>c</i> <i>ab bc</i> <i>ca</i> <i>ab bc</i> <i>ca</i>


<i>a b b c</i> <i>c a</i> <i>ab</i> <i>bc</i> <i>ca</i> <i>abc</i>


           


 


  <sub></sub>       <sub></sub>


0,25


Do đó:


2 2 2 2 2 2 2 2 2


2 2 2


2( ) 1 2 ( ) ( ) 3 4


1 2( )



<i>P</i> <i>a b b c</i> <i>c a</i> <i>a b b c</i> <i>c a</i> <i>ab</i> <i>bc</i> <i>ca</i> <i>abc</i> <i>abc</i>


<i>ab</i> <i>bc</i> <i>ca</i> <i>abc</i>


 


     <sub></sub>       <sub></sub>


    


0,25


Khơng mất tính tổng qt có thể giả sử <i>a</i> <i>b</i> <i>c</i>.
Suy ra <i>a a b b c</i>(  )(   ) 0 (<i>a</i>2<i>ab b c</i>)(  ) 0


<i>a b a c ab</i>2  2  2<i>abc</i> 0 <i>ab</i>2<i>ca</i>2<i>a b abc</i>2 


0,25


Do đó


2 2 2 2 2 2 2 2 2


( ) ( ) ( )


<i>ab</i> <i>bc</i> <i>ca</i> <i>abc</i> <i>ab</i> <i>ca</i> <i>bc</i> <i>abc</i> <i>a b</i><i>abc</i> <i>bc</i> <i>abc</i><i>b a</i><i>c</i> 0,25
Với các số dương <i>x, y, z</i> ta ln có:


 

 

2

 

2

2


3 3 3


3 3 3


3 1 3 3 3


3 0


2


<i>x</i>  <i>y</i> <i>x</i> <i>xyz</i>  <i>x</i> <i>y</i>  <i>z</i> <sub></sub> <i>x</i> <i>y</i>  <i>y</i> <i>z</i>  <i>z</i>  <i>x</i> <sub></sub>


 


Suy ra


3
3


3 (*)


3


<i>x</i> <i>y</i> <i>z</i>


<i>x</i>  <i>y</i> <i>z</i> <i>xyz</i> <i>xyz</i>    <sub></sub>


 



Dấu bằng xảy ra khi và chỉ khi <i>x</i> <i>y</i> <i>z</i>.
Áp dụng bất đẳng thức (*) ta có


3


3


2 2 2 4


( ) 4 4 4


2 2 3 3 27


<i>a</i> <i>c</i> <i>a</i> <i>c</i>


<i>b</i>


<i>a</i> <i>c</i> <i>a</i> <i>c</i> <i>a</i> <i>b</i> <i>c</i>


<i>b a</i> <i>c</i> <i>b</i>


 


 <sub></sub> <sub></sub> 


 


   


    



  <sub></sub> <sub></sub> <sub></sub> <sub></sub> <sub></sub>  <sub></sub> <sub></sub> 


   <sub></sub> <sub></sub>  


 


0,25


Suy ra 2 2 2 2 4 19


1 2( ) 1 2 ( ) 1 2.


27 27
<i>P</i>  <i>ab</i> <i>bc</i> <i>ca</i> <i>abc</i>   <i>b a</i><i>c</i>   


Vậy 19


27


<i>MinP</i> .<i> P</i> đạt giá trị nhỏ nhất khi 1
3
<i>a</i>  <i>b</i> <i>c</i> .


</div>
<span class='text_page_counter'>(75)</span><div class='page_container' data-page=75>

Trang 1/3


<b>ĐỀ CHÍNH THỨC</b>


<b>PHÚ THỌ</b> <b>LỚP 9 THCS NĂM HỌC 2017-2018</b>



<b>Mơn: TỐN</b>


<i><b>Thời gian làm bài: 150 phút, khơng kể thời gian giao đề</b></i>
<i>(Đề thi có 03 trang)</i>


<i><b>Thí sinh làm bài (trắc nghiệm khách quan và phần tự luận) ra tờ giấy thi; không làm </b></i>


<i>bài vào đề thi.</i>



<b>A. PHẦN TRẮC NGHIỆM KHÁCH QUAN (8,0 điểm)</b>
<b>Câu 1. </b>Cho phương trình <i><sub>x</sub></i>2<sub></sub><i><sub>mx</sub></i><sub> </sub><sub>4 0.</sub>


Tập hợp các giá trị của tham số <i>m</i> để phương trình có
nghiệm kép là


<b>A. </b>

4; 4 .

<b>B. </b>

 

4 . <b>C. </b>

 

4 . <b>D. </b>

 

16 .


<b>Câu 2. </b>Trên mặt phẳng tọa độ <i>Oxy,</i> góc tạo bởi hai đường thẳng có phương trình <i>y</i> 5 <i>x</i> và


5


<i>y</i> <i>x</i><sub> bằng</sub>
<b>A.</b><sub>70 .</sub>o


<b>B. </b><sub>30 .</sub>o


<b>C. </b><sub>90 .</sub>o


<b>D. </b><sub>45 .</sub>o


<b>Câu 3. </b>Cho





3<sub>10 6 3</sub> <sub>3 1</sub>
.
6 2 5 5


<i>x</i>  


  Giá trị của biểu thức


<sub>3</sub>

2018


4 2


<i>x</i>  <i>x</i>


bằng


<b>A. </b><sub></sub><sub>2</sub>2018<sub>.</sub> <b><sub>B. </sub></b><sub>2</sub>2018<sub>.</sub> <b><sub>C. </sub></b><sub>0.</sub> <b>D. </b>1.


<b>Câu 4.</b> Trên mặt phẳng tọa độ <i>Oxy,</i> cho hai điểm <i>A</i>(2018; 1) và <i>B</i>( 2018;1). Đường trung trực của
đoạn thẳng <i>AB</i> có phương trình là


<b>A.</b> 2018.


<i>x</i>


<i>y</i> 



<b>B. </b> 2018.


<i>x</i>


<i>y</i> <b>C. </b><i>y</i>2018 .<i>x</i> <b>D. </b><i>y</i> 2018 .<i>x</i>


<b>Câu 5. </b>Cho biểu thức <i>P</i> 2<i>x</i> 8<i>x</i> 4 2<i>x</i> 8<i>x</i>4 , khẳng định nào dưới đây đúng ?


<b>A</b>.


2


<i>P</i> 


với mọi


1
2


<i>x</i>


. <b>B. </b><i>P</i> 2 với mọi <i>x</i>1.


<b>C. </b><i>P</i> 2 2<i>x</i>1 với mọi <i>x</i>1. <b><sub>D.</sub></b> <i>P</i> 2 2<i>x</i>1<sub> với mọi </sub>1<sub>2</sub> <i>x</i> 1.


<b>Câu 6. </b>Trong góc phần tư thứ nhất của hệ trục tọa độ <i>Oxy</i> cho điểm <i>M</i>, biết rằng <i>M</i> cách đều trục
tung, trục hoành và đường thẳng <i>y</i> 2 <i>x</i>. Hoành độ của điểm <i>M</i> bằng


<b>A. </b>2 2. <b>B. </b>2 2.



<b>C. </b>


1
.


2 <b>D. </b> 2.


<b>Câu 7.</b> Trong mặt phẳng tọa độ <i>Oxy,</i> khoảng cách từ điểm <i>M</i>

2018; 2018

đến đường thẳng


2


<i>y</i> <i>x</i> <sub> bằng</sub>


<b>A. </b>2. <b><sub>B. </sub></b> <sub>2.</sub> <b>C. </b>4. <b>D. </b>1.


<b>Câu 8.</b> Trong mặt phẳng tọa độ


,


<i>Oxy</i>


cho điểm


2


10 .
3


<i>A</i><sub></sub> <i>m;m -</i> <sub></sub>



  Khi


<i>m</i>


thay đổi thì khẳng định nào
dưới đây đúng ?


<b>A. </b>Điểm <i>A</i> thuộc một đường thẳng cố định. <b>B. </b>Điểm <i>A</i> thuộc một đường tròn cố định.


<b>C. </b>Điểm <i>A</i> thuộc một đoạn thẳng cố định. <b><sub>D.</sub></b><sub> Điểm </sub><i>A</i> <sub>thuộc đường thẳng </sub><i>y x</i> 10<i>.</i>


<b>Câu 9. </b>Cho tam giác <i>ABC</i> có<i>AB</i>3 <i>cm AC</i>, 4 <i>cm</i> và <i>BC</i>5 .<i>cm</i> Kẻ đường cao <i>AH</i>, gọi <i>I K</i>, lần
lượt là tâm đường tròn nội tiếp của tam giác <i>HAB</i> và tam giác <i>HAC</i>. Độ dài của đoạn thẳng <i>KI</i> bằng


</div>
<span class='text_page_counter'>(76)</span><div class='page_container' data-page=76>

Trang 2/3
bằng


<b>A.</b> 2 <i>cm</i> . <b><sub>B.</sub></b> <sub>2</sub><sub></sub> <sub>3 </sub><i><sub>cm</sub></i><sub>.</sub> <b><sub>C. </sub></b> <sub>1</sub><sub></sub> <sub>5 </sub><i><sub>cm</sub></i><sub>.</sub> <b><sub>D. </sub></b> <sub>2</sub><sub></sub> <sub>3 </sub><i><sub>cm</sub></i><sub>.</sub>


<b>Câu 11.</b> Cho hai đường trịn

 

<i>I 3</i>; và

<i>O</i>;6

tiếp xúc ngồi với nhau tại .<i>A</i> Qua <i>A</i> vẽ hai tia vng
góc với nhau cắt hai đường trịn đã cho tại <i>B</i> và .<i>C</i> Diện tích lớn nhất của tam giác <i>ABC</i> bằng


<b>A. </b>6. <b>B. </b>12. <b>C. </b>18. <b>D. </b>20.


<b>Câu 12</b>. Cho hình thoi <i>ABCD</i> có cạnh bằng 1. Gọi <i>x y</i>, lần lượt là bán kính đường trịn ngoại tiếp của
tam giác


<i>ABC</i>


và tam giác


.


<i>ABD</i>


Giá trị của biểu thức 2 2
1 1


<i>x</i>  <i>y</i> <sub> bằng</sub>


<b>A. </b>4. <b><sub>B. </sub></b> 2.


<b>C. </b>


3
.


2 <b>D. </b>


1
.
4


<b>Câu 13.</b> Cho tứ giác <i>ABCD</i> nội tiếp đường trịn

<i>O R</i>;

đường kính <i>AC</i> và dây cung <i>BD R</i> 2.
Gọi , , , <i>x y z t</i> lần lượt là khoảng cách từ điểm <i>O</i> tới <i>AB CD BC DA</i>, , , . Giá trị của biểu thức <i>xy zt</i>
bằng


<b>A. </b><sub>2 2 .</sub><i><sub>R</sub></i>2 <b><sub>B. </sub></b> <sub>2 .</sub><i><sub>R</sub></i>2


<b>C. </b>



2


2
.


2 <i>R</i> <b>D. </b>


2


2
.


4 <i>R</i>


<b>Câu 14.</b> Cho tam giác <i>ABC</i> ngoại tiếp đường tròn (<i>I</i>; 2 <i>cm</i>) và nội tiếp đường tròn

<i>O</i>;6 <i>cm</i>

. Tổng
khoảng cách từ điểm <i>O</i> tới các cạnh của tam giác <i>ABC</i> bằng


<b>A. </b>8 .<i>cm</i> <b>B. </b>12 .<i>cm</i> <b>C. </b>16 .<i>cm</i> <b>D. </b>32 .<i>cm</i>


<b>Câu 15. </b>Nếu một tam giác có độ dài các đường cao bằng 12,15, 20 thì bán kính đường trịn nội tiếp
tam giác đó bằng


<b>A. </b>5. <b>B. </b>4. <b>C. </b>3. <b>D.</b>6 .


<b>Câu 16. </b>Trên một khu đất rộng, người ta muốn rào một mảnh đất nhỏ hình
chữ nhật để trồng rau an toàn, vật liệu cho trước là 60<i>m</i> lưới để rào. Trên
khu đất đó người ta tận dụng một bờ rào <i>AB</i> có sẵn (<i>tham khảo hình vẽ </i>
<i>bên</i>) để làm một cạnh hàng rào. Hỏi mảnh đất để trồng rau an tồn có diện
tích lớn nhất bằng bao nhiêu ?



<b>A.</b><sub>400 </sub><i><sub>m</sub></i>2<sub>.</sub> <b><sub>B.</sub></b><sub>450 </sub><i><sub>m</sub></i>2<sub>.</sub> <b><sub>C. </sub></b><sub>225 </sub><i><sub>m</sub></i>2<sub>.</sub> <b><sub>D.</sub></b><sub>550 </sub><i><sub>m</sub></i>2<sub>.</sub>


<b>B. PHẦN TỰ LUẬN (12,0 điểm)</b>
<b>Câu 1 (3,0 điểm).</b>


a) Cho <i>a b c</i>2

<i>b c a</i>2

2018 với <i>a b c</i>, , đôi một khác nhau và khác không. Tính giá trị
của biểu thức <i>c a b</i>2

.


b) Tìm tất cả các số nguyên dương <i>a b c</i>, , thỏa mãn <i>a b c</i>  91 và <i><sub>b</sub></i>2 <sub></sub><i><sub>ca</sub></i><sub>.</sub>


<b>Câu 2 (3,5 điểm).</b>


a) Giải phương trình <i>x</i>22<i>x</i> <i>x</i>22<i>x</i> 2 0.


b) Hai vị trí <i>A</i> và <i>B</i> cách nhau 615 <i>m</i> và cùng nằm về
một phía bờ sơng. Khoảng cách từ <i>A B</i>, đến bờ sông lần lượt là
118 <i>m</i> và 487 <i>m</i>(<i>tham khảo hình vẽ bên</i>). Một người đi từ <i>A</i>


đến bờ sông để lấy nước mang về .<i>B</i> Đoạn đường ngắn nhất
mà người đó có thể đi được bằng bao nhiêu mét (<i>làm tròn đến </i>
<i>đơn vị mét</i>).


</div>
<span class='text_page_counter'>(77)</span><div class='page_container' data-page=77>

Trang 3/3


( ,<i>B C</i><sub> là các tiếp điểm). Một cát tuyến thay đổi qua </sub><i>A</i><sub> cắt </sub>

 

<i>O</i> <sub>tại </sub><i>D</i><sub> và </sub><i>E AD</i> ( <i>AE</i>).<sub> Tiếp tuyến </sub>


của

 

<i>O</i> tại <i>D</i> cắt đường tròn ngoại tiếp tứ giác <i>ABOC</i> tại các điểm <i>M</i> và .<i>N</i>


a) Gọi <i>I</i> là trung điểm của đoạn thẳng <i>AD</i>. Chứng minh rằng bốn điểm <i>M E N I</i>, , , cùng thuộc
một đường tròn

 

<i>T</i> .


b) Chứng minh rằng hai đường tròn

 

<i>O</i> và

 

<i>T</i> tiếp xúc nhau.
c) Chứng minh rằng đường thẳng <i>IT</i> luôn đi qua một điểm cố định.


<b>Câu 4 (1,5 điểm).</b>


Chứng minh rằng

2 2 2


3 3 3


9


<i>a b</i> <i>b c</i> <i>c a</i>


<i>a b c</i>


<i>a</i> <i>ab b</i> <i>bc c</i> <i>ca</i>


  


 


  <sub></sub>   <sub></sub>


  


  với


, ,



<i>a b c</i>


là độ dài ba cạnh của
một tam giác.


--- HẾT


<i><b>---Họ và tên thí sinh:………Số báo danh:………..</b></i>


<i><b>Cán bộ coi thi khơng giải thích gì thêm.</b></i>


<b>SỞ GIÁO DỤC VÀ ĐÀO TẠO</b>


<b>HƯỚNG DẪN CHẤM</b>
<b>PHÚ THỌ</b>


<b>KỲ THI CHỌN HỌC SINH GIỎI CẤP TỈNH</b>
<b>LỚP 9 THCS NĂM HỌC 2017-2018</b>


<b>Mơn: TỐN</b>


</div>
<span class='text_page_counter'>(78)</span><div class='page_container' data-page=78>

Trang 4/3


<b>A</b> <b>C</b> <b>B</b> <b>C</b> <b>B,D</b> <b>A,B</b> <b>B</b> <b>A</b>


<b>Câu</b> <b>9</b> <b>10</b> <b>11</b> <b>12</b> <b>13</b> <b>14</b> <b>15</b> <b>16</b>


<b>D</b> <b>B</b> <b>C</b> <b>A</b> <b>C</b> <b>A</b> <b>A</b> <b>B</b>


<b>B. PHẦN TỰ LUẬN (12 điểm)</b>


<b>Câu 1</b>


<b>3,0 điểm</b>


a) Cho <i>a b c</i>2

<i>b c a</i>2

2018 với <i>a b c</i>, , đôi một khác nhau và khác khơng.
Tính giá trị của biểu thức <i>c a b</i>2

.


b) Tìm tất cả các số nguyên dương <i>a b c</i>, , thỏa mãn <i>a b c</i>  91 và <i><sub>b</sub></i>2 <sub></sub><i><sub>ca</sub></i><sub>.</sub> <b>Điểm</b>


<b>a)</b>
<b>1.5 điểm</b>


Ta có



2 2 <i>a</i> <i>b</i> <i>a b</i> 1<sub>.</sub>


<i>a b c</i> <i>b c a</i>


<i>bc ab</i> <i>ab ca</i> <i>c b a</i> <i>c</i>




       


   <b>0,25</b>


Suy ra <i>ab bc ca</i>   0 <i>bc</i> <i>a b c</i>

 <i>abc a b c</i> 2

2018.(1) <b>0,5</b>


2




0 .(2)


<i>ab bc ca</i>   <i>ab</i> <i>c a b</i>  <i>abc c a b</i>  <b><sub>0,5</sub></b>


Từ (1) và (2) ta được <i>c a b</i>2

2018. <b>0,25</b>


<b>b)</b>
<b>1,5 điểm</b>


Đặt <i>b qa c q a q</i> ;  2

1

thì ta được a 1

 <i>q q</i>2

91 13.7. <b>0,25</b>


Trường hợp 1: Nếu <i>q</i> là số tự nhiên thì ta được
2


1 1


1; 9; 81.
9


1 91


<i>a</i> <i>a</i>


<i>a</i> <i>b</i> <i>c</i>


<i>q</i>
<i>q q</i>


 



 


    


 <sub> </sub> <sub></sub> <sub> </sub>





<b>0,25</b>


2


7 7


7; 21; 63.
3


1 13


<i>a</i> <i>a</i>


<i>a</i> <i>b</i> <i>c</i>


<i>q</i>
<i>q q</i>


 


 



    


 <sub> </sub>


   <sub></sub>


 <b>0,25</b>


2


13 13


13; 26; 52.
2


1 7


<i>a</i> <i>a</i>


<i>a</i> <i>b</i> <i>c</i>


<i>q</i>
<i>q q</i>


 


 


    



 <sub> </sub>


   <sub></sub>


 <b>0,25</b>


Trường hợp 2: Nếu


<i>q</i>


là số hữu tỷ thì giả sử

3; 2 .



<i>x</i>


<i>q</i> <i>x</i> <i>y</i>


<i>y</i>


  


Khi đó a 1

 <i>q q</i>2

91<i>a x</i>

2<i>xy y</i> 2

91<i>y</i>2

<i>x</i>2<i>xy y</i> 2 19



<b>0,25</b>


Ta có


2


2 2 2



2 2 91 6; 5.


<i>ax</i> <i>a</i>


<i>c</i> <i>a ty</i> <i>x</i> <i>xy y</i> <i>x</i> <i>y</i>


<i>y</i> <i>y</i>


            


và <i>a</i>25;<i>b</i>30;<i>c</i>36.


Vậy có 8 bộ số

<i>a b c</i>; ;

thỏa mãn

1;9;81 , 81;9;1 , 7; 21;63 , 63; 21;7 ;...

 

 

 



<b>0,25</b>


<b>Câu 2.</b>
<b>3,5 điểm</b>


a) Giải phương trình <i><sub>x</sub></i>2<sub></sub><sub>2</sub><i><sub>x</sub></i><sub></sub> <i><sub>x</sub></i>2<sub></sub><sub>2</sub><i><sub>x</sub></i><sub> </sub><sub>2 0.</sub>


b) Hai vị trí <i>A</i> và <i>B</i> cách nhau 615<i>m</i> và cùng nằm về một phía bờ sông. Khoảng
cách từ<i>A B</i>, đến bờ sông lần lượt là 118<i>m</i> và 487<i>m</i>(<i>tham khảo hình vẽ dưới đây</i>).
Một người đi từ <i>A</i> đến bờ sông để lấy nước mang về <i>B</i>. Đoạn đường ngắn nhất mà
người đó có thể đi được bằng bao nhiêu mét (làm tròn đến 1 chữ số thập phân).


<b>a)</b>





2 <sub>2</sub> 2 <sub>2</sub> <sub>2 0</sub> 2 <sub>2</sub> <sub>2</sub> 2 <sub>2</sub> <sub>2 2 0.</sub>


<i>x</i>  <i>x</i> <i>x</i>  <i>x</i>   <i>x</i>  <i>x</i>  <i>x</i>  <i>x</i>   <b>0,25</b>


2
2


2 2 1( )


2 2 2


<i>x</i> <i>x</i> <i>L</i>


<i>x</i> <i>x</i>


 <sub></sub> <sub>  </sub>





 <sub></sub> <sub> </sub>




<b>0,25</b>
<b>0,25</b>


2 <sub>2</sub> <sub>2 4</sub> 2 <sub>2</sub> <sub>2 0</sub>



<i>x</i> <i>x</i> <i>x</i> <i>x</i>


</div>
<span class='text_page_counter'>(79)</span><div class='page_container' data-page=79>

Trang 5/3


.


1 3


<i>x</i>
 


  


 <b>0,25</b>


<b>b)</b>
<b>2,0 điểm</b>




Gọi <i>C D</i>, lần lượt là hình chiếu của <i>A B</i>, lên bờ sông. Đặt <i>CE</i><i>x</i>

0 <i>x</i> 492



Ta có



2
2


615 487 118 492.


<i>CD</i>    <b>0,25</b>



Quãng đường di chuyển của người đó bằng <i>AE EB</i>


2


2 <sub>118</sub>2 <sub>492</sub> <sub>487</sub>2


<i>x</i> <i>x</i>


    


<b>0,25</b>


Ta có với mọi <i>a b c d</i>, , , thì

 



2 2


2 2 2 2 <sub> (1).</sub>


<i>a</i> <i>b</i>  <i>c</i> <i>d</i>  <i>a c</i>  <i>b d</i> <b>0,25</b>


Thật vậy

 



 



2 2


2 2 2 2 <sub>2</sub> 2 2 2 2


1 <i>a</i> <i>b</i>  <i>c</i> <i>d</i>  <i>a</i> <i>b</i> <i>c</i> <i>d</i>  <i>a c</i>  <i>b d</i>


<i><sub>a</sub></i>2 <i><sub>b</sub></i>2



<i><sub>c</sub></i>2 <i><sub>d</sub></i>2

<i><sub>ac bd</sub></i><sub> (2)</sub>


    


Nếu <i>ac bd</i> 0 thì (2) ln đúng. Nếu <i>ac bd</i> 0bình phương hai vế ta được <b>0,25</b>


(2) trở thành

<i>ad bc</i>

20.Dấu đẳng thức sảy ra khi <i>ad bc</i> . <b>0,25</b>
Áp dụng (1) thì

 



2 2


492 487 118 608089 779,8


<i>AE EB</i>  <i>x</i> <i>x</i>     <i>m</i> <b>0,25</b>


Dấu đẳng thức xảy ra khi 487<i>x</i>118 492

<i>x</i>

 <i>x</i> 96<i>m</i> <b>0,25</b>
Vậy quãng đường nhỏ nhất là 780 <i>m</i> <b>0,25</b>
<b>Câu 3</b>


<b>4,0 điểm</b> Cho đường tròn

 

<i>O</i> và điểm


<i>A</i><sub> nằm ngoài </sub>

 

<i>O</i> .<sub>Qua </sub> <i>A</i><sub> kẻ hai tiếp tuyến </sub><i>AB AC</i>,


với

 

<i>O</i> (<i>B C</i>, là các tiếp điểm), một cát tuyến thay đổi qua <i>A</i> cắt

 

<i>O</i> tại <i>D</i> và


( ).


<i>E AD</i><i>AE</i> <sub> Tiếp tuyến của </sub>

<sub> </sub>

<i>O</i> <sub>tại </sub><i>D</i><sub> cắt đường tròn ngoại tiếp tứ giác </sub><i>ABOC</i><sub> tại </sub>


các điểm <i>M</i> và .<i>N</i>



a) Gọi <i>I</i> là trung điểm của đoạn thẳng <i>AD</i>, chứng minh rằng bốn điểm


, , ,


<i>M E N I</i><sub> cùng thuộc một đường tròn </sub>

 

<i><sub>T</sub></i> <sub>.</sub>


</div>
<span class='text_page_counter'>(80)</span><div class='page_container' data-page=80>

Trang 6/3


Ta có 蹷<i><sub>ABO ACO</sub></i><sub></sub>蹷 <sub></sub>180<i>o</i><sub> nên tứ giác </sub><i>ABON</i><sub> nội tiếp </sub>


Gọi <i>J</i> là giao điểm của <i>AD</i> với đường tròn

<i>ABOC</i>

.Suy ra <i>DMA</i> đồng dạng


<i>DNJ</i>




<b>0,25</b>


Suy ra <i>DM DN</i>. <i>DA DJ</i>. <b>0,25</b>




1


2 ; .


2


<i>DA</i> <i>DI DJ</i>  <i>DE</i> <b>0,25</b>



Nên <i>DM DN</i>. <i>DI DE</i>. <i>DMI</i> đồng dạng <i>DEN</i> <b>0,25</b>


Vậy tứ giác <i>MINE</i> nội tiếp hay có đpcm. <b>0,25</b>
<b>b)</b>


<b>1,5 điểm</b> Dễ thấy khi


<i>MN</i> <i>OA</i><sub>thì </sub>

<sub> </sub>

<i>O</i> <sub> và </sub>

 

<i>T</i> <sub> tiếp xúc nhau tại .</sub><i>E</i> <b>0,25</b>


Khi <i>MN</i> khơng vng góc <i>OA</i>. Gọi <i>K</i> là giao điểm của <i>MN</i> với tiếp tuyến của


 

<i>O</i> <sub> tại .</sub><i>E</i>


<b>0,25</b>


Ta có <i>O J K</i>, , thẳng hàng <b>0,25</b>


Trong tam giác <i>OEK KJ KO KE</i>: .  2 (1) ( Định lý hình chiếu) <b>0,25</b>
Trên đường trịn

<i>ABOC</i>

ta có <i>KJ KO KN KM</i>.  . (2). <b>0,25</b>
Từ (1) và (2) suy ra <i>KE</i>2 <i>KN KM</i>. nên <i>KE</i> tiếp xúc

 

<i>T</i> <b>0,25</b>


<b>c)</b>


<b>1,0 điểm</b> Ta có


蹷 蹷 蹷


<i>OED ODE TIE</i>  <b>0,25</b>



Nên <i>IT OD</i>‖ .<sub> Gọi W</sub><i>OA IT</i> . <b>0,25</b>


Vì <i>I</i> là trung điểm của <i>AD</i> nên W là trung điểm <i>OA</i> (đpcm) <b>0,25</b>


Khi <i>MN</i> <i>OA</i> thì W IT. <b>0,25</b>


Chứng minh rằng

2 2 2


3 3 3


9


<i>a b</i> <i>b c</i> <i>c a</i>


<i>a b c</i>


<i>a</i> <i>ab b</i> <i>bc c</i> <i>ca</i>


  


 


  <sub></sub>   <sub></sub>


  


  với


, ,



<i>a b c</i>


là độ dài ba
cạnh của một tam giác.


Giả sử <i>a b c t</i>   và đặt <i>a tx b ty c tz</i> ;  ;     <i>x y z</i> 1. <b>0,25</b>


Ta chứng minh


 

<i>t x y</i><sub>2</sub>

<sub></sub>

3<sub>2</sub>

<sub></sub>

<sub>2</sub><i>t</i>

<sub></sub>

3<i>y z</i><sub>2</sub>

<sub></sub>

<i>t z x</i><sub>2</sub>

<sub></sub>

3<sub>2</sub>

<sub></sub>

9


<i>t x y z</i>


<i>t x</i> <i>xy</i> <i>t y</i> <i>yz</i> <i>t z</i> <i>zx</i>


 <sub></sub> <sub></sub> <sub></sub> 


 


    


  


 


 


2 2 2


3 3 3



9.


<i>x y</i> <i>y z</i> <i>z x</i>


<i>x</i> <i>xy</i> <i>y</i> <i>yz</i> <i>z</i> <i>zx</i>


  


   


  


</div>
<span class='text_page_counter'>(81)</span><div class='page_container' data-page=81>

Trang 7/3


<b>1,5 điểm</b>  <i><sub>x x y</sub></i>

<sub></sub>

<sub></sub>

<sub></sub>

 <i><sub>y y z</sub></i>

<sub></sub>

<sub></sub>

<sub></sub>

 <i><sub>z z x</sub></i>

<sub></sub>

<sub></sub>

<sub></sub>

 9 <sub>1</sub><sub></sub><i><sub>z</sub></i> <i><sub>x</sub></i> <sub>1</sub><sub></sub><i><sub>x</sub></i> <i><sub>y</sub></i> <sub>1</sub><sub></sub><i><sub>y</sub></i> <i><sub>z</sub></i> 9


2 2 2


5 1 5 1 5 1
9


<i>x</i> <i>y</i> <i>y</i>


<i>x x</i> <i>y y</i> <i>z z</i>


  


   



  


<b>0,25</b>




, ,


<i>a b c</i>


là ba cạnh của một tam giác nên


1


, , 0; .


2


<i>a b c</i>  <i>x y z</i><sub></sub> <sub></sub>


  <b>0,25</b>


Ta có:


 

2



2


5 1



18 3 3 1 2 1 0


<i>x</i>


<i>x</i> <i>x</i> <i>x</i>


<i>x x</i>


 <sub></sub> <sub> </sub> <sub></sub> <sub> </sub>


 <sub> đúng </sub>


1
0;


2


<i>x</i>  


 <sub></sub> <sub></sub>


 

2



2
5 1


18 3 3 1 2 1 0


<i>y</i>



<i>y</i> <i>y</i> <i>y</i>


<i>y y</i>


 <sub></sub> <sub> </sub> <sub></sub> <sub> </sub>


 <sub> đúng </sub>


1
0;


2


<i>y</i>  


 <sub></sub> <sub></sub>


 

2



2


5 1


18 3 3 1 2 1 0


<i>z</i>


<i>z</i> <i>z</i> <i>z</i>


<i>z z</i>



 <sub></sub> <sub> </sub> <sub></sub> <sub> </sub>


 <sub> đúng </sub>


1
0;


2


<i>z</i>  


 <sub></sub> <sub></sub>


<b>0,25</b>


Suy ra 2 2 2



5 1 5 1 5 1


18 9


<i>x</i> <i>y</i> <i>y</i>


<i>x y z</i>


<i>x x</i> <i>y y</i> <i>z z</i>


  



      


   2 2 2


5 1 5 1 5 1
9


<i>x</i> <i>y</i> <i>y</i>


<i>x x</i> <i>y y</i> <i>z z</i>


  


   


  


<b>0,25</b>


</div>
<span class='text_page_counter'>(82)</span><div class='page_container' data-page=82>

<b>Hết---ĐỀ CHÍNH THỨC </b>


<b>Số báo danh </b>
...


<b>Mơn: Tốn </b>


Thời gian: <b>150</b> phút (<i>khơng kể thời gian giao đề</i>)
Ngày 15 tháng 03 năm 2019


<i>(Đề có 01 trang, gồm 05 câu) </i>



Tên : Trương Quang An .Địa chỉ : Xã Nghĩa Thắng ,Huyện Tư Nghĩa ,Tỉnh Quảng


Ngãi.Điện thoại : 01208127776.Nguồn gốc : SƯU TẦM



<b>A.TRẮC NGHIỆM (8điểm) </b>



<b>Câu 1.</b>

Có tất cả bao nhiêu số nguyên dương n sao cho



2
1024


15


<i>n</i>


<sub>là số tự nhiên. </sub>


A.1 B.4 C.3 D.5



<b>Câu 2.</b>

Cho hình thang ABCD có hai cạnh đáy AB,CD sao



cho

<i>AB</i>4, D<i>C</i> 9,<i>DAB</i><i>DBC</i>

. Độ dài đường chéo BD bằng .


A.6 B.7 C.8 D.10



<b>Câu 3.</b>

Trong mặt phẳng tọa độ Oxy đường thẳng đi qua điểm M(2;5) và song song với



đường thẳng y=2x có phương trình là



A.y = 2x+1 B.y = 2x-1 C.y =- 2x+9 D.y = -2x+1



<b>Câu 4 .</b>

Trong mặt phẳng tọa độ Oxy ,cho hai điểm A(2;3) và B(6;1).Độ dài đường cao




hạ từ đỉnh O của tam giác OAB bằng


A.

5


2

B.


5 2


2

C.

5 2

D.


2
2


<b>Câu 5.</b>

Trong mặt phẳng tọa độ Oxy, cho hai điểm A(2;3), B(2;-2), C(-2;-2), D(-3;3).



Diện tích tứ giác ABCD bằng


A.

15


2

B.


15 2


2

C.

15

D.30



<b>Câu 6.</b>

Cho hàm số

2 2 2


ax ; x ; x


<i>y</i> <i>y</i><i>b</i> <i>y</i><i>c</i>

có đồ thị như sau .Khẳng định nào đúng.


A.

<i>b</i> <i>a</i> <i>c</i>

B.

<i>a</i> <i>b</i> <i>c</i>

C.

<i>c</i> <i>b</i> <i>a</i>

D.

<i>c</i> <i>a</i> <i>b</i>


<b>Câu 7.</b>

Cho bốn điểm A,B,C,D nằm trên đồ thị hàm số

2



<i>y</i><i>x</i>

sao cho ABCD là một tứ


giác lồi nội tiếp đường trịn đường kính AC.Gọi

<i>M x y</i>( ;1 1);<i>N x y</i>( ;2 2)

lần lượt là trung



điểm của AC,BD giá trị

<i>y</i>1<i>y</i>2

bằng



A.

0

B.

2


2

C.

1

D.

2


<b>Câu 8.</b>

Cho tam giác ABC vng tại A có AB=3 ,AC=4 và phân giác AD.Gía trị



DC-DB bằng


A.

1


7

B.


3


7

C.


4


7

D.


5
7


<b>Câu 9.</b>

Gọi S là tập nghiệm của phương trình số nghiệm của phương trình



2


1 ... 2019 2019x 2020



<i>x</i>     <i>x</i> <i>x</i> <i>x</i>  



</div>
<span class='text_page_counter'>(83)</span><div class='page_container' data-page=83>

Giá trị

2
1


<i>a</i> <i>a</i>


  

bằng



A.

3 1


2




B.

3 1

C.

6 2
2




D.

3 1


<b>Câu 11.</b>

Cho tam giác ABC vng tại A có đường cao AH,trung tuyến AM.Biết



24
25


<i>AH</i>


<i>AM</i> 

và cạnh huyền BC=35.Độ dài bán kính đường trịn nội tiếp tam giác ABC



bằng



A. 3,5

5

B.

7

C.

8,75

D. 14



<b>Câu 12.</b>

Cho tam giác ABC vng tại A có

<i>AB</i> 5,

đường cao AH=2.Kẻ HK vng



góc AC (K thuộc AC).Độ dài CK bằng


A.

3 5


2

B.


8 5


2

C.


5 5


2

D.


16 5


5


<b>Câu 13.</b>

Một học sinh đứng ở mặt đất cách tháp ăng-ten 100m.Biết rằng học sinh đó



nhìn thấy đỉnh tháp ở góc

0


19

so với đường nằm ngang ,khoảng cách từ mắt đến mặt đất


bằng 1,5m. Chiều cao của tháp (làm tròn đến đơn vị mét) bằng



A.

34

B.35

C.

36 D. 38



<b>Câu 14.</b>

Tỉ số giữa bán kính đường trịn nội tiếp và bán kính đường trịn ngoại tiếp của




một tam giác đều bằng


A.

1


4

B.


1


3

C.


1


2

D.


2
3


<b>Câu 15.</b>

Cho tam giác ABC vuông tại A ,đường tròn nội tiếp tam giác ABC tiế xúc với



BC tại D.Biết BD=2DC=10.Diện tích tam giác ABC bằng


A.

25

B.50

C.

50 2

D. 100



<b>Câu 16.</b>

Có tất cả bao nhiêu cách xếp bạn An, Bình ,Cường ,Thắng,Việt ngồi thành một



hàng ngang sao cho hai bạn Thắng và Việt không ngồi cạnh nhau.


A.

48

B.72

C.

96 D. 118



<b>B.TỰ LUẬN (12điểm) </b>


<b>C 1 (3 0 điểm : </b>



a)Chứng minh rằng trong 5 số nguyên dương đôi 1 phân biệt ln tồn tại 4 số có t ng


là hợp số.




b)Bạn Thắng lần lượt chia số 2018 cho 1, 2, 3, 4, , 2018 rồi viết ra 2018 số dư tương


ứng sau đó bạn Việt chia số 2019 cho 1, 2, 3, 4, , 2019 rồi viết ra 2019 số dư tương


ứng. H i ai có t ng số dư lớn hơn và lớn hơn bao nhiêu.



<b>C 2 (3 0 điểm : </b>



a) Giải hệ phương trình:





3 3


2 1 0


3 32 0


<i>x</i> <i>y</i> <i>xy</i>


<i>x</i> <i>y</i> <i>x</i> <i>y</i>


    





    






b) Giải phương trình






2
1


10 11 1


<i>x</i> <i>x</i> <i>x</i>


<i>x</i>


 <sub></sub>  <sub></sub> <sub></sub> <sub></sub>


 


 

.



</div>
<span class='text_page_counter'>(84)</span><div class='page_container' data-page=84>

(O) tại M, N.



a)Chứng minh rằng MN song song EF



b)Chứng minh rằng MC tiếp xúc với đường tròn ngoại tiếp tam giác KFC


c)Chứng minh EF luôn đi qua điểm cố định khi D chạy trên BC



<b>Câu 4</b>

<b>(1 0 điểm </b>

Cho các số thực x

1

, x

2

,..., x

n

 

0;1

.



Chứng minh rằng

2

<sub>2</sub> <sub>2</sub> <sub>2</sub> <sub>2</sub>



1 2 3 1 2 3


1 <i>x</i> <i>x</i> <i>x</i>  ... <i>x<sub>n</sub></i> 4 <i>x</i> <i>x</i> <i>x</i> ....<i>x<sub>n</sub></i>




--- Hết ---



<i>Thí sinh khơng sử dụng tài liệu. Giám thị khơng giải thích gì thêm. </i>



<b>ĐÁP ÁN </b>


<b>C 1( 3 điểm </b>



a)Chứng minh rằng trong 5 số nguyên dương đôi 1 phân biệt luôn tồn tại 4 số có t ng


là hợp số.



b)Bạn Thắng lần lượt chia số 2018 cho 1,2,3,4, ,2018 rồi viết ra 2018 số dư tương


ứng sau đó bạn Việt chia số 2019 cho 1,2,3,4, ,2019 rồi viết ra 2019 số dư tương ứng


. H i ai có t ng số dư lớn hơn và lớn hơn bao nhiêu.



<b>Giải </b>


<b>a)</b>

Áp dụng quy tắc chẵn –lẻ. Xét các trường hợp:



Ta có a, b, c c ng chẵn nên đương nhiên chọn bất kỳ cặp nào cũng có t ng và cả hiệu


của chúng là số chia hết cho 2.



Ta có a, b, c c ng lẻ nên đương nhiên chọn bất kỳ cặp nào cũng có


t ng và cả hiệu của chúng là số chia hết cho 2.



Ta có a, b, c có 1 cặp là số lẻ nên hiệu và t ng của 2 số lẻ chia hết cho 2


a, b, c có 1 cặp là số chẵn nên hiệu và t ng của 2 số chẵn chia hết cho 2.



Hai trường hợp đầu có 3 cặp số th a mãn đầu bài. Hai trường hợp cuối có 1 cặp số


th a mãn đầu bài. Vậy có ít nhât 1 cặp số mà t ng và hiệu của chúng chia hết cho 2


nên là hợp số.




</div>
<span class='text_page_counter'>(85)</span><div class='page_container' data-page=85>

cũng có ít nhất 1 cặp số có số dư là (1+4) hoặc (2+3) nên t ng 1 cặp số đó chia hết cho


5. Với nhóm số có số dư (1,2,3,4) nên suy ra 2 cặp có t ng chia hết cho 5



*Cả 4 số có số dư tr ng nhau nên 6 cặp từng đơi một có hiệu bằng 0 nên chia hết cho


5.



*Cả 2 cặp có số dư tr ng nhau nên hiệu của 2 cặp đó bằng 0 nên chia hết cho 5


*Cả 1 cặp có số dư tr ng nhau nên hiệu của 1 cặp đó bằng 0 nên chia hết cho 5


Vậy ít nhất cũng chọn ra 1 cặp số mà t ng hoặc hiệu của chúng chia hết cho 5.Hay


trong 5 số nguyên dương đôi 1 phân biệt luôn tồn tại 4 số có t ng là hợp số.



b)Gọi T là là t ng các số dư của thắng, V là t ng các số dư của Việt.Gọi

<i>t</i><sub>1</sub>;...<i>t</i><sub>2018</sub>

là số



dư chia 2018 cho 1,2,...,2018, gọi

<i>v</i><sub>1</sub>;.., v<sub>2019</sub>

là số dư chia 2019 cho 1,2, ,2019.Ta thấy



rằng

<i>T</i>   <i>t</i><sub>1</sub> <i>t</i><sub>2</sub> ... <i>t</i><sub>2018</sub>;<i>V</i>   <i>v</i><sub>1</sub> <i>v</i><sub>2</sub> ... <i>v</i><sub>2019</sub>

với

<i>i</i>1.2.3...,2018

.Nếu


    <sub>1</sub>


2019 <i>i</i> <i>v</i> 0 <i>t<sub>i</sub></i> <i>i</i> 1

. Nếu

<i>v</i><sub>1</sub>    <i>i</i> 1 <i>v</i><sub>1</sub> <i>t<sub>i</sub></i> 1


 



 <i>V</i> <i>t</i><sub>1</sub> 1 <i>t</i><sub>2</sub>  1 ... <i>t</i><sub>2018</sub> 1 <i>S</i>(2019 <i>T</i> 2018<i>S</i>(2019)

. Trong đó S(2019)


là t ng các ước không vượt quá 2018 của 2019 . Ta có 2019=1.3.773 . Suy ra



S(2019)=677 nên ta có V=T+2018-677=T+1341. Suy ra V > T và V-T=1341.



<b>C 2(3 5 điểm </b>




a) Giải hệ phương trình:





3 3


2 1 0


3 32 0


<i>x</i> <i>y</i> <i>xy</i>


<i>x</i> <i>y</i> <i>x</i> <i>y</i>


    





    






b)Giải phương trình





2
1


10 11 1



<i>x</i> <i>x</i> <i>x</i>


<i>x</i>


 <sub></sub>  <sub></sub> <sub></sub> <sub></sub>


 


 



<b>H n đ n </b>



a)Cách 1.Ta có







3

 



3 3


2 1 0


2 1 0


3 32 0 3 3 32 0


<i>x</i> <i>y</i> <i>xy</i>



<i>x</i> <i>y</i> <i>xy</i>


<i>x</i> <i>y</i> <i>x</i> <i>y</i> <i>x</i> <i>y</i> <i>xy x</i> <i>y</i> <i>x</i> <i>y</i>


    


    


 <sub></sub>


 


           


 


 


.Ta



đặt

   2 


; ( 4 )


<i>x</i> <i>y</i> <i>s xy</i> <i>p s</i> <i>p</i>

. Khi đó hệ tương đương với

<sub></sub>   


   


 3



2 1 0


3 3 32 0


<i>s</i> <i>p</i>


<i>s</i> <i>ps</i> <i>s</i>

.Giải hệ



trên ta có nghiệm.



Cách 2.Ta có







3

 



3 3


2 1 0


2 1 0


3 32 0 3 3 32 0


<i>x</i> <i>y</i> <i>xy</i>


<i>x</i> <i>y</i> <i>xy</i>



<i>x</i> <i>y</i> <i>x</i> <i>y</i> <i>x</i> <i>y</i> <i>xy x</i> <i>y</i> <i>x</i> <i>y</i>


    


    


 <sub></sub>


 


           


 


 










        


 


<sub></sub> <sub></sub>



           


 


 3  3


2 1 0 2 1


3 ( 1) 32 0 3 ( 1) 32 0


<i>x</i> <i>y</i> <i>xy</i> <i>x</i> <i>y</i> <i>xy</i>


<i>x</i> <i>y</i> <i>x</i> <i>y xy</i> <i>x</i> <i>y</i> <i>x</i> <i>y xy</i>






   



 


    


 3 2


2 1


6 32 0



<i>x</i> <i>y</i> <i>xy</i>


<i>x</i> <i>y</i> <i>x</i> <i>y</i>


</div>
<span class='text_page_counter'>(86)</span><div class='page_container' data-page=86>

 




   


   <sub>    </sub>


  


2


2 2


2


4 1 0


4 1 3 1 0


3 1 0


<i>x</i> <i>x</i>


<i>x</i> <i>x</i> <i>x</i> <i>x</i>



<i>x</i> <i>x</i>

.Từ đó suy ra nghiệm.



<b>C 3 ( 4 0 điểm </b>



Cho tam giác ABC nội tiếp (O), D thuộc BC( D không tr ng B,C) và (O

/

<sub>) tiếp xúc với </sub>



trong với (O) tại K tiếp xúc với đoạn CD,AD tại F,E . Các đường thẳng KF, KE cắt (O)


tại M,N.



a)

Chứng minh rằng MN song song EF



b)

Chứng minh rằng MC tiếp xúc với đường tròn ngoại tiếp tam giác KFC


c)

Chứng minh EF luôn đi qua điểm cố định khi D chạy trên BC



<b>H n d n </b>



<b>(d)</b>


<b>I</b>


<b>H</b>


<b>M</b>


<b>N</b> K


<b>O</b>


<b>O/</b>



<b>B</b> <b><sub>C</sub></b>


<b>A</b>


<b>D</b>
<b>E</b>


<b>F</b>


a)Qua K kẻ tiếp tuyến chung (d) với (O) và (O ) gọi H là giao (d) và BC



/ / EF


<i>KEF</i><i>FKH</i><i>MNK</i><i>MN</i>

.



b) Ta có tam giác HKF cân tại H suy ra

<i>HKF</i><i>HFK</i> <i>MB</i><i>MC</i>

suy ra



</div>
<span class='text_page_counter'>(87)</span><div class='page_container' data-page=87>

<i>AKN</i> <i>AMN</i><i>AIE</i>

nên tứ giác AEIK nội tiếp .



Suy ra

<i>DEF</i><i>EKF</i><i>EAI</i><i>EIA</i><i>EKI</i><i>IKE</i><i>EIA</i><i>IKF</i>

hay

<i>MIF</i><i>IKF</i>

.


Suy ra

<i>MIF</i>

đồng dạng

<i>MKI g g</i>( . )<i>MI</i>2 <i>MK MF</i>.

(1).



Ta có MC là tiếp tuyến (KFC) suy ra

2


. (2)


<i>MC</i> <i>MF MK</i>

.



Từ (1) và (2) suy ra MI = MC .Lúc đó ta có




<i>MIC</i><i>MCI</i> <i>IAC</i><i>ICA</i> <i>MCB</i><i>BCI</i><i>ICA</i><i>BCI</i>

.



Nên CI là phân giác

<i>ABC</i>

, mà AM là phân giác

<i>BAC</i>

nên I cố định



<b> Câu 4</b>

( 1,5 điểm) Cho các số thực x

1

, x

2

, .., x

n

 

0;1

. Chứng minh rằng



2

<sub>2</sub> <sub>2</sub> <sub>2</sub> <sub>2</sub>



1 2 3 1 2 3


1 <i>x</i> <i>x</i> <i>x</i>  ... <i>x<sub>n</sub></i> 4 <i>x</i> <i>x</i> <i>x</i> ....<i>x<sub>n</sub></i>



<b>Giải </b>



Áp dụng BĐT

2


4



<i>A</i>

<i>B</i>

<i>AB</i>

với

<i>A</i>1;<i>B</i>  <i>x</i><sub>1</sub> ... <i>x<sub>n</sub></i>

ta có



2



1 2 3 1 2 3


1

 

<i>x</i>

<i>x</i>

 

<i>x</i>

...

<i>x</i>

<i><sub>n</sub></i>

4

<i>x</i>

<i>x</i>

 

<i>x</i>

...

<i>x</i>

<i><sub>n</sub></i>

với x

1

, x

2

, .., x

n

 

0;1


nên

<i>x x</i><sub>1</sub>

<sub>1</sub>  1

0 <i>x</i><sub>1</sub>2    <i>x</i><sub>1</sub> 0 <i>x</i><sub>1</sub> <i>x</i><sub>1</sub>2

. Tương tự ta có



2 2 2 2 2 2



2 2

;...;

<i>n</i> <i>n</i> 1 2 3

...

<i>n</i> 1 2 3

...

<i>n</i>


<i>x</i>

<i>x</i>

<i>x</i>

<i>x</i>

 

<i>x</i>

<i>x</i>

  

<i>x</i>

<i>x</i>

<i>x</i>

<i>x</i>

<i>x</i>

 

<i>x</i>

.



Suy ra

    

2 

  

2 2 2  2



1 2 3 1 2 3 1 2 3


</div>
<span class='text_page_counter'>(88)</span><div class='page_container' data-page=88></div>

<!--links-->

×